You are on page 1of 103

PLEASE NOTE means very sure means not very sure

The child is 12 years old. In 2 months ago, the direction was over. The parents noticed that the boy
was weak and started drinking more. This condition is gradually worsening and generally lasts
about 2 weeks. Fasting glycemia - 5.7 mmol / l (capillary blood). Which autoantibodies can be
tested to diagnose diabetes?
A. to islands (IAA)
B. to glutamate decarboxylase (anti-GAD);
C. to tyrosine phosphatase (IA-2, IA-2β),
D. endogenous insulin (IAA);
E. to thyroid-stimulating hormone (TRAb) receptors;
F. antibodies to thyroperoxidase (ATPO);
G. antibodies to thyroglobulin (ATTG)

A boy of 16 years complains of a pronounced general weakness, lethargy, thirst, increased


urination, itching of the skin, weight loss, recently contracted epidemic mumps. What antibodies to
glutamate decarboxylase have been established. What is your stage of pathogenesis of type 1
diabetes in this patient now and what will be next?
A. immunological changes with the development of stroke
B. initial immune processes
C. CD1 manifest
D. complete destruction of β-cells
E. Active immune processes
F. complete destruction of alpha –cells

Patient 38 years old, constantly taking glucocorticoids for bronchial asthma. Lately, a thirst has
emerged, appetite has increased, and urination has become frequent. Glycemia during the day: 8.9;
7.7; 9.1; 7.8; 8.1 mmol / L. Which diagnosis options can be considered correct?
A. Steroid Diabetes
B. Functional impairment of carbohydrate metabolism
C. Type 1 diabetes.
D. Type 2 diabetes
E. Secondary diabetes.
F. Diabetes insipidus

Sick 35 years old, complained of dry mouth, thirst, increased urination. Claims that he generally
eats occasionally with frequent snacks, including at night. An oral glucose-tolerance test was
prescribed for diagnosis. What test results will confirm diabetes?
A. Fasting glycemia 5.0 mmol / l after 1 hours - 7.2 mmol / l, 2 hours after eating - 6.0 mmol /
l.
B. fasting glycemia 7.0 mmol / l, after 1 hours - 13,2 mmol / l, 2 hours after eating - 11.0 mmol
/ l.
C. fasting glycemia 6.0 mmol / l, after 1 hours - 11,2 mmol / l, 2 hours after eating - 15,0
mmol / l.
D. on an empty stomach glycemia 5.4 mmol / l, after 1 hours - 7.0 mmol / l, 2 hours after
eating - 7,4 mmol / l.
E. on an empty stomach glycemia 5.8 mmol / l, after 1 hours - 7.1 mmol / l, 2 hours after eating
- 7,0 mmol / l
F. on an empty stomach glycemia 6.2 mmol / l, after 1 hours - 12.2 mmol / l, 2 hours after
eating - 10,4 mmol / l.

A child of 11 years; 20 days after suffering a chicken pox, thirst, urinary incontinence began to
bother. Objectively: general condition of moderate severity, baby lost weight, skin dry, tongue dry,
lined with white bloom, lips dry. In the lungs, breathing is vesicular, heart tones are rhythmic.
Palpation of the abdomen is soft, painless, the liver is slightly enlarged, moderately painful. The
definitions should be:
A. General blood test,
B. General urine test
C. Markers of hepatitis
D. Blood glucose
E. Urine Ketones
F. postprandial glycemia (PPG)

Sick 55 years old, complains of itchy skin, especially in the perineum, frequent candidiasis,
increased appetite. On examination, the doctor diagnosed obesity (body mass index 38 kg / m2,
waist volume 121 cm), arterial hypertension (180/100 mm Hg), fasting glycemia of 6.3 mmol / l.
To determine the diagnosis, you must:
A. Definition of lipidogram.
B. Oral glucose tolerance test.
C. Definition of ketonemia.
D. Definition of acetonuria.
E. Determination of glycated hemoglobin
F. Determination of leptin

A patient of 61 years complains of frequent problems with teeth, recurrent periodontal disease,
constant dry mouth. Has been suffering from hypertension for 12 years. Objectively: abdominal
obesity, body mass index 35.8 kg / m2. In the lungs, breathing is vesicular. Heart tones rhythmic,
blood pressure 170/105 mm Hg, rs-76 / min. Palpation of the abdomen is soft, painless, the liver is
not enlarged. Cautiousness of extremities. On the ECG: signs of left ventricular hypertrophy.
Laboratory examination revealed fasting glycemia - 6.2 mmol / l, hyperlipidemia, GFR of 81 ml /
min / 1.73 m². What additional diagnoses should be suspected.
A. 1 diabetes, ketoacidosis.
B. Type 2 diabetes mellitus.
C. AIDS.
D. Impaired glucose tolerance
E. Impaired glycemic.
F. Prediabetes

Patient 41 years for 19 years with bronchial asthma. In recent years, due to worsening of the
condition and more severe attacks of asthma, prednisolone therapy was started. During hospital
admission, he complained of polydipsia, dry mouth, increased appetite and polyuria. The blood
glucose level was found to be 10.9 mmol / l. Which diagnoses will be correct?
A. Secondary diabetes mellitus.
B. Type 1 diabetes.
C. Type 2 diabetes.
D. Steroid Diabetes.
E. Pituitary diabetes mellitus
F. MODY diabetes

A man of 37 years complains that in 6 months lost 5.5 kg of weight, moderate dry mouth, constant
thirst, polyuria. The complaints described above develop within 8 months. Objectively:
temperature - 36,5C, CDR - 19 per minute, pulse - 72 beats / min, blood pressure - 125/80 mm Hg.
Art. Skin, mucous membranes and tongue of low humidity. Fasting glycemia - 13.4 mmol / l;
glucosuria - 25 g / l, urine response to acetone - negative. Which study will help determine the type
of diabetes?
A. Determination of fructosamine level.
B. Determination of glucagon level.
C. Antibodies to endogenous insulin (IAA);
D. Thyrotropic Hormone Receptor Antibodies (TRAb)
E. Glutamate decarboxylase antibodies
F. Determination of C-peptide

Man N., 58-year-old man has been screened by an endocrinologist. Well-being, no complaints.
Medical history: 5 years ago diagnosed with type 2 diabetes. Dietary and phytotherapy is used to
correct carbohydrate metabolism. Objective status: body temperature 36.5C, blood pressure on the
left arm - 140/85 mm Hg. Art., heart rate 74 ', BH 19'. Fasting glycemia ranges from 4.8-5.8 mmol
/ l; in urine - glucosuria. Which of the following methods do we use to evaluate diabetes
compensation?
A. Determination of glycated hemoglobin
B. Definition of glucosuric profile
C. Definition of glycemic profile
D. Determination of glucose tolerance
E. Fructosamine Level Determination
F. Determination of fasting glycemia

Young woman of 34 years. Complains of moderate dry mouth, constant thirst, polyuria. Has been
ill for about a year during this period, lost 8 kg. Objectively: temperature - 36,6С, CDR - 17 / min,
pulse - 81 beats / min, blood pressure - 120/70 mm of mercury. Art. Low humidity skin and
mucous membranes. Fasting glycemia - 11.9 mmol / l; glucosuria - 20 g / l, urine response to
acetone - 0. What kind of research will help determine the type of diabetes?
A. Determination of glucagon level.
B. Definition of HLA antigens.
C. Determination of insulin level.
D. Glycemic profile.
E. Glutamate decarboxylase antibodies
F. Determination of C-peptide

The 55-year-old patient appeared for another medical examination at a family doctor. Complaints
are not expressed. 3 years ago diagnosed with CD-2. Treatment: Metformin 2000 mg daily.
Objective status: body temperature 36.7C, blood pressure - 140/80 mm Hg. Art., heart rate 76 beats
/ min., heart rate 18 / min. Fasting glycemia does not exceed 6.4 mmol / l; in the urine: glucose - 0,
ketone bodies - 0. Which of the following methods is most informative for evaluating diabetes
compensation?
A. Definition of glucosuric profile
B. Definition of glycemic profile
C. Determination of glycated hemoglobin
D. Determination of glucose tolerance
E. Fasting glucose determination
F. Determination of fasting glycemia

The endocrinologist was approached by the parents of a child of 8 years concerned about the
frequent urination of the child. According to the parents, over the past two months the baby lost
weight by 4 kg, started drinking more, complaining of fatigue, drowsiness. On examination: the
height of the child 142 cm, weight 32 kg, sunken reddish cheeks against a pale face, tongue dry, the
smell of acetone from the mouth, skin turgor reduced, traces of cracks in the groin area. The
thyroid gland does not palpate. Internal organs without features. Daily diuresis of about 1.9-2.2
liters. What should be done in the laboratory rapid examination to verify the diagnosis?
A. General blood test, general urine test
B. Pituitary thyroid hormone
C. Nechyporenko test
D. Blood glucose,
E. Urine ketones
F. Biochemical test

Patient O., 61, went to the doctor complaining of an increase in the frequency of urination and
itching in the groin. On examination: BMI 34.1 kg / m2, moderate skin, no stretch, internal organs
without features, diuresis about 2.5 liters. Laboratory: fasting glycemia 11.2 mmol / l, glycated
hemoglobin 9.6%. ZAC without features. Establish the components of the diagnosis.
A. Type 1 diabetes, decompensation
B. Obesity 1st degree
C. Type 2 diabetes, decompensation
D. Obesity 2nd degree
E. MODY diabetes
F. Type 1 diabetes, subcompensation

A 60-year-old man was consulted by a family doctor for the purpose of preventive examination.
Feels good, no complaints. Objectively: height 178 cm, weight 115 kg, BMI 36.3 kg / m2,
distribution of subcutaneous adipose tissue uniform, dry tongue, thyroid does not palpate, lungs and
heart without features. Laboratory: ZAC without features, fasting glycemia (capillary blood) 6.1
mmol / l. What is recommended for the patient?
A. Ultrasound of internal organs
B. Glucose tolerance test
C. Glycated hemoglobin
D. Definition of thyrotropic hormone
E. Nothing, patient is healthy
F. Ultrasound diagnosis of the lower extremity

A 54-year-old woman went to a cardiologist for an increase in blood pressure; Objectively: BMI
33.8 kg / m2, heart - boundaries shifted to the left, heart rate 72 beats / min, blood pressure 155/100
mm. Hg Art .; lungs - vesicular breathing, without pathological noises .; abdomen - soft, painless,
in the groin area traces of lacerations. Daily diuresis of 2-2,5 l. Laboratory: ZAK Er - 4.2 1012 / l,
Le - 6.4 109 / l, glycemia - 10.9 mmol / l. What laboratory indicators will allow you to estimate the
average sugar level in the last 3 months and another in 2-3 weeks.
A. Hemoglobin general blood test
B. Fructosamine
C. GAD Antibodies
D. Glycated hemoglobin
E. Glucose Tolerance Test
F. Level of C-peptide

Patient L., 44, referred for consultation in recurrent boils. Preliminary data on fasting glycemia:
5.9- 6.8 mmol / l. Glucosuria in the night portion of urine - 0. Which of the following examinations
should the patient carry out to assess the state of carbohydrate metabolism?
A. Repeated definition of fasting glycemia.
B. Conduct standard GTT.
C. To determine postprandial glycemia.
D. Determine glucose in daily urine.
E. Glycated hemoglobin
F. Determine glucose in daily blood

In a 28-year-old woman, fasting glycemia for the first time was 7.8 mmol / l, postprandial - 11.9
mmol / l, glucosuria 1.5%, ketonuria (+). What are the possible dianosides to suspect:
A. Type 1 DM.
B. Type 2 diabetes mellitus.
C. Gestational diabetes.
D. Symptomatic diabetes.
E. CD of unspecified genesis.
F. MODY diabetes

Patient Y., 27 years old, became ill with diabetes for the first time. Among the examinations, the
doctor prescribed a C-peptide and immunoreactive insulin (IRI) blood test. What is the purpose of
this survey?
A. CD2 severity prediction.
B. Differentiation of the diagnosis of diabetes.
C. Determination of the starting dose of insulin preparations.
D. Correction of insulin therapy regimen.
E. Functional status assessment of beta cells.
F. Fasting glucose and PPG

Patient U., 33, complains of persistent dental problems and periodontal disease that is untreatable.
Examination data: fasting glycemia (capillary blood) 6.4 mmol / l, 2 hours after eating 10.2 mmol /
l. Glucose is absent in the morning urine. Which of the following surveys will help you evaluate
your carbohydrate metabolism in the previous period?
A. Determination of fructosamine.
B. Determine glucose in daily urine.
C. Determine glycated hemoglobin.
D. To carry out TSH.
E. Determine the concentration of antibodies to glutamate decarboxylase.
F. For antibodies to TSH receptors.

A woman of 57 years suffers from obesity. For a long time complains of itchy vagina. He is
treated by a gynecologist in connection with candidiasis of the vulva. Glucosuria is absent. Fasting
glycemia three times: 4.4; 6,3; 5.1 mmol / L. Which of the following examinations may reveal the
root cause of the patient's suffering?
A. Determine glycated hemoglobin.
B. To determine the sensitivity of candida to medicines.
C. Conduct TSH.
D. Determine daily glucosuria.
E. Determine protein concentration in urine
F. Conduct FT4

The parents of a boy of 6 years have addressed to the endocrinologist with complaints: for the last
month the child grew polyuria, thirst, weight loss up to 4 kg. It is known that earlier, about 1 month
ago was diagnosed with diabetes 1type, but parents have refused insulin therapy. Objectively: pale
skin, blush on the cheeks, dry tongue, acetone odor in the mouth. What lab tests should be done
first?
A. Fasting glycemia
B. Ketone bodies in urine
C. Glucosuria
D. General blood test
E. General urinalysis
F. Fructosamine

A 16-year-old girl found 6 g / l glucose for the first time in the urine. A history of a viral infection.
No complaints. Objectively: BMI 19.8 kg / m2, heart - no deviations, heart rate 71 beats / min,
blood pressure 110/70 mm. Hg Art .; lungs - vesicular breathing, without pathological noises .;
abdomen - mild, painless. Fasting glycemia - 5.3 mmol / l. Which of the studies most exclude
diabetes?
A. Glucose tolerance test.
B. Daily fluctuations in glycemia.
C. Plasma insulin levels.
D. Daily glucosuria.
E. Fructosamine
F. postprandial glycemia (PPG)

Patient R. 35 years old, suffering from diabetes for 20 years. Receives insulin therapy. Repeatedly
in the history of a coma. Worried about visual acuity, weakness and pain in the legs. Objectively:
BMI 25.8 kg / m2, heart - no deviations, heart rate 71 beats / min, blood pressure 120/80 mm. Hg
Art .; lungs - vesicular breathing, without pathological noises; abdomen - soft, painless. At
ophthalmoscopy on the fundus revealed microaneurysms of the vessels, hemorrhages in the retina,
neovascularization of the retina. Fasting glycemia 9.9 mmol / l. Glycated hemoglobin 7.9%, no
ketone bodies in urine. Make the right diagnosis (combine the right components of the diagnosis):
A. Decompensated
B. Severe form
C. Type 1 diabetes mellitus,
D. Subcompensated
E. Medium weight
F. Compensated

The parents of the twin children addressed the doctor. One girl, A., 10 years old, had type 1
diabetes 2 years ago. Last month, everyone in the family contracted pneumonia. Another girl S.
has no appetite for the last week and is constantly nauseous and started to get up at night. The skin
is dry, pale. Pulse rate - 70 / min, blood pressure - 100/65 mm Hg. Art. The palpation of the
abdomen responds. Liver +2 cm. What examination is necessary to verify the diagnosis?
A. HOMA-IR
B. Fructosamine
C. BAD
D. Ketone bodies in urine
E. Glucose Tolerance Test
F. postprandial glycemia (PPG)

Patient 34 years old complained of thirst, dry mouth, weight loss, increased urination. Weight 95
kg, height 168 kg, BMI - 33,65 kg / m2, НВА1с - 8,2%, fasting glycemia 7,9 mmol / l, urine sugar
1,5%, acetone 0. C-peptide 130 (298, 0-2350) pmol / l, insulin - 3.58 (2-29.1) mcU / dl, blood
pressure to GADA - 697.0 (up to 10) mg / ml. To appoint adequate therapy:
A. Prescribe diet therapy and biguanides
B. To prescribe diet, biguanides, and medium-duration insulin
C. To prescribe diet therapy, biguanides and sulphonylureas
D. Prescribe diet and basic insulin therapy
E. Prescribe diet, long-acting insulin, and short-acting insulin (Glargin + Aspart)
F. Prescribe diet and short-acting insulin

Patient K., aged 19, has diabetes of type 1 diabetes for 3 years, complains of severe thirst, dry
mouth, more in the morning. Receives insulin therapy of Actropid NM 12 of Protafan NM 16 in the
morning, Actropid of NM 8 in the afternoon, Actropid of NM 8 in the evening and Protafan NM of
12 in the evening. HbA1c - 7.9%, glycemic profile 8.00-7.4 mmol / l; 11.00 - 13.3 mmol / l;
13.00-14.9 mmol / l; 16.00-7.1 mmol / l; 21.00 - 7.6 mmol / l; 3.00 - 5.8 mmol / l. What is your
treatment tactic?
A. No changes are needed
B. Strengthen dietary regimen
C. Increase Morning Actropid NM and Morning Protafan NM
D. Strengthen dietary regimen and increase morning Actropid NM and morning Protapan NM
E. Increase Morning Insulin Protafan NM

Patient G., 37 years old complains of frequent hypoglycemic States, mainly at night and in the
morning were 2 hypoglycemic coma with the arrival of the emergency medical service carriage. A
diabetes type 1 from 10 years of age, labile course with frequent ketoacidotic States. Gets Aktrapid
NM insulin 10 IU Protofan HM 18 IU in the morning, Aktrapid NM 10 IU at lunch, 10 IU Actrapid
NM and Protofan HM 16 IU in the evening. Recently increased the complications of the kidneys.
Urine test: urine specific gravity -1020; R-tion-on natural hair.; protein - 0,99%.; epithelium 1-2 in
p/s; L - 3-5 n/vision, red blood cells – 3-5 in p/z, hyaline cylinders-3-4 to p/z, mucus (-), salt (-).
Blood biochemistry: creatinine-145,9 µmol/l; urea 10.6 mmol/l, ALT-24.5 mmol/L. h; AST is 21.3
mmol/L. h, BLP – 31 mind.ed., cholesterol was 3.56 mg/DL. Your treatment tactics?
A. Strengthen diet (diet No. 9/7)
B. To consult nephrologist
C. Transfer on analog forms of insulin (basis–bolus and ultrashort)
D. Transfer on analog forms of insulin (basis-bolus and ultrashort) and a consultation of the
nephrologist
E. Transfer the patient to hemodialysis and analog forms of insulin (basis-bolus and ultra-short)

Patient L., 38 years, suffer from type 1 diabetes for 12 years. After hypothermia became ill with
pneumonia. Health is relatively satisfactory, but he suffers from cough, chest pain, recurrent thirst,
dry mouth, increased urination. Gets Apidra (Gluin) morning, 10 IU, 12 IU lunch, 10 IU in the
evening НвА1с of 7.3%. Glycemic profile: 8.00 - 13,1 mmol/l; 11.00 - 12.7 mmol/l; 13.00-13,6
mmol/l; 16.00-12.3 mmol/l; 21.00 – 14.9 mmol/l; 3.00 – 12,4 mmol/l. Urine sugar 1.5%, acetone
(+). Choose the treatment tactics.
A. Change of therapy not needed
B. Strengthen diet regime
C. To increase each injection to insulin by 2-4 units
D. To increase the insulin Apidra on 2-4 units.
E. Transferred to the short-acting insulin prior to the completion of pneumonia

Patient F., 36 years old with type 1 diabetes for 8 years, receiving insulin therapy Pharmasulin H 12
IU and Pharmasulin HNP 18 IU (8.30), lunch Pharmasulin H 14 IU (13.30), evening Pharmasulin H
10 IU and Farmasulin HNP 14 units (6.30pm). Patient HBA1c 8.4%. glycemic profile: 8.00- 14.1
mmol / l; 11.00 - 8.7 mmol / l; 13.00-5.7 mmol / l; 16.00-8.3 mmol / l; 21.00 - 8.9 mmol / l; 3.00
- 4.4 mmol / L. Urine sugar 0.5%, acetone (0). Is it advisable to change treatment tactics?
A. No change in therapy needed
B. Strengthen dietary regimen
C. Increase dietary regimen and reduce evening dose of Pharmasulin HNP
D. Increase dietary regimen Increase the dose of evening Pharmasulin HNP
E. Increase dietary regimen and shift evening dosage of Pharmasulin HNP to 22.00-23.00

Patient B., 31, complains of recurrent hypoglycemic conditions that are unrelated to physical
activity or eating disorders that occur 2 hours after dinner. The patient receives insulin therapy.
Actropid NM 12 IU, Protafan NM 20 IU, Actropid NM 8 IU, Actropid NM 12 IU and Protafan NM
14 IU in the evening. HbA1c - 7.6%, glycemic profile 8.00-7.4 mmol / l; 11.00 - 8.3 mmol / l;
13.00-6.8 mmol / l; 16.00-7.3 mmol / l; 21.00 - 3.6 mmol / l; 3.00 - 8.2 mmol / L. What is your
treatment tactic?
A. Reduce dinner calories
B. Increase calorie supper
C. Increase the insulin dose of Actropid NM in the evening
D. Reduce insulin dose of Actropid NM in the evening
E. Increase calorie supper and reduce insulin dose Actropid NM for the evening
Patient V., 23 years old, recently became ill with type 1 diabetes. In the CRL where the patient was
treated, she was prescribed insulin therapy with Pharmasulin H 30/70 at a dose of 30 units (18 in the
morning and 12 in the evening). The constitution is normostenic, height 165 cm, weight - 64 kg,
BMI - 22,9 kg / m2, НВА1с - 8,6%. glycemic profile 8.00-8.4 mmol / l; 11.00 - 13.1 mmol / l;
13.00-11.8 mmol / l; 16.00-16.2 mmol / l; 21.00 - 6.6 mmol / l; 3.00 - 10.5 mmol / L. Your
treatment tactics.
A. Strengthen dietary regimen
B. To translate into the basis-bolus scheme of insulin therapy
C. Switch to insulin short and medium in the morning, short in the afternoon, short and
medium in the evening
D. Increase the evening dose of Farmasulin H 30/70
E. Increase morning and evening doses of Farmasulin H 30/70

Which of the following is characteristic of type 1 diabetes?


A. Onset of the disease in old age.
B. The gradual beginning.
C. Weight loss.
D. Need for insulin therapy.
E. Steady flow.

The level of C-peptide is usually:


A. Reduced for type 2 diabetes.
B. Reduced for type 1 diabetes.
C. Reduced in diabetes with LADA.
D. Activated by insulin therapy.
E. Increases by glucagon.

Determination of the level of immunoreactive insulin is carried out:


A. Persons receiving continuous insulin.
B. Patients with a tendency to labile diabetes.
C. In individuals who have never received insulin.
D. Patients with type 1 diabetes are first identified.
E. Diabetic patients with a tendency to ketonuria.

Which of the following insulins is characterized by the onset of action 0.5-1 h, peak activity 2-3 h,
duration of action 5-8 h:
A. Humulin P
B. Protafan NM
C. Actropide NM
D. Humulin 30/70
E. Insuman Combe 25

Which of the following insulins is characterized by an onset of 0.5-1.5 h, a peak of 4-6 h, a


duration of 8-16 h:
A. Insuman Basal
B. Insuman Rapid
C. Novorapid (Aspart)
D. Protafan NM
E. Insuman Combe 25

Which of the following insulins are for two-phase drugs?


A. Actropid NM
B. Lantus (Glorgin 100)
C. Mixtard NM
D. Protafan NM
E. Insuman Comb 25

Which of the following insulins are recombinant short-acting insulin analogues


A. Humologist (Insulin Lispro)
B. Lantus (Glorgin 100)
C. Togeo (Glorgin 300)
D. Protafan NM
E. Novorapid (Aspart)

Which of the following insulins are recombinant long-acting insulin analogues


A. Togeo (Glorgin 300)
B. Tresiba (Degluedek)
C. Humal Mix 25
D. Protafan NM
E. Novorapid (Aspart)

Which of the following insulins are recombinant analogues of combined action insulin:
A. Levemir (Detemir)
B. Raisodegh
C. Novomix 30
D. Humalog (Lispro)
E. Novorapid (Aspart)

In which cases are porcine monocomponent insulins allowed?


A. Patients with type 1 diabetes
B. In patients with type 2 diabetes in intercurrent diseases
C. Patients with type 2 diabetes during surgery
D. In patients with LADA diabetes
E. In patients with MODY diabetes

Patient M., 26 years old with type 1 diabetes 1 year, receives 20 U of insulin in the morning and 12
U of the same insulin in the evening; complains of dry mouth, thirst for the day. Fasting glycemia -
6.1 mmol / l, daily glucosuria - 36 g / l. HbA1c -9.4%. The patient adheres to the diet. What
treatment should be prescribed?
A. Increase insulin dose in the morning
B. Reduce your insulin dose in the morning
C. Increase insulin dose in the evening
D. Switch to basic bolus insulin therapy
E. Add the short acting insulin 3 times daily to the above insulin

Patient P., 32 years old, ill with type 1 diabetes for 4 months, treated with Protafan NM at a dose
of 36 from the morning, which he was prescribed in the CDL. What method of administration does
the patient need for insulin?
A. Insert Protafan NM twice a day
B. Insulin Protafan NM in the morning and evening and short-acting insulin at lunch
C. Combine Protafan NM insulin with short acting insulin twice daily
D. Insulin Protafan NM twice daily and short-acting insulin before each meal
E. Prescribe basis- bolus insulin therapy to the patient

What are the benefits / disadvantages of intensive insulin therapy over traditional insulin therapy?
A. You may not adhere to the diet regimen
B. Can not limit digestible carbohydrates
C. Improves diabetes control
D. Diabetes control is worsening, and the risk of severe hypoglycemia is reduced
E. Improved diabetes control, increased risk of hypoglycemia

The girl, whose parents are ill with diabetes, experienced nausea, vomiting, weakness and the smell
of acetone in her mouth. In order to prevent diabetic coma in the baby, his father injected her with
8 IU of Protafan NM. How true are your father's actions?
A. Certainly true
B. The girl's hereditary predisposition to diabetes and the introduction of Protafan will prevent
its development
C. It is better to use oral antipyretics
D. The cause of the vomiting and odor of acetone may not be diabetes related
E. The father should go to an infectious hospital to check for glycemia and acetonuria and
prescribe corect treatment

Patient P., 34 years old, suffering from type 1 diabetes for 6 months, receiving 20 U of intermediate
insulin in the morning and 12 IU of the same insulin in the evening, complaining of dry mouth,
thirst for the day. Fasting glycemia of the heart - 5.6 mmol / l, glucosuria - 36 g / l, necturia is
absent. The patient adheres to the diet. What treatment should be prescribed?
A. Increase insulin dose in the morning
B. Reduce your insulin dose in the morning
C. Increase insulin dose in the evening
D. Add short-acting insulin three times a day
E. Transfer to base- bolus insulin therapy

Patient with type 1 diabetes uses intensive insulin therapy with the insulin Levemir (Detemir) and
Epidra (Gulizin). Insulin Epidra titrates with respect to the number of consumed cereal units with
physical activity adjustment. Excludes products with high glycemic index as much as possible.
Despite this regimen, diabetes mellitus remains unsatisfactory, HBAc - 8.5%, and hypoglycemia is
common, especially at night. What do you recommend to the patient?
A. Continue to apply the previous mode
B. Apply a combination of Detemir (Levemir) and Asparta (Novorapid)
C. Lantus (Glargin) replaced by Tresiba (Degludek)
D. Transfer patient to insulin pump
E. Apply only Gulizin (Epidra)

Patient K., 24 years old, ill with type 1 diabetes for 2 years, receiving insulin Farmasulin H 30/70
28 IU before breakfast and 24 IU before dinner. Glycemic profile 8.00- 7.4 mmol / l; 11.00 - 8.7
mmol / l; 13.00-7.9 mmol / l; 16.00-15.4 mmol / l; 21.00 - 7.9 mmol / l; 3.00 - 8.8 mmol / l.
Urine sugar 1.5%, acetone (0), HBAc - 8.8%. What is your tactic?
A. Reduce carbohydrates.
B. Add to Dinner Pharmasulin H 6-8 IU
C. Combine insulin therapy with biguanides
D. Combine insulin therapy with Pharmasulin H and HNP in the morning, Pharmasulin H in
the afternoon, Pharmasulin H and HNP in the evening
E. Change the dosage of Pharmasulin H 30/70 before breakfast by 24 IU, before dinner by 28
IU

Patient K., 25 years old, suffering from type 1 diabetes, receives in the morning 16 U of insulin
Actropid NM and 20 IU of Protafan NM, 6 IU of Actropid NM at lunch, 6 IU of Actropid NM and
10 IU of Protafan NM in the evening. 2-3 times a week the patient has symptoms of hypoglycemia
2 hours after breakfast. What is your tactic?
A. Increase dose of XO in the morning.
B. Decrease the dose of early Actropid NM.
C. To reduce the dose of earlier Protafan NM.
D. Increase the dose of earlier Protafan NM.
E. Reduce evening dose of Protafan NM

Patient I., 49 years old, ill with type 2 diabetes for 8 years, from the time of the disease constantly
takes glibenclamide 5 mg 3 times a day. She has not consulted an endocrinologist for the past three
years. Over the last year, she has lost 9 kg without motivation, however, she has not caused this
weight loss. When examined, body weight 81 kg, height 166 cm, BMI - 29 kg / m2. Laboratory:
fasting blood sugar 12.0 mmol / l, after eating - 15 mmol / l, glycated hemoglobin - 13.2%. Other
laboratory parameters within normal limits. Choose treatment tactics:
Add biguanides to glibenclamide.
Replace glibenclamide with metformin
Cancel glibenclamide and assign III generation sulfonylureas
Transfer to insulin therapy with prolonged insulin.
Translate to combination with NSCTG 2 inhibitors

55-year-old female patient with type 2 diabetes for 5 years. It is treated with metformin at a dose
of 850 mg three times a day. Glycemia of the heart7.8 mmol / l, after two hours after seeding - 9.5
mmol / l, glycated hemoglobin - 8.3%. In ZAS - proteinuria up to 0,099% protein. On examination
- height 167 cm, weight 88 kg. (BMI - 31.6 kg / m2). Consulted by a cardiologist: Hypertension II
century. Assign treatment.
Do not change therapy, but recommend careful adherence to diet # 9.
Combine metformin with prolonged insulin
Translate into combination with sulfonylurea derivatives.
Keep the daily dose of metformin unchanged
Add NZKTG-2 inhibitors to the antipyretic therapy

Female 49 years old, suffering from type 2 diabetes 6 years. It is treated with metformin at a dose
of 500 mg /day. Glycemia on an empty stomach 8.2 mmol / l, two hours after a meal - 14.5 mmol /
l, glycated hemoglobin - 8.9%. In urine test - proteinuria is up to 0.033% of protein. On
examination - height 162 cm, weight 102 kg. (BMI - 39.2 kg / m2). Make an adjustment to
diabetes.
Do not change therapy, but recommend careful adherence to diet # 9.
Combine metformin with thiosolidinediones
Add GPP-1 agonists to metformin.
Translate into combination with sulfonylurea derivatives.
Increase daily dose of metformin to 3000 mg /day

Woman 58 years suffering from diabetes15 years old. Uses glimepiride at a dose of 4 mg / day.
Strictly adheres to the diet regime. History - coronary heart disease, angina pectoris II FC. In the
laboratory, it was diagnosed: fasting glycemia 8.2 mmol / l, postprandial - 9.3 mmol / l, glycated
hemoglobin 8.5%. Physical examination - BMI 24 kg / m2. Perform therapy correction.
Increase daily dose of glimepiride to 6mg and add repaglinide.
Glimepiride is combined with metformin at a dose of 500 mg per day.
Convert to Metformin at a dose of 2000 mg per day
Dietary regimen, insulin therapy with prolonged insulin.
Add NZKTG-2 (emphagliflozin) inhibitors to treatment

Patient 64 years old, suffering from type 2 diabetes mellitus. Receives 15 mg glibenclamide and
2000 mg metformin daily. After physical activity I felt weakness, dizziness, shortness of breath,
slight pain in the heart area. A / T 70/30 mm Hg Pulse rate - 110 beats / min. According to the
ECG: domed rise of the CT segment, depression of the CT segment in the reciprocal zone,
appearance of the QS tooth. The patient was admitted to the infarct ward. What is the patient's
tactics for diabetes?
Convert to insulin therapy with short insulin in 4 injections
Leave previous therapy and add prolonged insulin
Cancel metformin
Cancel glibenclamide
Prescribe emfagliflozin
Prescribe dapagliflozin

Woman 57 years old, with obesity 3 tbsp. diagnosed with diabetes two months ago.
endocrinologist recommended sub-calorie diet and metered exercise. fasting glycemia level of 9.2
mmol / l. Which antipyretic drug is not advisable to recommend to the patient?
Biguanides
Derivatives of sulfonylureas
Alpha-glucosidase inhibitors
Meglitinides
Thiazolidinediones
Sodium-glucose transport protein 2 inhibitor (SGLT2)
Dipeptidyl peptidase-4 inhibitor (DPP-4)

Patient B, age 57 years. During the last three years, he has been receiving 60 U of insulin a day for
the treatment of diabetes. Prior to this, he was treated with antipyretic drugs from the group of
sulfonylureas. Due to excessive appetite, over the past 6 months he has weighed 6 kg. Vascular
complications progress. Fasting blood sugar is 11 mmol / l, the level of glycated hemoglobin is
9.4%. Frequent nocturnal hypoglycemia. Please enter the wrong diagnosis:
Type 2 diabetes, secondary insulin dependent, moderate in decompensated state.
Type 2 diabetes, secondary insulin dependent, severe, decompensated. Somoji syndrome.
Type 2 diabetes, severe, subcompensated.
Type 1 diabetes, severe, decompensated.
None of the above
Type 2 diabetes, moderate in subcompensated state

A 57-year-old patient, BMI 29 kg / m2, referred to a gynecologist with complaints of itching in the
area of the inner thighs and external genitals. In history - transmural MI (PV 55%).
Prophylactically, a blood test for sugar and glycated hemoglobin revealed glycemia of 8.7 mmol / l,
glycated hemoglobin of 9.2%, and no acetone in the urine. What is your healing tactic?
Diet therapy.
Biguanides.
II generation sulfonylurea derivatives (gliclazide MR).
Short-acting insulin administration.
NZKTG-2 preparations (emphagliflozin)
Diet therapy, biguanides, short-acting insulin administration

64-year-old female patient with type 2 diabetes mellitus treated with 4 mg / day glimepiride.
Glycemic ulcer 7.8 mmol / l, after two hours after the treatment - 10.5 mmol / l. Consulted by an
ophthalmologist. Conclusion - "Diabetic proliferative retinopathy." What is the further treatment
of the patient?
Laser photocoagulation of the retina
Immediate transfer to insulin therapy with prolonged insulin.
Prescribe Metformin.
Transfer to combination with NSCTG 2 inhibitors
Translate to combination with GPP-1 agonists
Prescribe II generation sulfonylurea derivatives
Patient with type 2 diabetes, 59 years old, is treated with glibenclamide at a dose of 15 mg / day.
Frequent elevations of blood pressure, persistent cardialgia, especially at night, sweating, constant
feeling of hunger are worried. What can this patient's condition not be related to?
With unstable angina.
With climacteric neurosis.
With hypoglycemia.
With psycho-neurotic syndrome.
With angina of rest
With stable angina
With hyperglycemia

Which of the following methods is required in the treatment of all clinical forms of type 2 diabetes?
Diet therapy.
Treatment with metered exercise
Physiotherapy treatment.
Treatment of sugar-lowering tableted drugs.
Short-acting insulin
~ Long-acting insulin

A 45-year-old woman came to a consult an endocrinologist with concern about the possibility of
developing type 2 diabetes. Complained of moderate dry mouth and thirst. From the anamnesis of
life it is known that heredity in endocrinopathy is not burdened. When viewed BMI - 23.2 kg / m2.
In biochemical analysis of blood deviations from the norm were not detected. Fasting glycemia -
5.2 mmol / l. Choose high-risk groups for type 2 diabetes.
First degree of affinity with diabetic patients.
People with impaired fasting glycemia.
People with hypertriglyceridemia.
Women with polycystic cervical syndrome and BMI ≥ 30 kg / m2.
People with acute pancreatitis
People with hypoglycemia
Women with polycystic cervical syndrome

A 45-year-old woman came to see an endocrinologist for complaints of dry mouth and thirst. From
the anamnesis of life it is known that his father had type 2 diabetes. When viewed BMI - 21.2 kg /
m2. In biochemical analysis of blood deviations from the norm were not detected. Fasting
glycemia - 9.2 mmol / l, glycated hemoglobin - 8.4%. The doctor prescribed her glyclazide at a
dose of 60 mg / day. What mechanism of hypoglycemic action is not inherent in the drugs of this
group?
Increasing the sensitivity of peripheral insulin-dependent tissues to insulin action
Inhibition of glucose reuptake in the distal nephron tubules
Stimulation of exocytosis of insulin by beta cells of a pancreas.
Increased glucagon secretion.
Reduced intestinal glucose uptake
Decrease glucagon secretion
Increased intestinal glucose uptake

Patient B, 47, had diabetes for the first time. On examination: fasting glycemia 8.6 mmol / l,
postprandial - 13.2 mmol / l, glucosuria 2%, HbA1c level 9.3%. Make the wrong diagnosis.
Type 2 diabetes, first detected in a state of subcompensation.
Type 2 diabetes, first detected in a state of decompensation.
Type 2 diabetes, mild form in decompensated state.
Type 2 diabetes, mild form in a state of subcompensation.
Type 2 diabetes, moderate in decompensation
Type 2 diabetes, severy form in a state of decompensated
Type 2 diabetes, severy form in a state of subcompensation

A 52-year-old patient with diabetes who was first diagnosed with 2 overweight (BMI - 28.6 kg /
m2) was diagnosed with fasting glycemia 7.2 mmol / l, postprandial 11.3 mmol / l, glycated
hemoglobin level - 7,6%. Choose appropriate treatment tactics.
Assign biguanides.
Lifestyle modification (diet therapy + metered exercise)
Recommend sulfonylureas derivatives.
Recommend GPP-1 agonists
Recommend α-glucosidase inhibitors

Patient 52 years old, suffering from type 2 diabetes for 7 years. For the medical treatment of
diabetes takes metformin at a dose of 1000 mg/day and gliclazide at a dose of 60 mg/day. The last
6 months have been a serious concern for complaints of pain, numbness and freezing in the legs. At
additional examination: BMI - 34 kg / m2, blood pressure 160/100 mm Hg, GFR - 78 mm / h / 1,73
m2, the level of glycated hemoglobin 9,7%. Choose the appropriate method of diabetes treatment.
Increase metformin dose to maximum.
Leave gliclazide in the same dose
Apply a DPP-4 inhibitor instead of gliclazide
Appoint a GPP-1 agonist instead of metformin
Add NSCTG 2 inhibitor to these drugs
Decrease metformin dose

Patient, 43 years old, suffering from type 2 diabetes for 6 months, adheres to diet therapy. On
examination: fasting glycemia 5.5 mmol / l, postprandial-7.6 mmol / l, HbA1c 6.8%, daily urine
glucosuria 0%. The level of blood pressure 125/80 mm Hg The correct diagnosis is chosen.
Type 1 diabetes, moderate in compensation. Honeymoon.
Type 2 diabetes, moderate in compensation.
Type 2 diabetes, first identified in a state of subcompensation.
Type 1 diabetes, mild in compensation.
Diabetes mellitus type 2, easy to compensate for
Type 1 diabetes, moderate in subcompensation
Type 2 diabetes, first identified in a state of decompensation

Which of these sugars-reducing drugs doesn’t increase the risk of acute pancreatitis?
Metformin.
Glimepiride.
Nateglinide.
Gliclazide.
Exenatide
Emphagliflozin)
Dapagliflozin

Specify the mechanism of action other than the biguanides.


Stimulation of glucagon production.
Stimulation of insulin exocytosis from beta cells of pancreatic islets.
Enhancement of insulin action at receptor and post-receptor levels in insulin-dependent tissues.
Inhibition of sodium dependent type 2 glucose counterpart in distal nephron tubules
Inhibition of type 4 dipeptidyl peptidase enzyme
Inhibition of glucagon production
Attenuation of insulin at receptor and postreceptor levels in insulin-dependent tissues

Specify the mechanism of antipyretic action not peculiar to agonists of GPP-1.


Stimulation of glucagon production.
Stimulation of insulin exocytosis from beta cells of pancreatic islets.
Enhancement of insulin action at receptor and post-receptor levels in insulin-dependent tissues.
Inhibition of sodium dependent type 2 glucose counterpart in distal nephron tubules
Inhibition of type 4 dipeptidyl peptidase enzyme
Attenuation of insulin at receptor and postreceptor levels in insulin-dependent tissues
Inhibition of glucagon production.

Specify the mechanism of antipyretic action not peculiar to the sulfonylurea derivatives:
Stimulation of glucagon production.
Stimulation of insulin exocytosis from beta cells of pancreatic islets.
Enhancement of insulin action at receptor and post-receptor levels in insulin-dependent tissues.
Inhibition of sodium dependent type 2 glucose counterpart in distal nephron tubules
Inhibition of type 4 enzyme dipeptidyl peptidase
Decrease in intracellular calcium concentration
Reducing the number of insulin receptors

Choose the correct statements for the mechanism of action of biguanides:


Decreased gastrointestinal glucose absorption.
Inhibition of gluconeogenesis in the liver.
Increased tissue sensitivity to insulin at the periphery.
Reduction of TG, CBS, LDL HD.
Beta cell apoptosis proliferation and inhibition
Increased gastrointestinal glucose absorption
Decreased tissue sensitivity to insulin at the periphery

A patient with type 2 diabetes with obesity wants to lose weight. What slimming recommendations
are wrong?
Do not change your eating habits, only reduce the amount of salt consumed.
Do not change your eating habits, but exclude alcohol.
Start doing morning gymnastics daily.
Do not change eating habits, but enter 2 days off per week.
Hypocaloric diet and exercise needs to be adhered to

Indications for insulin therapy for type 2 diabetes are:


Diabetic ketoacidosis.
Diabetic foot syndrome.
Coronary heart disease.
Pregnancy and lactation.
Surgery
Chronic pancreatitis
Arterial hypertension

A diabetic patient who was feeling well suddenly lost consciousness at 11 o'clock. It is known that
the patient did not eat breakfast after insulin administration. Doctor's tactics?
A. Enter glucagon 1.0 in / m.
B. Enter 20-40ml. 40% glucose intravenously jet.
C. Call a lab technician and wait for blood test results.
D. Set up an intravenous drip of glucose with insulin.
E. Introduce hydrocortisone 2.5% - 2.0 v / m
F. Introduce NaCl 0.9% - 2.0 v / m

The patient of 48 years was admitted to the gynecological ward with uterine bleeding. For 5 years,
she was observed by a gynecologist for fibromyoma. Three years ago, the patient was diagnosed
with diabetes 2-t, which was offset by diet and diabetes 1t / d. Blood glucose is 8 mmol / l, urine
glucose is 0.5%, and the reaction to acetone is negative. The patient is shown surgical treatment.
What is your tactic?
A. Schedule a diet.
B. Convert short-acting insulin monotherapy.
C. In addition to diabetes appoint maninil.
D. To appoint a combination of short and prolonged insulin.
E. Leave Diabetes Sick.

Woman 62 years old with obesity. Diabetes sick 22 years. Constantly takes oral antipyretics.
Diabetes in a state of unstable subcompensation. Strong epigastric pain, constipation, abdominal
distension, unformed stool are disturbed. Possible diagnosis?
A. Chronic pancreatitis.
B. Chronic gastritis.
C. Chronic cholecystitis.
D. Diabetic hepatosis.
E. Diabetic gastropathy
F. Diabetic nephropathy
ination of For
hypoglycemic coma use all of these drugs, except:
the elimination of hypoglycemic coma use all of these drugs, except:
A. 40% glucose.
B. Insulin.
C. Cordyamine.
D. Prednisone.
E. Glucagon.

In a pregnant woman who has not previously had diabetes, glycemia 7.73 mmol / l, ketonuria. In
the diet regimen glycemia decreased to 7.0 mmol / l, ketonuria remains. What kind of post-
treatment regimen should you choose?
A. Exclude fats and easily digested carbohydrates from the diet.
B. Increase the quota of easily digestible carbohydrates.
C. Add Metformin.
D. Convert to insulin therapy.
E. Combine insulin with glureorm.

Patient with diabetes 2t and obesity patients with pulmonary tuberculosis. Previously received
Metformin 2000 mg / day. What is your tactic?
A. Continue oral therapy.
B. Schedule a diet.
C. Convert to insulin therapy.
D. Add diabetone to maninil.
E. Add Victose
F. Add lixisenatide

In the treatment of lactic acidosis use all except:


A. Low dose insulin therapy.
B. High-dose insulin therapy.
C. Intravenous intake of 40% glucose.
D. Intravenous drip sodium hydrocarbonate.
E. Oxygen inhalations

Pregnant patient with diabetes mellitus in a state of precoma is hospitalized to the intensive care
unit. Which insulin therapy regimen would you choose?
A. Low dose insulin regimen
B. Baseball therapy.
C. Large doses
D. 5% glucose with short acting insulin intravenously

For the treatment of diabetic ketoacidosis, use:


A. Short-acting insulin administration.
B. Introduction of potassium-containing solutions from the very beginning of treatment.
C. Trisamine administration.
D. Introduction of 40% glucose solution.
E. Introduction of methylene blue 1% at a rate of 2.5 mg / kg intravenously

Are all of these mechanisms involved in the pathogenesis of hypoglycemic coma except?
A. Activation of sympathetic adrenal system.
B. Increased glycogenesis.
C. Insufficient insulin supply.
D. Release of contraceptive hormones into the bloodstream.
E. Dehydration.

Which of the following examinations should be performed in diabetic patients before a planned
pregnancy?
A. Determination of glucosuria in the morning portion of urine.
B. Determination of fasting glucose.
C. Detection of vascular and neurological complications.
D. Determination of the level of NSAIDs.
E. Definition of thyrotropin

What should a diabetic patient do during pregnancy?


A. «Eat for two»
B. Reduce the amount of easily digested carbohydrates.
C. Ignore the state of diabetes compensation.
D. Carefully compensate for diabetes.
E. Increase dietary fat quota

The endocrinologist was approached by a woman in connection with not bearing pregnancy. The
anamnesis revealed that the first pregnancy ended with the death of the fetus in the womb at 38
weeks, two after which ended with early spontaneous abortions. Each time under. glucosuria is
observed during pregnancy. When viewed, obesity. Which of these surveys is worth conducting?
A. Determine C peptide and insulin.
B. Medico-genetic consultation.
C. Perform basal thermometry.
D. Determine glucose tolerance.
E. Determine the level of IRI.

Characteristic signs of diabetic neurotrophic ulcer are:


A. Great pain.
B. The ulcer is not painful.
C. Arterial blood flow to the arteries of the feet is significantly reduced.
D. Blood flow on the arteries is preserved.
E. Most common localization on the lateral surface of the foot.

"Diabetic foot" is:


A. The manifestation of peripheral polyneuropathy with increasing destruction of one or more
joints of the foot.
B. Trophic changes in the foot due to arterial hypoxia.
C. Trophic manifestations on the foot due to venous stagnation.
D. Traumatic fractures; bones of the foot in a patient with diabetes.
E. Gangrenous foot ulcer in a diabetic patient

Diabetic autonomic neuropathy of the heart manifests itself:


A. Persistent hypertension.
B. Hypotension.
C. No pain in the heart area.
D. Persistent tachycardia.
E. Flashing arrhythmia.

Which examinations are most appropriate for a patient with diabetic xanthoma?
A. Immune status survey.
B. Determination of lipid content in blood.
C. Determination of blood triglyceride content.
D. Lipidogram.
E. Plaque biopsy and histological examination.

What treatment should be prescribed for a patient with diabetic xanthoma?


A. Exogenous cholesterol diet regimen.
B. Increasing exercise is sufficient.
C. Indispensable persistent diabetes compensation.
D. Prescription of statins.
E. Carbon enterosorbents
F. Prescription of hepatoprotective drugs

Which treatment is considered most effective for hemorrhage into the vitreous?
A. Introduction of cerebrolysin into the conjunctiva.
B. Introduction of salkoseryl into the conjunctiva.
C. Glycemia normalization.
D. Laser-photocoagulation.
E. Sub-conjunctival injections of biogenic stimulants.

A 64-year-old female patient with type 2 diabetes is treated with 60mg Diabetes in the morning.
Fasting glycemia 6.8 mmol / l, one hour after meals - 9.5 mmol / l. Consulted by an optometrist.
Conclusion - "Diabetic proliferative retinopathy". What is the next method of treatment for the
patient?
A. Leave Diabetes MR therapy.
B. Immediately switch to short-acting insulin therapy.
C. Carry out laser -photocoagulation.
D. Transfer to therapy with prolonged drugs, 2 injections a day.
E. Transfer to combination insulin therapy with Diabeton MR.

A 52-year-old IDD patient complains of diarrhea, which bothers her mostly at night, is imperative.
The number of faeces can reach 30-40 per night. The digestive system has been experiencing
discomfort for the last two years. The bloating under the spoon, periodic constipation and diarrhea
were a concern, which was interpreted as chronic pancreatitis. Took enzyme preparations without
much success. Diabetes has been ill for 18 years. Is treated with insulin monotard - 46 IU in the
morning. Glycemia on an empty stomach, usually in the range of 11-13 mmol / l. Despite diarrhea,
there is no weight loss. What is your tactic?
A. Papaverine electrophoresis on the epigastric region.
B. Electrophoresis with novocaine on the epigastric region.
C. Glycemia normalization.
D. Amplipus with novocaine of the upper half of the abdomen.
E. Diet therapy correction.
The patient has diabetic gastroenteropathy, erosive bulbitis. Which of the methods of
physiotherapy is most appropriate in this case?
A. Papaverine electrophoresis on the epigastric region.
B. Novocaine electrophoresis on the epigastric region.
C. Glycemia normalization.
D. Amplipulse theraphy with novocaine of the upper half of the abdomen.
E. Diet table № 9A with the exception of eggs and salt
F. Amplipulse therapy with hydrocortisone in the upper half of the abdomen.

A patient of 42 years with type 1 diabetes and pre-nephrotic stage diabetic nephropathy wants
treatment at Truskavets. Basic against the indications for sanatorium and rehabilitation in the
sanatorium in Truskavets?
A. Diabetic polyneuropathy.
B. Decompensated diabetes.
C. Chronic renal failure terminal stage.
D. Diabetic foot IVst. After Wagner.
E. Pre-proliferative retinopathy
F. Chronic renal failure III stage.

The buyer of 18 years went to the pharmacy with the intention to buy medicine for shivering and
sweating, told that he had diabetes and was taking insulin. Aggressively behaved, refused the sweet
tea offered by the pharmacist, lost consciousness delivered to the emergency room. What will be
the treatment tactics?
A. Introduction of 40 ml of 40% glucose solution intravenously
B. Introduction of 50 ml of 5% glucose solution intravenously
C. Administration of 20 IU of insulin subcutaneously
D. Introduction of glucagon 1, about intramusculary.
E. Introduction of 500 ml of 0.9% sodium chloride solution

Patient T., 26, is in the intensive care unit in connection with a ketoacidotic coma. The
consciousness is blurred, the tone of the eyeballs is reduced, the blood pressure is 90/60 mm Hg, the
pulse is 130 beats / min. Glycemia 25 mmol / l, pH 7.1. Content of ketone bodies (++++). What is
your initial tactic?
A. Introduction of 500 ml 5% glucose + 10 IU short-acting insulin.
B. Introduction of 4% sodium bicarbonate 2.5 ml / kg.
C. 10-20 administration of short-acting insulin into insulin / injection, and then drip at a rate of
0.1 U / kg / h to eliminate ketoacidosis.
D. The introduction of 10-20 insulin short-acting insulin / injection, and then drip at the rate of
0.05 IU / kg / h to eliminate ketoacidosis.
E. Introduction of 500 ml of 0.9% sodium chloride / dropwise.

Patient 38 years old complained of weight gain, drowsiness, difficulty concentrating, secondary
amenorrhea. Objectively: height 165 cm, weight 78 kg, female phenotype, t = 35,8C, HR = 58 /
min, blood pressure = 105/60 mm Hg. Other internal organs unchanged. Thyroid gland diffusely
compacted 1 degree. Celebrated galactorrhea 1 st. Laboratory study found an increase in TSH and
prolactin levels, and decrease FT4 level. What should be the subject of differential diagnosis?
A. Primary hypothyroidism.
B. Secondary hypothyroidism.
C. Tertiary hypothyroidism.
D. Polycystic ovary.
E. Prolactinoma.
A resident of the mountain district of Ivano-Frankivsk region, 23 years old, complains of
miscarriage (in history - 2 miscarriages). Palpatory thyroid gland 1 degree, elastic, homogeneous;
ultrasound - slightly reduced echo and heterogeneity of the structure. There are no clinical signs of
thyroid dysfunction. Laboratory: TSH = 7.2 mIU / l (N: 0.3–4.0), FT4 = 1.22 ng / dL (N: 0.93–1.7),
anti-TPO = 120 IU / ml ( N: <100). What is the probable cause of this situation?
A. Autoimmune thyroiditis, subclinical hypothyroidism.
B. Iodine deficiency subclinical hypothyroidism.
C. Thyrotoxicosis.
D. Manifest hypothyroidism.
E. Post-stress disorder.

Sick 54 years old, complains of rapid fatigue, swelling in the neck, feeling cold, constipation. Sick
for 3 years. Objectively: height 172 cm, weight 94 kg, face pasty, dry skin, hair thin, pulse 60
/minute, blood pressure 100/70 mm Hg. The thyroid gland is diffusely enlarged 2 degree,
moderately dense, heterogeneous, painless. Ultrasound: 34 cm3 volume, echogenicity reduced,
structure heterogeneous, blood flow increased. What is the likely diagnosis?
A. Endemic diffuse goiter, hypothyroidism.
B. Hashimoto's thyroiditis, hypothyroidism.
C. Diffuse toxic goiter.
D. Autoimmune thyroiditis, hypothyroidism.
E. Autoimmune thyroiditis, euthyroidism.

In a 35-year-old woman, who has no complaints, palpation of the thyroid gland found Ø 1.5 cm in
the left lobe, moderately dense in consistency, indistinctly delineated, sensitive to palpation. In
ultrasound examination, it has reduced echogenicity, heterogeneous structure, with fuzzy contours.
What is the scope of the examination that is appropriate to clarify the diagnosis?
A. TSH level determination.
B. Determination of TSH, FT4 and FT3 levels.
C. Fine needle aspiration biopsy.
D. Determination of thyroglobulin content.
E. Determination of calcitonin level
F. Determination of TSH, FT4 and ATTG levels

Woman 65 years old, complains of dry skin, memory loss, constipation, feeling cold. On
examination: dry, cold to the touch skin, slow reflexes, swelling of the face and extremities. These
symptoms are not typical for:
A. Graves-Bazedov Disease.
B. Manifest hypothyroidism.
C. Subclinical hypothyroidism.
D. Iodine deficiency state.
E. Subacute thyroiditis
F. Autoimmune thyroiditis
G. Acute thyroiditis

Patient M., 39, who had a sore throat 2 weeks ago, had sudden pain in the right half of his neck
with irradiation into his lower jaw and ear. Objectively: thyroid gland of the second century. due to
the right lobe, which is dense and sharply painful on palpation; body temperature 38,7˚С; strong
weakness, hyperhidrosis. Which of the following may be likely in a patient:
A. Nodal Toxic Goiter.
B. Subacute thyroiditis (de Kerven).
C. Hashimoto's thyroiditis.
D. Graves-Bazedov's disease.
E. Acute purulent thyroiditis.

Patient 74 years old, for 12 years is being monitored for multiple nodular euthyroid goiter IB
degree. Heredity is not burdened. Clinically and sonographically, the nodes do not show active
growth. Cytological examination of the dominant biopsy specimen confirmed a benign process.
Which therapeutic tactic does not have sufficient grounds in this case?
A. Surgical treatment.
B. Radiotherapy.
C. Use of levothyroxine.
D. Observation, annual control.
E. Iodine preparations
F. Use of thiamazole
G. Use of celecoxib

A 43-year-old man who suffered a sore throat 3 weeks ago had pain in the left half of his neck with
irradiation in his lower jaw and ear. Objectively: thyroid gland of the second century. at the
expense of the left lobe, which is considerably thickened and sharply painful on palpation; body
temperature 38,3˚С; clinical and laboratory signs of thyrotoxicosis, ESR = 46 mm / h. Which of the
following drugs should not be used in this patient?
A. Thiamazole.
B. Ciprofloxacin.
C. Prednisone.
D. Levothyroxine.
E. Celecoxib
F. Cortisol

A man of 47 years, drew attention to the swelling of the right half of the neck, which has gradually
increased over the past 10 months; predisposition to diarrhea. Palpator in the right lobe of the
thyroid gland revealed a Ø 3 cm, dense consistency, indistinctly delimited, restrictedly mobile, not
painful. At ultrasound examination it has low echogenicity, with fuzzy contours and increased
blood flow. Serum concentrations of TSH, FT4 and thyroglobulin within normal limits, calcitonin
– significantly increased. What is the likely diagnosis?
A. Differentiated thyroid cancer.
B. Medullary thyroid cancer.
C. Anaplastic thyroid cancer.
D. C-cell carcinoma of the thyroid gland.
E. Subacute thyroiditis.

A 58-year-old woman complains of apathy, lethargy, memory loss, feeling cold, dry skin,
constipation, facial swelling. Objectively: temperature is 36.00 C, heart rate is 56 rpm, blood
pressure is 110/85. ECG decrease in voltage, decrease in gear P and flattening of T. T. Hormone
concentrations: FT4 = 0.77 ng / dl (N: 0.93-1.7), TSH = 22.4 mIU / l (N: 0.3 -4.0), anti-TPO =
328MO / ml (N: <100). Thyroid enlarged to II degree, moderately dense and patchy, hilly, not
painful. What is your diagnosis?
A. Autoimmune thyroiditis, euthyroid condition.
B. Autoimmune thyroiditis, latent hypothyroidism.
C. Autoimmune thyroiditis, manifest hypothyroidism.
D. Hashimoto's disease, manifest hypothyroidism.
E. Endemic goiter, manifest hypothyroidism.

Patient M., 46 years old, has been treated for chronic hypochromic anemia for 3 years. In the last 4
- 5 months, she began to worry about frostbite, drowsiness, constipation, memory loss. Objectively:
leather, dry, hair loss on the temples. HELL 100/70 mm Hg. bradycardia. Hb = 108 g / l, ferritin =
105 ng / ml (N), TSH = 14,8 mMO / l (N: 0,3–4,0). Which statement is most likely?
A. Iron deficiency anemia → continue treatment.
B. Iron deficiency anemia → intensify treatment.
C. Hypothyroidism → iodine preparations.
D. Hypothyroidism → replacement therapy.
E. Hypothyroidism → Levothyroxine.

Patient K., 52, complains of unpleasant sensations in the neck, difficulty breathing, hoarseness.
Duration is unknown. There are no objective signs of thyroid dysfunction. The thyroid gland is
enlarged up to the 2nd century. It is very dense in consistency, with a smooth surface, it is limited in
mobility, it is not painful. Laboratory: TSH = 2.16 mIU / l (N: 0.3–4.0); anti-TPO, thyroglobulin
and calcitonin within the hormone. What diagnosis should be considered?
A. Endemic goiter.
B. Subacute thyroiditis (de Kerven).
C. Autoimmune thyroiditis (Hashimoto).
D. Thyroid cancer.
E. Fibrous thyroiditis (Riddle).

Patient M., 39, who had a sore throat 3 weeks ago, had pain in the right half of his neck with
irradiation in his lower jaw and ear. Objectively: thyroid gland of the second century. due to the
right lobe, which is fairly dense and sharply painful on palpation; body temperature 38,0˚С;
moderate signs of thyrotoxicosis. The scope of examination in this patient should include all items
except:
A. Thyroid ultrasound.
B. TSH, BT4, BT3, thyroglobulin.
C. Antibodies to TPO and TG.
D. Calcitonin level.
E. The content of C-reactive protein.

Patient K., 66 years old, addressed the appearance of a circular formation on the neck 3 months ago.
Worked in the Chernobyl zone in 1987-1988 Objectively: in the lower pole of the right lobe of the
thyroid gland, the nodular formation of an oval shape of 3.2 x 2.5 cm in size, dense consistency,
with uneven hilly surface, painless, motile. Ultrasound: a large part of the right lobe is occupied by
a heterogeneous structurestructure of a fuzzy contour, containing multiplemicrocalcins. Suggest
optimal tactics.
A. Surgical treatment.
B. Active surveillance.
C. Prolonged use of levothyroxine.
D. Intraoperative rapid biopsy.
E. Preparations of iodine + levothyroxine.

A 35-year-old man has been approached by an endocrinologist who is disturbed by a neck


discomfort on the right. Objectively: skin and internal organs without abnormal abnormalities,
heart rate 72 beats / min, blood pressure 115/70 mm Hg. Art. Thyroid gland IV. due to the right
lobe, which palpates an indistinctly delimited formation with a diameter of 2–2.5 cm, medium
density, not painful. Regional lymph nodes are not palpated. What is your previous diagnosis?
A. Nodal Toxic Goiter.
B. Autoimmune thyroiditis, thyrotoxicosis.
C. Nodal non-toxic goiter.
D. Subacute thyroiditis.
E. Thyroid cancer.

Even nowadays, endemic Cretinism has been reported in some regions of the planet. The following
is true of this pathology:
A. Due to iodine deficiency in the environment.
B. Levothyroxine treatment prevents the development of mental and physical disorders.
C. A mandatory component is a pronounced psychoneurological defect.
D. Accompanied by a violation of the skeletal development.
E. Treatment ~ with iodine supplements the development of mental and physical disorders
F. Due to the deficiency of L-thyroxine in the environment

In a patient of 45 years revealed a diffuse enlargement of the thyroid gland II degree and clinical
signs characteristic of hypothyroidism.In order to clarify the diagnosis, the following examinations
should be performed, except:
A. Thin-needle aspiration biopsy.
B. Determination of TSH levels free of T4 in the blood.
C. Determination of thyroglobudine level.
D. Thyroid ultrasound.
E. Determination of the content of autoantibodies to TPO.

Patient 36 years old, underwent right-sided hemithyroidectomy for endemic nodal non-toxic goiter.
After 6 months, excessive drowsiness, frostbite began to worry. The thyroid gland does not palpate.
Laboratory: TSH = 14.2 mIU / L (N: 0.3-4.0), FT4 = 0.89 ng / dL (N: 0.93-1.7), anti-TPO = 52 IU /
ml (N: <100) .Take rational tactics.
A. Potassium iodide 200 mcg / day.
B. Levothyroxine for substitution therapy.
C. Levothyroxine to prevent relapse.
D. Observation.
E. Non-steroidal anti-inflammatory drugs.

Patient S., 26 years old, was diagnosed with hypothyroidism a year ago due to chronic autoimmune
thyroiditis. Stably takes levothyroxine 75 mcg / day, has no complaints. Asked about pregnancy 1
- 6 weeks, in connection with this additionally takes potassium iodide 200 mcg / day. The thyroid
gland of the 1 degree, moderately compacted, is not painful. Laboratory: TSH = 3.92 mIU / L (N:
0.3-4.0), FT4 = 1.12 ng / dL (N: 0.93-1.7), anti-TPO = 68 IU / ml (N: <100). What is your tactics?
A. Continue potassium iodide.
B. Increase the dose of levothyroxine by 25 mcg / day.
C. Increase the dose of levothyroxine by 50 mcg / day.
D. Increase the dose of levothyroxine to achieve a TSH level ≤ 2.5mMO / L.
E. Increase the dose of levothyroxine to achieve TSH level ≤ 3.3mMO / L.

At the Lviv Center for Endocrine Surgery, every year 700 - 800 (in some years up to 1000) surgical
interventions are performed on the thyroid gland. In which pathologythis organ is surgical
treatment consideredmethod of choice?
A. Subacute thyroiditis (de Kerven).
B. Autoimmune thyroiditis (Hashimoto's variant).
C. Tillary cancer T1N0M0.
D. Postpartum thyroiditis.
E. Multi-site toxic goiter.

Patient 76 years was delivered to the intensive care unit. According to the anamnesis, objective
examination and individual laboratory parameters, the most probable diagnosis is defined by
hypothyroid coma. Which of the following is not used in the treatment of this critical condition:
A. Passive warming of the patient.
B. Diazepam 10 mg (2 ml) IU / muscle every 12 hours.
C. Levothyroxine prevenously or via gastric probe 500 μg the first day.
D. Thiamazole via gastric tube 20 mg every 8 hours.
E. Intravenous infusion of 125 mg of hydrocortisone in 250 mg of 5% glucose solution.
A 68-year-old woman was prescribed amiodarone according to a standard regimen due to cardiac
arrhythmias. A few months later, she started complaining of daytime drowsiness, memory loss,
frostbite, constipation. Laboratory: TSH = 15.2 mIU / L (N: 0.3-4.0), FT4 = 0.79 ng / dL (N: 0.93-
1.7), anti-TPO = 132 IU / ml (N: <100) .Take rational tactics.
A. Cancel amiodarone.
B. Continue amiodarone, add iodine.
C. Continue amiodarone.
D. Prescribe levothyroxine.
E. Continue amiodarone, add prednisone.

The 36-year-old patient was diagnosed with primary manifest hypothyroidism. Takes
levothyroxine 75 mcg / day, has no complaints. From internal organs without deviations. Control
examination: TSH = 8.52 mIU / L (N: 0.3–4.0), TT4 = 1.04 ng / dL (N: 0.93–1.7). Twice attempts
were made to increase the dose to 100 mcg / day, but each time was accompanied by tachycardia.
What can you recommend?
A. Increase dose of levothyroxine + bisoprolol.
B. Increase the dose of levothyroxine + propranolol.
C. Increase the dose of levothyroxine + atenolol.
D. Increase the dose of levothyroxine + psychotherapy.
E. Leave a dose of levothyroxine 75 mcg / day.

Much of the Earth's population lives in chronic iodine deficiency. In order to maintain the normal
state of iodine metabolism and adequate production of thyroid hormones, the body "includes" the
following compensatory mechanisms, except:
A. Increased uptake of iodine by thyrocytes.
B. Acceleration of thyroid hormone biosynthesis.
C. Increased secretion of TSH by the pituitary gland.
D. Weakening of TSH secretion by the pituitary gland.
E. Increased sensitivity of the pituitary to iodine.

Patient N., 59, has no complaints. During prophylactic ultrasound, a nodular formation in the lower
pole of the right lobe of the thyroid gland was detected. On the basis of a comprehensive
examination, a diagnosis of "Papillary carcinoma T2N0M0" was established. The treatment
program should include:
A. Hemithyroidectomy.
B. Thyroidectomy.
C. Radiotherapy.
D. Levothyroxine suppressive therapy.
E. Chemotherapy
F. Prophylactic lymphadenectomy

Patient 66 years of age is on the record for chronic enterocolitis. Periodic treatment was
accompanied by temporary relief of symptoms. An expanded examination revealed that the patient
manifested hypothyroidism in the soil of chronic autoimmune thyroiditis, and prescribed
replacement hormone therapy had a positive effect on the functioning of SCC. What other atypical
monosymptomatic forms of hypothyroidism are known to you?
A. Edema.
B. Anemic.
C. Hypertension
D. Depressive.
E. Cachectic
F. Hypotension
G. Diarrhea

In a 56-year-old woman, nodular goiter and moderate clinical signs of thyrotoxicosis were detected.
Laboratory: TSH = 0.12 mIU / l (N: 0.3–4.0), FT4 = 1.76 ng / dL (N: 0.93–1.7), FT3 = 5.88 pg / dL
(N: 2.5-4.3). On the scintigram - "hot zone" in the projection of the node. Which of the following
is suitable for diagnosis?
A. Diffuse toxic goiter.
B. Thyrotoxicosis syndrome.
C. Nodular goiter, euthyroidism.
D. Toxic adenoma of the thyroid gland.
E. Functional autonomy of the thyroid gland
F. Nodular goiter

Patient O. with postoperative hypothyroidism, who was prescribed 125 μg of L-thyroxine per day,
tachycardia, sweating, sleep disorders, tremor of the fingers. How can a diagnosis be made?
A. Iatrogenic thyrotoxicosis.
B. Iatrogenic hypothyroidism.
C. Drug thyrotoxicosis.
D. Thyroid hormone resistance.
E. Toxic goiter recurrence.

The patient after experiencing psycho-emotional stress complains of irritability, sweating, tremor of
hands, palpitations (≈ 110 beats / min.), Weight loss with the appetite preserved. The thyroid gland
is enlarged to the second degree at the expense of all departments, elastic, not painful, auscultative
systolic noise is heard over it. The above symptoms are most consistent with:
A. Diffuse toxic goiter.
B. Neurasthenia.
C. Autoimmune thyroiditis.
D. Graves-Bazedov's disease.
E. Hypoparathyroidism.

Patient M., 33 years. Complains of irritability, palpitations, sweating, general weakness, shortness
of breath on loading. Weight lost 7 kg. Objectively: height 168 cm, weight 58 kg. The skin is
moist. The thyroid gland is enlarged at the expense of all departments, elastic, without nodular
formations. There is a glint, a slight bilateral exophthalmos, tremor of the fingers. Pulse 120 for/
min., BP 150/60 mm Hg. Art. Which diagnoses should not be considered?
A. Toxic thyroid adenoma.
B. Diffuse toxic goiter.
C. Thyrotoxicosis syndrome.
D. Autoimmune thyroiditis, euthyroid state.
E. Chronic fibrous thyroiditis
F. Autoimmune thyroiditis

Patient S., 45 years. Has been suffering from diffuse toxic goiter for 6 years and has had
recurrences twice. Which of the medicines administered at different times has anti-thyroid effects?
A. Prednisone
B. Thiamazole.
C. Carbimazole.
D. Phytosedative complex
E. Anaprolin.
Patient S. has been suffering from diffuse toxic goiter for 10 years. Due to a postponed
appendectomy, a thyrotoxic crisis developed due to poor compensation of thyrotoxicosis. To
withdraw a patient from this critical condition use:
A. Thiamazole.
B. Hydrocortisone.
C. Insulini.
D. Levothyroxine.
E. Iodine in large doses
F. Euthyrox

Woman 25 years old, emotionally labile, on examination - tremor of fingers, skin moist, warm,
apical impulse increased, atrial fibrillation. Temperature 37.2 C, heart rate 96 beats / min., blood
pressure 170/70 mm Hg. Art. Thyroid gland II degree. The patient's pathogenetic mechanisms are
at the heart of impaired cardiovascular function:
A. Increased receptor sensitivity to corticosteroids.
B. Increasing the sensitivity of receptors to catecholamines
C. Increased catabolism in the myocardium.
D. The effect of thyroid stimulating antibodies.
E. Effect of excess thyroid hormones on the myocardium
F. Decrease receptor sensitivity to corticosteroids

In a patient of 48 years after surgical treatment of diffuse toxic goiter seizures of chewing muscles,
hands with predominance of flexor tone are observed. Seizures are painful, symmetrical. On
examination positive symptoms of Hvostek, Trusso. Which of the following is suitable for
diagnosis?
A. Postoperative hypoparathyroidism.
B. Latent form.
C. Manifest form.
D. Epilepsy.
E. Hyperparathyroidism

A woman 25 years old in the postpartum period experienced nausea, vomiting, severe psychomotor
agitation with partial disorientation, hyperthermia up to 39.5˚C, tachyarrhythmia, increased blood
pressure up to 180/80 mm Hg. , abdominal pain. From the anamnesis it is known that the patient
was treated for a long time with mercazolyl. Suspected thyrotoxic crisis. What pathologies may
require differential diagnosis?
A. Acute psychosis.
B. Hypercalcemic crisis
C. Hypertensive crisis.
D. Sepsis.
E. "Acute abdomen"
F. Hypocalcemic crisis
G. Hypophosphatemic crisis

Patient R., 37, received complaints of weight loss, severe weakness, palpitations, limb tremor,
sweating. Objectively: the thyroid gland enlarged to the second century, at the expense of all
departments, elastic, not painful, blood pressure 180/60 mm Hg. Art., pulse 120 beats / min.
Bilateral ophthalmopathy of the III-IV centuries. Laboratory: TSH = 0.02 mIU / l (N: 0.3–4.0), FT4
= 2.26 ng / dL (N: 0.93–1.7), FT3 = 6.88 pg / dL (N: 2.5–4.3), antibodies to the TSH receptor =
18.5 IU / l (N: <1.75). In the etiopathogenesis of this disease the leading role is played by:
A. Provoking factors.
B. Genetic predisposition.
C. Immune mechanisms of development.
D. Non-immune mechanisms of development.
E. Adrenal dysfunction
F. Pancreatic dysfunction

Patient N., 50 years old, complains of headache, tearing, feeling of sand in the eyes, fissure,
sometimes diplopia. He became ill suddenly after suffering from the flu. He was treated by an
optometrist, but to no effect. Objectively: skin moist, heart rate 92 beats / min, blood pressure
140/75 mm Hg. Art. Pronounced bilateral exophthalmos, conjunctivitis, movements of the
eyeballs are limited. The thyroid gland is diffusely enlarged IB degree, smooth not painful. Signs
of which pathology was present in the described clinical picture?
A. Hypothyroidism syndrome.
B. Graves-Bazedov's disease.
C. Subacute thyroiditis.
D. Autoimmune thyroiditis.
E. Endocrine orbitopathy.

In the patient30, after experiencing stress, general weakness, weight loss of 3 kg, irritability,
palpitations, tremor of the fingers appeared. Objectively: heart rate 100 beats / minute, blood
pressure 140/65 mm Hg Art., skin moist, warm, light bilateral exophthalmos. The thyroid gland
enlarged to the 2nd century, moderately compacted, homogeneous, slightly painful. Which of the
following diagnoses should be considered?
A. Acute purulent thyroiditis.
B. Endemic goiter.
C. Diffuse toxic goiter.
D. Autoimmune thyroiditis, thyrotoxic stage.
E. Subacute thyroiditis, thyrotoxic stage
F. Acute thyroiditis

Patient S., 62, complains of increased irritability, emotional lability, increased sweating,
palpitations, tremor of the fingers. Objectively: heart rate ≈ 110 beats / minute, atrial fibrillation,
blood pressure 150/60 mm Hg The thyroid gland (thyroid gland) is enlarged to the second century,
palpated by several well-defined nodes with a diameter of 1 to 2 cm in both parts. Laboratory: TSH
= 0.02 mIU / L (N: 0.3–4.0), FT4 = 1.86 ng / dl (N: 0.93–1.7), FT3 = 9.66 ng / dl ( N: 2,5-4,3),
antibodies to the TSH receptor = 0.65 IU / l (N: <1,75). Which of the following definitions matches
the diagnosis?
A. Diffuse toxic goiter.
B. Endemic nodular goiter.
C. Multi-site toxic goiter.
D. Functional autonomy of the thyroid gland, Art. decompensation.
E. Subacute thyroiditis, thyrotoxic stage.

The intensive care unit received a patient with suspected hypercalcemic crisis. Blood calcium 3.3
mmol / l. What should the treatment program include?
A. Sedative therapy.
B. Use of glucocorticoids.
C. Rehydration, forced diuresis.
D. Introduction of calcitonin and / or bisphosphonates.
E. Antihypertensives
F. Introduction of calcitonin and / or levothyroxine

The intensive care unit received a patient with convulsions of the upper and lower extremities,
"obstetrician's hand", "horse's foot". On examination the skin is pale, moist, pulse 80 beats / min,
blood pressure 150/100 mm. Mm. Art. It is known that three weeks ago the patient underwent
thyroidectomy. What is your previous diagnosis?
A. Postoperative hypoparathyroidism.
B. Epilepsy.
C. Hypocalcaemic crisis.
D. Manage.
E. Acute hypocorticism.

Patient S., 35, suffers from Graves-Bazedov's disease. In this disease, the following ocular
symptoms occur:
A. Kocher.
B. Trusso.
C. Chvosteca ( Tail).
D. Gref.
E. Mobius
F. Chvosteca 2st.

Female 25 years old, emotionally labile, on examination of tremor of fingers, skin moist, pulse of
112 beats / min (N: 0.93–1.7), FT3 = 6.88 ng / dL (N: 2.5–4.3), antibodies to the TSH receptor =
14.5 IU / l (N: <1.75 ). Graves-Bazedov's disease has been established. Specify the
ultrasonographic features of this pathology:
A. Echogenicity is evenly increased
B. Reduced echogenicity.
C. Heterogeneous echostructure.
D. Uneven gland contour.
E. Increased blood flow in the parenchyma.

In a patient of 57 years with biochemical blood test revealed hypercalcemia, hypophosphatemia,


hypomagnesemia, a significant increase in the level of alkaline phosphatase, a slight - bilirubin,
AST, ALT. Patients with which diseases are at risk for primary hyperparathyroidism?
A. Thyrotoxicosis.
B. Chronic pancreatitis.
C. Kidney-stone disease.
D. Peptic ulcer of GCC.
E. Addison's disease
F. Coronary heart disease

Patient L., 42 years. operated for the diffuse toxic goiter of the III degree. A month after surgery,
the patient had paresthesia and limb muscle twitching. Objective: Pulse 76 beats / minute, blood
pressure 132/80, symptoms of Chvosteca ( Tail) I-II, Trusso positive. Blood calcium is 2.0 mmol /
l, TSH = 3.68 mIU / l (N: 0.3-4.0). Which of the following is suitable for diagnosis?
A. Postoperative hypothyroidism, Art. decompensation.
B. Postoperative hypoparathyroidism.
C. Latent form.
D. Manifest form.
E. Relapse of thyrotoxicosis.

Female 36 years old, admitted to the intensive care unit. It was operated on about the diffuse toxic
goiter of the third century. Sudden contractions of the muscles of the extremities suddenly appeared
in the patient. symptoms of the Chvosteca ( Tail), Trusso positive. Pulse 76 beats / minute, blood
pressure 136/80. The diagnosis of hypoparathyroidism has been established. Which of the
following drugs is not suitable for eliminating tetany attacks?
A. 10% solution of calcium chloride IV.
B. 10% calcium gluconate solution IV.
C. Prednisolone 30 mg I/V.
D. 40% glucose solution IV.
E. Furosemide 40 mg
F. 10% solution of calcium chloride IM

A 35-year-old patient complained to the doctor about complaints of fatigue, weight loss. Likes
salty food. On examination, the face and hands are hyperpigmented, BMI 18 kg / m2 blood
pressure 80/55 mm Hg. Laboratory revealed increased levels of ACTH and potassium in the blood,
decreased levels of cortisol, sodium, testosterone and glucose, tuberculosis is not confirmed. The
diagnosis of primary chronic insufficiency of the adrenal cortex was established. What treatment
should be prescribed?
A. Glucocorticoids
B. Mineralocorticoids
C. Diuretics
D. α-blockers
E. β-blockers

In a woman, 37, the blood pressure gradually increases to 230/110 mm Hg, accompanied by
nausea, vomiting, tachycardia, and sweating. After the attack there is an abundant discharge of
urine. The sonography of the kidneys revealed the area of illumination adjacent to the upper pole of
the right kidney, which belongs to the adrenal gland. What additional research should be done to
diagnose it?
A. Determination of insulin and C-peptide in blood.
B. Determination of glomerular filtration rate by endogenous creatinine.
C. Determination of the level of vanillylmudic acid in the urine.
D. Determination of catecholamine excretion.
E. Determination of blood renin level.

Sick 26 years old, taken to the waiting room. The disease started gradually, 6 months ago. The
condition worsened dramatically after SARS. Objectively: Heart tones are weakened, heart rate is
60 beats / min, blood pressure is 80/40 mm Hg. The abdomen is somewhat painful in the epigastric
region. Leukocytes - 8,1x109 / l, blood glucose - 3,0 mmol / l. Diagnosed: Addison crisis. What
clinical features are characteristic of this condition?
A. Nausea, vomiting.
B. Muscle weakness.
C. Abdominal pain.
D. Bronze skin color.
E. Dizziness
F. Constipation

A woman of 47 years was brought to the hospital. Complaints: sharp headache, palpitations, fear,
heartache, abdomen, nausea. On examination: pale skin, heart rate 130 beats / min., Blood pressure
220/110 mm Hg. Suspected pheochromocytoma. What diseases should differential diagnosis be
made of?
A. Myocardial infarction.
B. Pancreatitis.
C. Thyrotoxicosis.
D. Vegetative-vascular dystonia.
E. Migraine.

In a patient of 50 years, the urine test: the share of urine 1006; proteinuria. K-2.4 mmol / l .;
serum 18-hydroxycorticosterone level - 98 ng (norm <30); on ECG signs of hypokalemia. Connor
syndrome was suspected. What blood test should you do to confirm your diagnosis?
A. Blood plasma renin determination.
B. Blood tests for cortisol.
C. General blood test.
D. ACTH research.
E. Blood test for aldosterone.

The patient complains of weakness, excitability, hyperpigmentation of exposed skin, nausea and
vomiting, decreased potency. HELL 85/60 mmHg, ECG: reduced voltage, depression of the S-T
segment. In the blood test: lymphocytosis, eosinophilia, normochromic anemia, cortisol in the
blood 110 nmol / l. Which statement will be correct for changes in blood in chronic adrenal
insufficiency?
A. Hypokalemia.
B. Hypercholesterolemia,
C. Hypocholesterolemia,
D. Hypernatremia,
E. Hyperkalemia
F. Hypercholesterolemia

In a patient of 40 years, who has long been treated for hypertension, a cardiologist suspected Conn's
disease. What blood tests should be done to confirm the diagnosis?
A. Blood aldosterone determination.
B. Determination of blood cortisol.
C. Determination of blood calcium.
D. Adrenal ultrasound.
E. Adrenal ultrasound

Patient L., 41, delivered by ambulance with complaints of severe general weakness, weight loss,
lack of appetite, nausea, vomiting, abdominal pain. On examination: hyperpigmentation of the skin,
blood pressure 70/45 mm Hg, bradycardia. In additional studies: reduced content of aldosterone,
cortisone in the blood, decreased excretion of 17-COP and 17-ACS with urine, hyponatremia,
hypochloremia, hypokalemia. What treatment measures should be taken?
A. Appointment of Vitamin C.
B. Purpose of glucocorticoids.
C. Purpose of mineralocorticoids.
D. The appointment of aldosterone.
E. Prednisolone appointment.

Patient C, 32, worries about weight gain, weakness, drowsiness, headache, decreased potency.
Overall: height 172 cm, weight 106 kg. Fat deposition mainly on the shoulders, torso. Dry skin
with cyanotic cyanotic pattern. On the shoulders, chest, thighs purple-blue distension distension.
AT 160/100 mmHg Pulse 76 beats / min. Evaluate the results of the determination of electrolytes
in the blood: potassium 3.0 mmol / l, sodium 160 mmol / l.
A. Hypokalemia.
B. Hyperkalemia
C. Normokalemia
D. Hypernatremia
E. Hyponatremia

What are the forms related to Adrenogenital syndrome (congenital adrenal cortical hyperplasia)
with 21 - hydroxylase deficiency?
A. Silent
B. True
C. Non-Classical (Post-Pubertal)
D. None of the above
E. Hypertonic
F. Classical
The patient has 39 complaints of periodic compression pain in the area of the heart, pronounced
weakness in the proximal muscles of the extremities and convulsions, pain in the neck, dizziness for
2 years. Height 176 cm, weight 80 kg. Pulse 92 / min. Blood pressure 190/100 mmHg.
Zymnitsky urine: polyuria, nicturia with isostenuria. Which of the statements about Connor
syndrome is correct?
A. Low renin level
B. Hyponatremia
C. Hypokalemia
D. Generalized edema
E. Hypertension
F. High renin level

The patient was 36, diagnosed with pheochromocytoma. Complains of high blood pressure,
polyuria, headache, general weakness, sweating, tremor. Objectively: BP= 175 / 100–210 / 115
mmHg, Рs 120 /min. The level of sodium in the blood is 150 mmol / l, K-2.8 mmol / l. What are
the hallmarks of hypertension in pheochromocytoma?
A. Increased systolic and diastolic blood pressure
B. Slow buying crisis
C. Expressed increase in pulse pressure
D. Repeated similar crises in history
E. Paradoxical hypertensive response to α-blockers
F. Increased systolic blood pressure

From the anamnesis it is known that the patient suffers from Addison's disease. Receives
prednisone daily. After suffering from the flu, the patient's condition worsened sharply: pain in the
heart area, weakness, dizziness, nausea, rare bowel movements appeared. Pulse 112 beats / min,
blood pressure 64/36 mm Hg. What underlies the development of an antihypertensive state in
Addison's disease?
A. Sodium loss
B. Dehydration
C. Increasing circulating blood volume
D. Reduced sensitivity of adrenergic receptors to catecholamines
E. None of the above
F. Increased sensitivity of adrenergic receptors to catecholamines

In a patient of 38 years complains of obesity, general weakness, drowsiness, headache, which


marks 4 years after the transferred viral infection. Objective: Height 171 cm, body weight 77 kg.
Laboratory: cortisol in the blood - 51 mcg / dl (4.3-22.4), ACTH - 45.35 ng / l (8.3-57.8), sodium -
158 mmol / l (132-146), potassium - 3.2 mmol / l (3.5-5.5), glucose tolerance test: 5.8-10.7 mmol /
l. Based on the examinations, the patient was diagnosed with Cushing-Cushing's syndrome. Which
of the following clinical features are characteristic of this disease:
A. Dysplastic obesity
B. Menorrhagia
C. Hypertension
D. Reduced potency
E. Blue-blue stretch marks
F. Hypotension

During the laboratory examination, the patient was 36 years old: potassium - 2.4 mmol / l (3.5-5.3),
sodium - 145 mmol / l (135-150), aldosterone-53.0 ng / dl ( 7-30), the aldosterone / renin ratio is
11.2 (3.8-7.7). According to the ECG revealed pronounced changes in myocardial diffuse nature.
According to the results of CT of the adrenal glands: right adrenal gland enlarged. Primary
aldosteronism is suspected. What are the characteristics of this disease?
A. Hyperkalemia
B. Muscle weakness
C. Disorders of the heart rhythm
D. The presence of signs of renal failure
E. Cramps
F. Menorrhagia

The teenager found obesity, pink thin stretch marks on the skin of the lateral surfaces of the
abdomen, blackheads. Schedule an examination:
A. Skull Radiography (Turkish saddle) \
B. Determination of blood cortisol
C. Glucose tolerance test
D. None of the above
E. Determination of catecholamine metabolites in urine
F. Determination of urine cortisol

Which of the following statements is true about Addison's disease?


A. Spots of hyperpigmentation on the gums
B. No pigmentation on mucous membranes
C. Hyperpigmentation of places of friction of clothing
D. Hyperpigmentation of the mammary glands
E. Hyperpigmentation of palm lines
F. Hypopigmentation of places of friction of clothing
G. No pigmentation of palm lines

What hormones are catecholamines related to?


A. Adrenaline
B. Norepinephrine
C. Dopamine
D. Cortisol
E. None of the above
F. Prednisolone

Patient A., complained of a sharp increase in weight (30 kg for 2 years), periodic headache,
reductionpotential. Objectively: height 170 cm, weight 114 kg. Fat deposition mainly on the chest,
abdomen. On the shoulders, abdomen and thighs of the band stretchingmagic color. Heart
tonesweakened, BP= 170/100 mm Hg. Art. Secondary sex traits and sex organs are well
developed. Analytical and urine without pathological changes. Define an algorithm for diagnostic
methods to verify diagnosis?
A. Blood biochemical analysis.
B. Determination of blood levels of cortisol and ACTH.
C. Adrenal ultrasound / CT scan.
D. Thyreliberin sample.
E. None of the above.

Patient 30, complains of a periodic sudden increase in blood pressure to 280/120 mm Hg,
accompanied by sweating, trembling of the whole body, a sense of fear of death. Seizures occur at
a frequency of 1-2 times a month, the course of malignant, resistant to most antihypertensive drugs.
After hypertensive crisis there is polyuria, sharp weakness. In the periods between crises, BP is
120/80 mm Hg. Suspected pheochromocytoma.Whichpathological conditions should be
madedifferentialdiagnosis?
A. Hypertension.
B. Hypothalamic vegetative-vascular crises.
C. Symptomatic hypertension of renal origin.
D. Kidney failure.
E. Itsenko-Cushing's Syndrome
F. Symptomatic hypotension of renal origin

A 60-year-old man recently treated for kidney tuberculosis complains of weight loss, weakness,
diarrhea. Hypotension and skin hyperpigmentation were detected. Which examination should be
scheduled?
A. Fecal analysis on helminth eggs
B. To determine the level of glycemia
C. Determine ACTH level
D. Determine cortisol levels
E. Vanillylmudic acid analysis

A patient of 42 years was suspected of Konn's disease. Complaints of headache, muscle weakness,
cramps, polyuria, nicturia. Objectively: BP - 195/120 mm Hg. The level of K - 3 mmol / l.,
glycemia 3.8 mmol / l. ECG: decrease in ST segment, change in gear amplitude T. What research
should be done to confirm the diagnosis?
A. Blood tests for calcium content.
B. Adrenal ultrasound / CT scan.
C. Blood tests for cortisol
D. Blood test for aldosterone.
E. General urine analysis.

The clinical features of the solvation form of adrenogenital syndrome (congenital adrenal cortical
hyperplasia) with deficiency 21 - hydroxylase include:
A. Cramps
B. Dizziness
C. Hypertension
D. Hypotension
E. Virilization

Patient H, suffering from diabetes for 30 years. Complains of weight loss, weakness, appearance
of vitiligo and hyperpigmentation of the skin. An imbalance of blood electrolytes was detected.
Assign a survey.
A. Determine ACTH level
B. Determine cortisol levels
C. Blood biochemical analysis
D. Urine analysis for vanillimagalic acid
E. Plasma Aldosterone

A patient, 36 years old, complains of weight loss, dizziness, dyspeptic symptoms, muscle and joint
pain. Objectively: skin is dry. Height 162 cm, body weight 45.3 kg, BMI 17.3 kg / m2. Blood
pressure 100/70 mm Hg, BPM - 92, rhythmic. No edema. General blood test: Hb 92 g/l, Er - 3.1 *
1012 /l, leukocytes 3.7 * 109 /l, platelets 162 * 109 / l. Fasting glycemia 4.0 mmol / l, K + 5.2 mmol /
l, Na + 130 mmol / l. What screening test is appropriate and which examinations should be
performed?
A. Urine cortisol, ACTH, aldosterone
B. FSH, LH, ACTH
C. Low-dose dexamethasone test
D. 17-ketosteroids urine test
E. Serum Na and K levels
The patient, 35tears old, complains of an increase in the size of the jaw, hands, feet, heart attacks,
menstrual disorders. On feet Rtg - subperiosteal deposits. Laboratory tests: normal level of TSH,
increased level of growth hormone. Which diagnoses might be correct?
A. Acromegaly
B. Giantism
C. Macroprolactinoma
D. Increased level of prolactin
E. Macroprolactinoma

Patient, 32 years old, has problems with menarche (oligomenorrhea). She had one pregnancy, one
natural delivery with the birth of a full-term healthy baby. Objectively: the skin is moderately moist,
acne, hirsutism, hypertrichosis. Height 180 cm, weight 76 kg, BMI 23.46 kg / m2. Stretch marks:
single, pale, located on the anterior surface of the abdomen and on the inner thighs surfaces.
Postpubertal adrenogenital syndrome is suspected. Which symptom and what laboratory test can be
used to confirm this diagnosis?
A. Increase of 17-hydroxyprogesterone level and normal cortisol level
B. Increased 17-oxyprogesterone levels and decreased cortisol level
C. Increase of FSH, LH
D. Development of hypertrichosis before menarche
E. Decreased 17-oxyprogesterone and decreased cortisonl

Patient, 32 years old, has problems with menarche (oligomenorrhea). She had one pregnancy, one
natural delivery with the birth of a full-term healthy baby. Objectively: the skin is moderately moist,
acne, hirsutism, hypertrichosis. Height 180 cm, weight 76 kg, BMI 23.46 kg / m2. Stretch marks:
single, pale on the anterior surface of the abdomen and the inner thighs surfaces. Postpubertal
adrenogenital syndrome is suspected. Which symptom can confirm this diagnosis and what
treatment would be appropriate in this case?
A. Prescribe 17-OH-progesterone
B. Prescribe mono-estrogenic drugs
C. Prescribe dexamethasone
D. Increased cortisol levels
E. Menstrualdisorders with menarche (oligomenorrhea)

The patient, 52 years old, was given hormone replacement therapy during her premenopausal
period. The absolute contraindications to the use of this group of drugs DO not include:
A. Genital tract bleeding
B. Urinary incontinence
C. Renal insufficiency
D. Endometriosis.
E. Untreated endometrial hyperplasia

The patient, 26 years old, complains of polyuria, nocturia, thirst. It is known that a year ago she
gwas admitted to the hospital due to a concussion after a car accident. Urine specific gravity 1004–
1008. Choose the most likely diagnose and electrolyte disorder which can be caused by this disease:
A. Secondary diabetes mellitus
B. Diabetes insipidus
C. Conn’s Syndrome
D. Hypernatremia
E. Hyponatremia

The patient, 38, complains of frequent headache,fatigue, disability. It was founf increased
intracranial pressure, secondary hypothyroidism. Skull Rtg: extension of sella turcica, double
contour of the ‘seat of the saddle’ with presellar pneumatization. MRI: cerebro-spinal fluid in
intrasellar area, flattened pituitary gland. What is the correct symptom and diagnosis:
A. Empty Sella Syndrome
B. Changes of sexual functions
C. A significant decrease of blood pressure
D. Diabetes mellitus
E. Pituitary Adenoma

The patient, 30 years old, complains of an increase of the jaw, hands, feet, heart attacks, menstrual
disorders (oligomenorrhea). It was found dyslipoproteinemia, increased levels of prolactin (42 ng /
ml) and somatotropic hormone. A 9 mm pituitary cystic adenoma was detected according to MRI.
Choose the symptoms and the correct diagnosis.
A. Acromegaly
B. Giantism
C. Face changes
D. Increased visual acuity.
E. Pain of the arms and legs (joints), limb sensitivity reduction
F. Diabetes insipidus

A woman, 26 years old, complains of central obesity, menstrual disorderes, hirsutism. The
examination revealed an increase ofcortisol and ACTH levels. An 11 mm pituitary adenoma was
detected regarding MRI. It was diagnosed Cushing's disease. Choose the false statements about this
disease.
A. "moon" face.
B. Arterial hypertension.
C. Osteoporosis
D. Frequent hypoglycemic conditions.
E. No menstrual disorders in women, increased libido

The patient complains of excessive thirst and polyuria. She drinks more than 8 liters of water per
day. She associates symptoms with a recent psycho-emotional trauma. It was found hypoisostenuria
of the urine and decrease of the antidiuretic hormone. What functional tests should becarried out to
diagnose diabetes insipidus?:
A. Captopril supression test
B. Lowdose dexametasone suppression test
C. Dexametasone test
D. Vasopressin test
E. Desmopressin Test

A patient, 25, complains of short stature and overweight. Low growth rate has been observed since
childhood, growth delay was more than 3 sigma deviation. The pituitary nanism was diagnosed.
What are the symptoms and criteria for this disease?
% -100%Height less than 100 cm
A. Height less than 110 cm
B. Height less than 120 cm
C. Short limbs or short torso
D. The disease is caused by insufficiency of the anterior pituitary gland
E. Height less than 130 cm

The patient, 36, complained of a sharp increase of weight (22 kg during the year), headache,
decreased libido. Objectively: height 170 cm, body weight 114 kg. Fat deposition mainly on the
chest, abdomen. On the shoulders, abdomen and thighs are blue stretch marks. Heart tones are deaf,
blood pressure 170/100 mm Hg. External and internal genitalia are well developed. Fasting
glycemia 7.6 mmol / l. What are the correct symptoms and diagnosis?
A. Constitutional obesity
B. Hypothyroid obesity
C. Cushing's Disease
D. Electrolyte disorders, lower extremities edema
E. Frequent hypoglycemic conditions

Precocious puberty can be a symptom of:


A. Hormone-producing testicular tumors
B. Andosterome
C. Congenital adrenal cortex hyperplasia
D. Macroprolactinoma
E. Gigantism
F. Hyperplasia in the pituitary gland

The patient complains of infertility, multiple pigment spots on the face. Turner syndrome is
suspected. What statements are false about this syndrome?
A. Epicanthus
B. Tall stature
C. Absence of webbed-neck
D. Reproductive sterility
E. No amenorrhea
F. The level of TSH is normal

What are the differential diagnoses for pituitary nanism?


Constitutionaldelay of growth and puberty
A. Laron Syndrome
B. Seckel Syndrome
C. Acromegaly
D. Turner Syndrome
E. Progesterone deficiency

The patient, 36, was examined for hypercorticism. According to the tests, the ectopic secretion of
ACTH is suspected. What statements are false about ACTH ectopic secretion syndrome?
A. Progesterone Deficiency
B. Excess of androgens
C. The disease has a long asymptomatic course
D. Positive dexamethasone test
E. Mostly affect women
F. Turner Syndrome

The patient, 30 years old, complains of severe weakness, anorexia, drowsiness, amenorrhea,
excessive hair loss, weight loss. It is known about complicated delivery with massive blood loss.
Objectively: asthenic-depressive syndrome, deficiency of body weight, dry cold skin, atrophy of
breasts, blood pressure 80/55 mm Hg, heart tones are deaf. What disease can be suspected and
which symptoms does it cause?
A. Hypothalamic syndrome
B. Primary hypothyroidism
C. Sheehan Syndrome
D. Hypoxia and pituitary necrosis
E. Significant decrease of LH and FSH
F. Significant increased of LH and FSH

The patient, 23 years old, complains of menstrual cycle disorders (oligomenorrhea), overweight,
acne. Objectively: female phenotype, moderate hirsutism, hypertrichosis, android type of obesity.
The examination revealed an anovulatory menstrual cycle, left ovary volume 11 cm3, right - 10.5
cm3. There are 14 follicles in the right ovary, 10 in the left, 2-9 mm in diameter. Increase of stroma
area and increase of echodensity. What disease can be diagnosed and what does it have in
pathogenesis?
A. Polycystic ovarian disease
B. Adreno-genital syndrome
C. Macroprolactinoma
D. Hyperprolactinemia
E. Caused by excess of androgens

The pathogenesis of Stein-Leventhal syndrome includes:


Ovarian overproduction of androgens
A. Giantism
B. Increased prolactin level
C. Hyperinsulinemia
D. Increased luteotropin production
E. Hypoinsulinemia
At what level of glucose concentration, on an empty stomach, diabetes mellitus proves to be
positive?
A. 5.55 mmol / L
B. 3.33 mmol / L.
C. 4.44 mmol / L.
D. 6.66 mmol / L.
E. 7.77 mmol / L.

Which of the biological effects is not common to insulin?


A. Stimulation of glucose transmembrane transport to cells.
B. Activation of glycogen synthesis from glucose.
C. Stimulation of lipids synthesis.
D. Stimulation of protein synthesis from amino acids.
E. Activation of gluconeogenesis from a protein.

Regarding the biological effects of glucagon, everything is correct except for:


A. Increases glycogenolysis.
B. Increases lipolysis.
C. Increases gluconeogenesis.
D. Increases ketogenesis.
E. Increases glycogenesis.

When carrying out the GTT ( glucose tolerance test), the results of glycemia were obtained: on
an empty stomach - 5.1 mmol / l, after 1 hour. - 10,7 mmol / l, after 2 years. - 8.0 mmol / l.
A. Norm.
B. Impairment of glucose tolerance.
C. Diabetes mellitus.
D. The test is questionable.
E. The test is unreliable.

The severe course of diabetes is characterized by all symptoms except for:


A. A history of coma.
B. Is compensated by insulin at a dose of more than 60 rpm.
C. There are severe complications.
D. Glucosuria greater than 100 g / d.
E. The smell of urine on exhalation.

Specify criteria not specific to offsetting the flow of diabetes:


A. Aglucosuria.
B. Ketonemia.
C. Normoglycemia.
D. Hb A1c less than 7%
E. Blood cholesterol is less than 6.4 mmol / l.

The criteria for diagnosis of type 2 diabetes are:


A. Often a labile course
B. Often in children after viral infection
C. Angiopathy is common at the time of illness
D. Is compensated by insulin
E. The level of C-peptide is decreased in the blood.

Insulin-dependent diabetes mellitus is characterized by:


A. The onset of the disease in old age.
B. The gradual beginning.
C. Weight gain.
D. Need for insulin therapy.
E. Stable flow.

1 type of diabetes is characterized by the following features:


A. It accounts for 90% of all diabetes.
B. 10% of all diabetes cases.
C. The inherent weight gain.
D. Etiology is not associated with traumatic injuries of the pancreas.
E. The etiology is not associated with the autoimmune destruction of beta cells.

What etiological factor does not play a role in the occurrence of type 1 diabetes?
A. Autoimmune.
B. Viral infections.
C. Genetic predisposition.
D. Antibacterial drug therapy.
E. Stress factors.

Patient K., 44 years old, complained of boils that did not disappear after prolonged treatment.
The family doctor prescribed a glucose tolerance test, the results of glycemia: on an empty
stomach - 5.4 mmol / l, after 1 hour. - 13,1 mmol / l, after 2 years. - 11.1 mmol / l. Which
diagnosis is most likely?
A. Chronic furunculosis without disturbing carbohydrate metabolism.
B. Chronic furunculosis. Impairment of glucose tolerance.
C. Type 2 diabetes, first detected.
D. Type 1 diabetes, first detected.
E. Chronic furunculosis, symptomatic diabetes mellitus.

The following shall be screened for diabetes:


A. People with severe weight loss.
B. People with glucosuria.
C. People with visual impairments.
D. Pregnant women from 24 to 28 weeks of pregnancy.
E. Patients with coronary heart disease.

Positive indicators of diabetes screening are:


A. Glycosylated hemoglobin level of 5%.
B. Glucose level at food load 8.3 mmol / l.
C. The presence of dry mouth, thirst, polyuria.
D. Fasting glucose level of the heart exceeding 6.0 mmol / l.
E. Glucosuria.

The criteria for the diagnosis of diabetes are:


A. The presence of glucosuria.
B. Weight loss.
C. The presence of obesity.
D. Daytime glycemia of 11.0 mmol / L.
E. Genital itching.

To diagnose diabetes, it is sufficient to:


A. The presence of pyoderma.
B. The presence of periodontal disease.
C. The presence of colpitis.
D. Presence of lipoid necrobiosis.
E. Double-verified increase in glucose concentration 2 hours after HTT 11.1 or greater
mmol / l.

Which of the following drugs can cause impaired glucose tolerance?


A. Lincomycin
B. Ascorbic acid.
C. Nicotinamide.
D. Hypothiazide
E. Clopidogrel.

List the endocrine diseases for which secondary diabetes mellitus is possible:
A. Hypothyroidism.
B. Pheochromocytoma.
C. Autoimmune thyroiditis.
D. Insulinoma.
E. Adreno-genital syndrome.

Secondary diabetes occurs in the following endocrine diseases except:


A. Acromegaly.
B. Itsenko-Cushing's disease.
C. Glucagon.
D. Prolactinoma.
E. Primary aldosteronism.

Glycosylation of proteins is:


A. The process of glucose attachment to proteins by the enzyme glucosyltransferase.
B. Glucose attachment to protein by glucotransferase enzyme.
C. Attachment of glucose to proteins by non-enzymatic method.
D. Combination of glucose with proteins in the glucose reductase reaction.
E. Combination of glucose with proteins with the participation of the enzyme
glucosynthetase.

Insulin-dependent diabetes mellitus is characterized by:


A. The onset of the disease in old age.
B. Sudden development.
C. Weight loss.
D. A labile course.
E. The onset of the disease at a young age.

A guy 16 years old complains of severe weakness, lethargy, thirst, increased urination, itching,
weight loss, recently contracted mumps. In your opinion, what is the most likely diagnosis?
Asthenic syndrome after viral infection.
A. Reinfection.
B. Puberty.
C. Diabetes mellitus.
D. Psychogenic polydipsia.

The 55-year-old artist is denied a diagnosis of diabetes. Fasting blood glucose: 4.46; 5,0; 5.4
mmol / l, determined during the day - 12.1; 11.12; 11.0 mmol / L. What are your actions?
A. Make the diagnosis: Diabetes mellitus.
B. Support the position of previous colleagues.
C. Assign TSH.
D. Assign a glycemic profile.
E. Determine daily glucosuria.

The patient with rheumatoid polyarthritis, about which he constantly receives various anti-
inflammatory therapy, is diagnosed with diabetes of mild form (glycemia - 7.2 mmol / l on an
empty stomach). The cause of hyperglycemia may be:
A. Reception of salicylates.
B. Acceptance of glucocorticoids.
C. Acceptance of gold preparations.
D. Violation of diet and consumption of foods rich in easily digestible carbohydrates.
E. Application of physical activity and physical procedures.

Glycosylation of hemoglobin in red blood cells occurs in:


A. The period of erythropoiesis.
B. Throughout the life of the erythrocyte.
C. During the first 10 days of erythrocyte life.
D. In the first month of erythrocyte life.
E. In the period of erythrocyte degradation.

Which of these glycosylation products most accurately reflects the nature of glycemia, which
was 10 days ago?
A. NV AIS.
B. NV AI.
C. NV AI.
D. Fructosamine.
E. Total glycosylation products of hemoglobin.

The definition of IRI is used to:


A. Diagnosis of type 1 diabetes.
B. Diagnosis of type 2 diabetes.
C. Dose adjustments for insulin.
D. Determination of insulin dose in the first diagnosed type 1 diabetes.
E. Assessment of functional status of pancreatic beta cells.

The definition of C-peptide allows you to:


A. Assess the status of carbohydrate metabolism compensation.
B. Correct the insulin therapy.
C. Correct the use of oral hypoglycemic agents.
D. Diagnose the cause of diabetes lability.
E. Set the type of diabetes.

The level of C-peptide, as a rule:


A. Reduced by type 2 diabetes.
B. Reduced by type 1 diabetes.
C. Reduced by insulin.
D. Activated by insulin therapy.
E. Increases with glucagon.

The definition of C-peptide allows to judge the secretion of insulin:


A. Only in patients who have never received insulin.
B. Only in the first cases of type 1 diabetes.
C. Only persons with type 2 diabetes who have never taken oral antihypertensives.
D. Against the background of insulin therapy.
E. For long-term hyperglycemia.

The definition of glucosuria allows:


A. Diagnose diabetes.
B. Diagnose diabetes type.
C. Diagnose the clinical course of diabetes.
D. Diagnose glucose tolerance disorders.
E. Assess the renal threshold for blood glucose.

A patient with suspected carbohydrate metabolism was administered an oral glucose tolerance
test: a fasting heart of 5.4 mmol / l, after 1 hour. - 12,5 mmol / l, after 2 years. - 8.1 mmol / l.
Evaluate the test results.
A. Impaired glucose tolerance.
B. Norm.
C. Clear diabetes.
D. The results are not reliable.
E. A thorough oral glucose tolerance test must be performed.

The 55-year-old artist is denied a diagnosis of diabetes. Fasting blood glucose: 4.46; 5,0; 5.4
mmol / l, determined during the day - 12.1; 11.12; 11.0 mmol / L. What are your actions?
A. Make the diagnosis: Diabetes mellitus.
B. Support the position of previous colleagues.
C. Assign TSH.
D. Assign a glycemic profile.
E. Determine daily glucosuria.

The patient with rheumatoid polyarthritis, about which he constantly receives various anti-
inflammatory therapy, is diagnosed with diabetes of mild form (glycemia - 7.2 mmol / l on an
empty stomach). The cause of hyperglycemia may be:
A. Acceptance of glucocorticoids
B. Reception of salicylates
C. Acceptance of gold preparations
D. Violation of diet and consumption of foods rich in easily digestible carbohydrates
E. Application of physical activity and physical procedures.

Glycosylation of hemoglobin in red blood cells occurs in:


A. The period of erythropoiesis.
B. Throughout the life of the erythrocyte.
C. During the first 10 days of erythrocyte life.
D. In the first month of erythrocyte life.
E. In the period of erythrocyte degradation.

Evaluate the results of the glucose tolerance test: fasting heart 7.0 mmol / l, after 1 hour. - 13,2
mmol / l, after 2 years. - 11.0 mmol / l.
A. Explicit diabetes mellitus.
B. Normal test.
C. Re-examination required.
D. Impaired glucose tolerance.
E. A glucose-prednisone test must be performed.

When carrying out GTT the results of glycemia were obtained: on an empty stomach - 5.1 mmol
/ l, after 1 hour. - 10.7 mmol / l after 2 years. - 8.0 mmol / l.
A. Diabetes mellitus
B. Impairment of glucose tolerance
C. Norm
D. The test is questionable
E. The test is unreliable.

The endocrinologist is addressed by the parents of the baby for 3 months, worried that the baby
is restless, especially at night, calms down only after feeding, does not gain weight. The mother
noted that urine, when dried on diapers, makes them stiff, as if starchy. What would you advise
your parents?
A. Determine blood sugar.
B. To bathe before bed in decoction of valerian root.
C. To bathe before bedtime in broth mistletoe and mint.
D. Do not use cotton diapers, but switch to disposable ones.
E. In addition to feeding, feed your baby

Female 54 years, menopause. For a long time the vagina itching is concerned. Gynecologist
treated for vulvar candidiasis. Glucosuria is absent. Fasting glycemia three times the heart: 4,5;
5,1; 5.4 mmol / L. Which of the following examinations may indicate that the patient is
suffering?
A. Conduct TSH.
B. Determine the sensitivity of the candidate to the medication.
C. Examine in the mycology laboratory to determine the serotype of the candidate.
D. Determine daily glucosuria.
E. Determine the concentration of fructosamine.

What indicators of glucose tolerance test should be diagnosed with diabetes mellitus:
A. Fasting heart more than 6.1 mmol / l, 2 hours after TSH more than 11.1 mmol / l.
B. Fasting from 5.5 to 6.1 mmol / L.
C. Even more than 6.1 mmol / l, more than 7.8 mmol / l 2 hours after TSH.
D. More than 11.1 mmol / L 2 hours after TSH.
E. Fasting from 3.3 to 5.5 mmol / l.

The definition of glycosylated hemoglobin is used to:


A. Estimates of diabetes compensation for three months.
B. Indicative dose of insulin therapy for first-time diabetes.
C. Dose adjustments for ketosis.
D. Dose adjustments for oral hypoglycemizates.
E. Diabetes mellitus daily monitoring.

Patient 38 years old, constantly taking glucocorticoids for bronchial asthma. Recently, there was
a thirst, increased appetite, became frequent urination. Glycemia during the day: 8.9; 7.7; 9.1;
7.8; 8.1 mmol / L. Probable diagnosis?
A. Kidney diabetes
B. Functional impairment of carbohydrate metabolism
C. Type 1 diabetes.
D. Type 2 diabetes
E. Secondary diabetes mellitus.

In the examination of the teenager revealed glucosuria - 2 g / l, glycemia 4, 5 mmol / l. What is


your previous diagnosis?
A. Renal glucosuria.
B. Diabetes mellitus.
C. Diabetes mellitus type 2.
D. Diabetes mellitus type 1.
E. None of the above diagnoses.

Patient G., 40 years old, has a height of 160 cm, body weight 100 kg. Diabetes mellitus is
actively detected for the first time. Fasting glycemia of the heart is 12 mmol / l, glucosuria is
16.6 mmol / l (30 g / l). What type of diabetes can we talk about?
A. 1 CD type.
B. Type 2 CD.
C. Secondary diabetes due to obesity.
D. Diabetes of the young.
E. Mature Diabetes.

In the patient of 56 years, the examination revealed hyperglycemia - 8.0 mmol / l and glucosuria
- 15 g / l. Similar indicators were also obtained 3 days ago. Make a preliminary diagnosis:
A. Impairment of glucose tolerance.
B. Diabetes mellitus first detected, type 2 (non-insulin-dependent), mild to moderate.
C. Diabetes mellitus type 2 (insulin-independent), moderate in decompensation.
D. Diabetes mellitus first detected, type 2 (insulin-independent), mild to severe in
decompensation.
E. Diabetes mellitus first detected, type 1 (insulin dependent), moderate in decompensation.

Patient K., 18 years old, who is 168 cm tall and weighs 60 kg, first discovered fasting heart
glycemia 16 mmol / l, daily glucosuria 60 g / l. Diagnose:
A. Diabetes mellitus, an insulin-dependent type, is first identified.
B. Diabetes mellitus is the first to be found, insulin-dependent, severe in compensation.
C. Diabetes mellitus, insulin-independent, moderate in decompensation.
D. Diabetes mellitus, insulin dependent, severe in subcompensation.
E. None of these diagnoses.

A woman was contacted by an endocrinologist in connection with pregnancy miscarriage. The


anamnesis revealed that the first pregnancy ended with the death of the fetus in the womb at 38
weeks, two after which ended with early spontaneous abortions. Each time under. glucosuria is
observed during pregnancy. When viewed, obesity. Which of the following surveys is a top
priority?
A. Determine the type of dyslipoproteinemia.
B. Medical genetic counseling.
C. Perform basal thermometry.
D. Determine glucose tolerance.
E. Determine the level of IRI.

Patient B., 16 years old, with complaints of persistent glucosuria. Glucosuria was first recorded
at the age of 6 and maintained within 0.5–1–1.5%. Glucosuria is not related to the nature of the
diet. Ketonuria has never been. Glycemia 3.6-4.5 mmol / L. Previous diagnosis?
A. Type 1 diabetes mellitus.
B. Type 2 diabetes in young people.
C. Kidney diabetes.
D. Transient glucosuria.
E. Juvenile glucosuria.

Patient B., 26 years old, diagnosed with "Diabetes mellitus first discovered" was referred for
consultation. During blood glucose examination during the day: 8 hours - 3,6; 11 years - 3,8;
14 years - 4,0; 17 - 4.2; 21 years - 3.6 mmol / l. Daily diuresis 3.6 l, glucosuria 3%, proteinuria
1.36 g / l. What is the most likely diagnosis in this case?
A. Type 1 diabetes mellitus.
B. Type 2 diabetes mellitus young.
C. Juvenile diabetes mellitus.
D. Kidney diabetes.
E. Tubulopathy.

Patient S., 32 years old, referred for consultation in connection with persistent incurable
periodontosis. Preliminary data: fasting glycemia 6.0 mmol / l, 10.2 mmol / l 2 hours after
eating. Glucose is absent in morning urine. Which of the following examinations is most
appropriate for the patient to assess carbohydrate metabolism?
A. Determine fasting glycemia three times in a row.
B. Determine glucose in the daily portion of urine.
C. Determine glycemia 4 times a day for nutritional load.
D. Carry out TSH.
E. Determine fructosamine concentration.

Female 54 years, menopause. For a long time the vagina itching is concerned. Gynecologist
treated for vulvar candidiasis. Glucosuria is absent. Fasting glycemia three times the heart: 4,5;
5,1; 5.4 mmol / L. Which of the following examinations may indicate that the patient is
suffering?
A. Examine in the mycology laboratory to determine the candidate serotype.
B. Determine the sensitivity of the candidate to the medication.
C. Carry out TSH.
D. Determine daily glucosuria.
E. Determine fructosamine concentration.

1. A 35-year-old man has type 1 diabetes, chronic cholecystitis. Receives Humodar B Insulin
(Medium Term Insulin) - 20 Units in the morning, 12 units Tonight. After eating, there was
pain in the right hypochondrium, nausea, vomiting, drowsiness, increased polyuria. What
assistance will most reliably prevent the development of a critical state in the coming hours?
A. The use of painkillers.
B. Changing insulin therapy to intensive.
C. The use of choleretic agents.
D. Exclusion of fats from food.
E. Reducing the carbohydrate quota in food.

2. A 17-year-old patient injects insulin into the lateral surfaces of the shoulders, abdomen, and
thighs. Feels most comfortable with the drug in the abdomen. How can this be caused?
A. By hinting
B. Fewer nerve receptors
C. The rate of insulin absorption
D. Convenience of introduction of insulin in a stomach
E. Injection accessibility.

3. The parents of a 12-year-old boy with type 1 diabetes for a month contacted an
endocrinologist for frequent hypoglycemic conditions. Receives a basic bolus scheme of insulin
therapy. Objectively: height 165 cm, weight 64 kg. Tongue moist, without layers. Internal
organs without features. Laboratory: glycemia - 3.6 mmol / l. What daily dose of insulin should
be given to a patient who is likely to have a remission phase?
A. 0.3 units / kg / day
B. 0.3-0.6 units / kg / day
C. 0.8-0.9 units / kg / day
D. 0.6-0.7 units / kg / day
E. 1 unit / kg / day

4. A 22 year old patient with type 1 diabetes periodically changes insulin injection sites. Which
of the following sites provides optimal pharmacokinetic characteristics of insulin drugs?
A. Lateral surfaces of the abdomen.
B. Lateral surfaces of the shoulders.
C. The semicellular area
D. Upper third of thigh.
E. Lower third of thigh.

5. Diabetic patients receiving insulin at a dose of 8 U of short action, 20 U of medium action


before breakfast and 6 U of short action, 16 U of average action before dinner, in the last days
increased sweating and feeling hungry at first in the morning . What changes in treatment
should be made?
A. Increase insulin dose in the middle of the morning.
B. Reduce mid-dose insulin dose in the evening.
C. Reduce short-acting insulin dose in the morning.
D. Reduce short-acting insulin dose in the evening.
E. Increase calorie intake in the evening.

6. Patient R., 33 years old, suffering from type 1 diabetes for 8 years, is on base-bolus therapy.
He went on a business trip forgetting insulin that he had not used for 2 days. On examination the
skin is dry, cold, blood pressure 115/60 mm Hg, pulse - 96 beats per 1 minute. HBA1c - 7.4%.
glycemic profile 8.00- 16.4 mmol / l; 11.00 - 18.3 mmol / l; 13.00-15.8 mmol / l; 16.00-17.5
mmol / l; 21.00 - 16.6 mmol / l; 3.00 - 14.5 mmol / L. Urine sugar 2.5%, acetone - traces.
What is your treatment tactic?
A. Strengthen dietary regimen
B. Resume insulin at previous doses
C. Increase previous insulin dose by 6-8 units
D. Transfer to short-acting insulin to compensate for the condition
E. Decrease previous insulin dose by 6-8 units.

7. A type 1 diabetic patient has a rational diet for diabetic patients, based on proteins, fats and
carbohydrates. What is the right ingredient ratio?
A. Carbohydrates 50-60%, fats 20-30%, proteins 15-20%
B. Carbohydrates 40-50%, fats 15-25%, proteins 25-30%
C. Carbohydrates 50-60%, fats 30-40%, proteins 10-20%
D. Carbohydrates 60-70%, fats 10-20%, proteins 15-20%
E. Carbohydrates 40-50%, fats 20-30%, proteins 20-30%

8. In a 37-year-old woman with influenza, fasting glycemia was first detected at -8.6 mmol / l,
and glucosuria at 22 g / l. Height 158 cm, body weight 61 kg. What is the most informative
study to clarify the diagnosis?
A. Fasting insulinemia.
B. Glycemia after eating.
C. Test glucose tolerance
D. Daily fluctuations in glucosuria.
E. Daily fluctuations in glycemia.

9. Patient, 19 years old, height 178 cm, weight 68 kg, first examination revealed fasting glycemia
16 mmol / l, HbA1c - 9.2%. Probable diagnosis:
A. Diabetes mellitus, first detected, type 1, compensation status.
B. Diabetes mellitus, first detected, type 1, subcompensation status
C. Diabetes mellitus, first detected, type 1, decompensation status.
D. Diabetes mellitus, type 1, severe, subcompensated.
E. Diabetes mellitus, type 1, mild, subcompensated

10. Diabetes mellitus was first detected in a patient 28 years after suffering SARS. Fasting
glycemia - 15.2 mmol / l; glucosuria - 2.5 g / l; glycated hemoglobin - 10.1%. What type of
diabetes therapy should you think of?
A. Biguanides.
B. Derivatives of sulfonylureas
C. Thiazolidinediones.
D. DPP-4 inhibitors
E. Insulin therapy.

1. Patient V., 23 years old, recently became ill with type 1 diabetes. In the CRL, where the
patient was treated, she was prescribed insulin therapy with Pharmasulin H 30/70 at a dose of 30
units (18 in the morning and 12 in the evening). The constitution is normostenic, height 165 cm,
weight - 64 kg, BMI - 22,9 kg / m2, НВА1с - 8,6%. glycemic profile 8.00-8.4 mmol / l; 11.00 -
13.1 mmol / l; 13.00-11.8 mmol / l; 16.00-16.2 mmol / l; 21.00 - 6.6 mmol / l; 3.00 - 10.5
mmol / L. Your treatment tactics.

A. Strengthen dietary regimen


B. Translate into the basis-bolus scheme of insulin therapy
C. Increase morning dose of insulin Farmasulin H 30/70
D. Increase the evening dose of insulin Farmasulin H 30/70
E. Increase morning and evening doses of Farmasulin H 30/70 insulin

2. A boy of 7 years has had diabetes for 1 year. Receives insulin (short and medium duration) at
the rate of 0.4 U / kg of body weight per day. Insulin should be injected under the skin of the
shoulder with a syringe. What measures should be taken to prevent lipodystrophy?
A. Restrict fats in your baby's diet.
B. Periodically switch to another type of insulin.
C. Decrease insulin dose.
D. Change the place of insulin injection regularly.
E. Prescribe antioxidants.

3. A patient of 15 years has the benefit. Complaints of thirst, polyuria, weakness, for 2 weeks
lost 1,5 kg. Objectively: the overall condition is satisfactory. Fasting blood glucose level is 11.3
mmol / l, in urine 4%, acetone +. What is your anti-aberrant therapy tactic?
A. Short acting insulin
B. Prolonged-acting insulin
C. Basic bolus insulin therapy
D. Biguanides
E. Thiazolidinediones

4. Patient P., 44 years old, has been suffering from type 1 diabetes for 12 years. 3 years ago, she
was diagnosed with diabetic nephropathy, a stage of microalbuminuria, seeking treatment at
Truskavets. НВА1с 7.1%, good self-control is good. Is resort rehabilitation appropriate in the
conditions of the Truskavets resort?
A.Categorically contraindicated.
B. Not desirable because of the threat of decompensation of diabetes
C. Possible, but without the resort's mineral water
D. There are no contraindications
E. The filtration capacity of the kidneys may be impaired.

5. The girl was hospitalized for 8 years. According to the parents, over the past two months the
baby lost 2 kg, started drinking more, complaining of fatigue, abdominal pain, nausea. On
examination: the height of the baby 130 cm, weight 28 kg, sore cheeks, tongue dry, skin turgor
reduced, the smell of acetone on exhalation. Daily diuresis of about 2 liters. Blood glucose -
11.4 mmol / l, urine acetone (-). What treatment should be prescribed at this time?
A. A diet high in carbohydrates
B. Biguanides
C. Ultra short-acting insulin analogues 4-6 times a day under glycemic control.
D. Long-acting insulin analogues once daily under glycemic control.
E. Short acting insulin 2 times a day and long acting insulin 2 times a day

6. A 24-year-old man who has been suffering from type 1 diabetes for 10 years has approached
an endocrinologist. He is currently receiving basic and short-acting insulin therapy. Due to
frequent glycemic conditions at night and at lunch, he requested to change the insulin therapy
regimen. Which insulin should be given instead of medium-acting insulin?
A. Insulin Glulizin
B. Insulin Glorgin
C. Insulin Aspart
D. Pharmasulin H
E. Pharmasulin HNP.

7. Patient 31 years old with diabetes for 25 years. Recently there have been frequent
hypoglycemic conditions, with daily insulin requirements reduced by 16. In the general urine
analysis, the protein is 1.4 g / l, single leukocytes and altered red blood cells; in the general
blood test - signs of anemia. GKF-27 ml / min / 1.73 m2. The specified dynamics of the disease
can be caused by:
A . Concomitant pyelonephritis.
B.Diabetic kidney disease
C. Systemic amyloidosis.
D. Iron deficiency anemia.
E. Decompensation of diabetes.

8. A guy 13 years old after a stressful situation at school complains of weight loss, thirst,
frequent urination, especially at night. Diuresis 3.2 liters Urine specific gravity 1034, fasting
blood glucose content of 7.5 mmol / l, after eating 12.1 mmol / l. What is the diagnosis?
A. Type 1 diabetes
B. Type 2 diabetes
C. Diabetes mellitus
D. Fasting glucose disorders
E. Secondary diabetes mellitus.

9. A 49-year-old woman consulted an endocrinologist to correct carbohydrate metabolism. Has


been ill for 5 years with type 2 diabetes. Objective status: BP 140/85 mm Hg, BH 18 beats /
min. Fasting glycemia does not exceed 6.9 mmol / l; in urine - glucosuria. Which of the
following methods is most informative for assessing diabetes compensation?
A. Determination of glucosuric profile
B. Determination of glycemic profile
C. Determination of glycated hemoglobin
D. Test glucose tolerance
E. Determination of fasting glucose

10. Girl 11 years hospitalized with complaints of polyuria, polydipsia, weight loss in the last 3
months by 25%. The examination revealed glycemia of 16 mmol / l, acetone in the urine (+++).
Diagnosis of diabetes for the first time. What should be the most likely daily dose of insulin?
A. 0.1 U / kg.
B. 0.25 U / kg.
C. 0.5 U / kg.
D. 0.7 U / kg.
E. 1 U / kg.

1. A 52 year old patient receives 90 U of insulin (0.9 U / kg body weight) per day for 3 years.
Prior to this he was treated with oral antipyretic drugs of a group of derivatives of sulfonylurea.
Over the last 12 months, his weight has increased by 8 kg with increased appetite. Fasting blood
sugar is 13 mmol / l, in daily urine glucosuria is 3%. What is the most likely diagnosis:
A. Diabetes mellitus, type 1, subcompensation.
B. Diabetes mellitus, type 1, severe decompensation.
C. Diabetes mellitus, type 2, severe decompensation.
D. Diabetes mellitus, type 2, severe decompensation. Somoji syndrome.
E. None of the above.

2. The patient is treated with insulin Farmasulin H 30/70 (medium and short duration combined
insulin) at a dose of 26 U in the morning and 14 U in the evening. He is being changed to a
combination of Pharmasulin H (short-acting insulin) and Pharmasulin HNP (long-acting insulin).
Will the daily dose of insulin change?
A. The total dose of insulin is usually increased.
B. No, it remains the same.
C. The evening dose of the drug is reduced.
D. Evening dose decreases, but morning dose increases.
E. The total insulin dose is reduced by 15%.

3. Patient 62 years old, diagnosed with type 2 diabetes 7 years ago. Adheres to the diet and takes
glibenclamide. HbA1c-7.1%. The patient is scheduled surgery for inguinal hernia. What should
be the tactics for diabetes treatment?
A. Replace glibenclamide with glimepiride.
B. Add biguanides.
C. Prescribe short-acting insulin preparations.
D. To prescribe long-acting insulin preparations.
E. To prescribe a combination of medium and short-acting insulin.

4. In a patient of 21 years, who has diabetes mellitus type 1 severe, decompensated disease with
the development of ketoacidotic condition. Treatment of this condition was carried out by
repeated dose administration of small doses of short-acting insulin and isotonic sodium chloride
solution. After 2 hours the patient had headache, sweating, tremor. Blood sugar is 3.1 mmol / l,
sodium content is 140 mmol / l, potassium is 3.5 mol / l. What caused this condition?
A. Side effect of the short-acting insulin preparation.
B. The development of hypokalemia.
C. The development of hypoglycemia.
D. Intercurrent disease.
E. Ketoacidosis intoxication.

5. Patient F., 36 years old with type 1 diabetes mellitus 8 years, receiving insulin therapy
Pharmasulin H 12 units and Pharmasulin HNP 18 units (8.30), in the evening Pharmasulin H 14
units (13.30), in the evening Pharmasulin H 10 units and Farmasulin HNP 14 units (6.30pm).
Patient HBA1c 8.4%. glycemic profile 8.00- 14.1 mmol / l; 11.00 - 8.7 mmol / l; 13.00-5.7
mmol / l; 16.00-8.3 mmol / l; 21.00 - 8.9 mmol / l; 3.00 - 4.4 mmol / L. Urine sugar 0.5%,
acetone (0). Is it advisable to change treatment tactics?

A. No change in therapy is needed


B. Strengthen dietary regimen
C. Reduce the evening dose of Farmasulin HNP
D. Increase the dose of evening Pharmasulin HNP
E. Postpone dose of evening Pharmasulin HNP at 22.00-23.00

6. In a patient of 27 years suddenly after a stressful situation there was dry mouth, frequent
urination, weight loss. Fasting glycemia 8.2 mmol / l, in the urine 2% sugar. In the development
of the disease the leading role is played by:
A. Increased glucagon secretion
B. Increased secretion of somatostatin
C. Reduction of functional capacity of pancreatic beta cells.
D. Reduction of permeability of the basement membrane of glomerular glomeruli
E. Adrenal hyperfunction

7. Determination of the level of immunoreactive insulin is carried out:


A. Persons receiving continuous insulin.
B. In individuals receiving a combination of insulin with antipyretic drugs.
C. In individuals who have never received insulin.
D. In patients with diabetes mellitus.
E. Diabetic patients with a tendency to ketonuria.

8. After a summer of ARVD, a 9 year old baby had a dry mouth and thirst, which parents regard
as a reaction to the hot weather, as well as nausea and headache, vomiting. During the
examination the baby is sluggish, drowsy, marked tachycardia, decreased turgor and dryness of
the skin, muscle weakness. Which of the following laboratory tests should you do first?
A. Glucose tolerance test
B. The concentration of insulin in the blood
C. The content of C-peptide in the blood
D. The content of acetone in the urine
E. Antibodies to the Langerhans Islands

9. A 55-year-old man complains of increased urination, general weakness, weight loss of 1 kg


per week, glucose tolerance test was performed: fasting heart capillary blood glucose - 5.8 mmol
/ l, after 8 hours - 8.6 mmol / l. Evaluate the test results.
A. Normal test.
B. Questionable test
C. Fasting glycemia.
D. Impairment of glucose tolerance.
E. Clear diabetes.

10. A man, 37 years old, complains of weight loss, moderate dry mouth, constant thirst,
polyuria. According to the patient has been ill for about 10 months, he has not received any
treatment. Objectively: temperature - 36,5C, CDR - 18 per minute, pulse - 76 beats / min, blood
pressure - 130/80 mm Hg. Art. The skin and mucous membranes are moderately dry. Fasting
glycemia - 14 mmol / l, glucosuria - 20 g / l. Which study will help determine the type of
diabetes?
A. Test glucose tolerance
B. Determination of glycated hemoglobin.
C. Determination of blood insulin levels.
D. Glycemic profile.
E. Determination of blood glucagon level.

1. The patient is 17 years old. Ill acutely. Thirst, polyuria, weakness, for 2 weeks lose 4 kg.
Objectively: the overall condition is satisfactory, there is no acetone odor. Glucose level: in the
fasting blood - 22 mmol / l, in the urine - 6%, acetone (+). What are your tactics for reducing
therapies?
A. Diet therapy.
B. Purpose of sugar-reducing herbs.
C. The purpose of insulin therapy.
D. Purpose of biguanides.
E. Purpose of sulfonylurea derivatives.

2. Patient 17 years old with type 1 diabetes 3 years. Receives insulin: before breakfast and
dinner - short and prolonged action, before lunch - short. Sleep restless, in the morning -
weakness, headache. Glycemia: 8-00 - 10.1 mmol / l, 12-00 - 6.6 mmol / l, 17-00 - 4.3 mmol / l,
21-00 - 3.7 mmol / l. In the morning portion of urine, acetone +. What is the cause of
acetonuria?
A. Extra dose of prolonged insulin before dinner
B. Insufficient dose of long acting insulin before dinner
C. Insufficient dose of long-acting insulin before breakfast
D. Excess dose of short acting insulin before dinner
E. Insufficient daily dose of long-acting insulin

3. During the study of type 1 diabetes patients, the nurse explained the benefits of intensified
insulin therapy over traditional ones. Which of the following statements is not true?
A. Provides rapid CD compensation.
B. Improves the quality of life of patients.
C. Simulates as much as possible the physiological secretion of insulin.
D. Minimizes the risk of hypoglycemia.
E. Minimizes the risk of developing chronic complications of diabetes.

4. 3 months after delivery, a 27-year-old woman who was previously diagnosed with gestational
diabetes showed an increase in blood glucose up to 7.6 mmol / l on an empty stomach and 11.8
mmol / l after a meal, HbA1c - 6.9%. What diagnosis should you think of now?
A. Type 1 diabetes
B. Type 2 diabetes
C. Gestational diabetes
D. Fasting glucose disorders
E. Impairment of glucose tolerance

5. Patient A., 23 years for 2 weeks to bother regular hypoglycemic states in the first half of the
day, not associated with physical activity and a violation of diet. Receives insulin therapy,
Farmasulin N 14 and Farmasulin НNP 26 ed (8.30 am), lunch Farmasulin N 10 (13.30), in the
evening Farmasulin N 10 units and 14 units Farmasulin HNP (18.30). The patient НвА1с of
9.1%. glycemic profile 8.00 - 7.1 mmol/l; 11.00 - 3.7 mmol/l; 13.00-3.2 mmol/l; 16.00-14,3
mmol/l; 21.00 – 7.8 mmol/l; 3.00 – 5.4 mmol/l; urine sugar 0.5%; acetone (0). Your tactics?
A. To enhance diet regime in the 2nd half of the day
B. To strengthen the therapy in the 2nd half of the day
C. To reduce the dose of evening Farmasulin HNP
D. To reduce the dose of the morning Farmasulin HNP
E. To reduce the dose of the morning Farmasulin N and Farmasulin HNP

6. The endocrinologist said the woman 21 years old, who suffers with diabetes since the age of
15. Recently noticed frequent changes in blood sugar levels, which connects with the irregular
and chaotic eating, as studies on stationary form of education. The doctor explained the necessity
of counting of pieces of bread and made a decision to change the short-acting insulin analogue of
human insulin ultra short-acting. Which of the following drugs chosen by the doctor?
A. Insulin Aspart
B. Insulin Actrapid
C. Farmasulin N
D. Farmasulin HNP
E. Insulin Glargin.

7. Patient 27 years old, who suffers from type 1 diabetes assigned to monotherapy with insulin
average duration of action of 22 IU in the morning. Will offer the optimal variant of correction
of insulin therapy of the following:
A. To distribute the daily dose of insulin average duration of action for two injections.
B. In the morning to inject insulin average duration of action, and in the evening short-acting
insulin.
C. In the morning to enter the short-acting insulin and night insulin average duration of action
D. To inject insulin average duration of action twice a day, and the short-acting insulin before
each eating.
E. Add Metformin.

8. Which of the following insulins is characterized by the onset of action 0.5-1 h, peak activity 2-
3 h, duration of action 5-8 h?
A. Humulin R.
B. Protafan NM
C. Insuman Basal
D. Humulin 30/70
E. Insuman Comb 25

9. In a patient of 30 years the first revealed glycemia 15 mmol / l, daily glucosuria 40 g / l,


acetone in the urine +++. The last two weeks I have experienced increased appetite, thirst,
frequent urination, weakness. Make a diagnosis?
A. Type 1 diabetes is first detected in the stage of decompensation.
B. Type 1 diabetes is first identified in the subcompensation stage.
C. Type 2 diabetes is first detected in the stage of decompensation.
D. Type 1 diabetes is the first to be found in severe compensation.
E. Type 1 diabetes is first identified as moderate in compensation.

10. A patient of 26 years has diabetes for 3 years. Receives Humodar P (short-acting insulin) 6
IU and Humodar B (medium-acting insulin) 20 IU in the morning and Humodar P 4 IU and
Humodar B 16 IU in the evening. Complains of moderate dry mouth and headache in the
morning, poor sleep at night. Objective data without pathological changes. Glycemic profile: 8
hours - 14 mmol / l, 14 years. - 7 mmol / l, 19 years. - 6 mmol / l, 3 years - 3.1 mmol / l.
Glucosuria (night serving) - 0.3 l - 0.5%. What is the most appropriate therapeutic tactic in this
case?
A. Reduce your morning dose of Humodar B
B. Reduce evening dose Humodar B.
C. Increase morning dose of Humodar B.
D. Increase your morning dose of Humodar R.
E. Increase evening dose of Humodar B.

1 A 42-year-old woman with a BMI of 24.0 kg / m2 receives short-acting insulin-type 1 diabetes


prior to each meal and prolonged exposure before breakfast and dinner. The dose of insulin
itself is not regulated. A woman eats for breakfast 5 HO, for lunch 6 HO, for dinner 5 HO.
Every day before dinner he feels thirsty, dry mouth (blood sugar 10.9 mmol / l). Height - 168
cm, weight - 76 kg. How to prevent the development of these symptoms?
A. Reduce Carbohydrate (BU=bread units))
B. Change the insulin dose.
C. To appoint metformin.
D. Assign sulfonylureas.
E. Apply beta blockers.

2. Patient M., 32 years old, ambulance driver complains of weight loss (18 kg in the last 2
months), polyuria, polydipsia. Fasting glycemia 18.4 mmol / l, glycosuria 3.0%, ketone bodies
(+). What is your healing tactic?
A. Diet Therapy table # 9 by Pevsner.
B. Diet therapy table # 9 by Pevsner and biguanides
C. Diet therapy table # 9 by Pevsner and sulphonylureas.
D. Diet therapy table # 9 by Pevsner and insulinotherapy.
E. Diet Therapy table # 9 by Pevsner and Solikva (Glargin + Lixisenatide).

3. A 35-year-old man has type 1 diabetes, chronic cholecystitis. Receives Lantus insulin (long
acting insulin) 28 in the morning. After eating, there was pain in the right hypochondrium,
nausea, vomiting, drowsiness, and polyuria increased. What assistance will most reliably
prevent the development of a critical condition in the coming hours?
A. The use of painkillers
B. The use of choleretic agents
C. Exclusion of fats from food
D. Adding short-acting insulin before meals
E. Apply Lantus insulin twice a day

4. The level of C-peptide, as a rule:


A. Reduced for type 2 diabetes.
B. Reduced for type 1 diabetes.
C. Reduced by insulin.
D. Activated by insulin therapy.
E. Increases by glucagon.

5. The patient is 17 years old. Ill acutely. Thirst, polyuria, weakness, for 2 weeks lose 4 kg.
Objectively: the overall condition is satisfactory, there is no acetone odor. Glucose level: in the
fasting blood - 22 mmol / l, in the urine - 6%, acetone (+). What are your tactics for reducing
therapies?
A. Diet therapy.
B. Purpose of sugar-reducing herbs.
C. The purpose of insulin therapy.
D. Purpose of biguanides.
E. Purpose of sulfonylurea derivatives.

6. Girl 11 years hospitalized with complaints of polyuria, polydipsia, weight loss in the last 3
months by 25%. The examination revealed glycemia of 16 mmol / l, acetone in the urine (+++).
Diagnosis of diabetes for the first time. What should be the most likely daily dose of insulin?
A. 0.1 U / kg.
B. 0.25 U / kg.
C. 0.5 U / kg.
D. 0.7 U / kg.
E. 1 U / kg.

7. The girl was hospitalized for 8 years. According to the parents, in the last two months the
baby lost 2 kg, started drinking more, complaining of fatigue, abdominal pain, nausea. On
examination: the height of the baby 130 cm, weight 28 kg, sore cheeks, tongue dry, skin turgor
reduced, the smell of acetone on exhalation. Daily diuresis of about 2 liters. Level blood
glucose - 11.4 mmol / l, urine acetone (-). What treatment should be prescribed at this time?
A. A diet high in carbohydrates.
B. Biguanides.
C. Ultra short-acting insulin analogues 4-6 times a day under glycemic control.
D. Long-acting insulin analogues once daily under glycemic control.
E. Short acting insulin 2 times a day and long acting insulin 2 times a day.

8. A 55-year-old man complains of increased urination, general weakness, weight loss of 1 kg


per week, glucose tolerance test was performed: fasting heart capillary blood glucose - 5.8 mmol
/ l, after 8 hours - 8.6 mmol / l. Evaluate the test results:
A. Normal test.
B. Doubtful test
C. Fasting glycemia.
D. Impairment of glucose tolerance.
E. Overt diabetes.

9. A man, 37 years old, complains of weight loss, moderate dry mouth, constant thirst, polyuria.
According to the patient has been ill for about 10 months, he has not received any treatment.
Objectively: temperature - 36,5C, CDR - 18 per minute, pulse - 76 beats / min, blood pressure -
130/80 mm Hg. Art. The skin and mucous membranes are moderately dry. Fasting glycemia -
14 mmol / l, glucosuria - 20 g / l. Which study will help determine the type of diabetes?
A. Test glucose tolerance
B. Determination of glycated hemoglobin.
C. Determination of blood insulin levels.
D. Glycemic profile.
E. Determination of blood glucagon level.

10. A 49-year-old woman consulted an endocrinologist to correct carbohydrate metabolism.


Has been ill for 5 years with type 2 diabetes. Objective status: BP 140/85 mm Hg, BH 18 beats /
min. Fasting glycemia does not exceed 6.9 mmol / l; in urine - glucosuria. Which of the
following methods is most informative for assessing diabetes compensation?
A. Determination of glucosuric profile.
B. Determination of glycemic profile.
C. Determination of glycated hemoglobin.
D. Test glucose tolerance.
E. Determination of fasting glucose.

1. Patient I., 49 years old, ill with type 2 diabetes for 8 years, from the time of the disease
constantly takes glibenclamide 5 mg 3 times a day. She has not consulted an endocrinologist for
the past three years. Over the last year, she has lost 9 kg without motivation, however, she did
not cause such weight loss. On examination, body weight 81 kg, height 166 cm, BMI - 29 kg /
m2. Laboratory: fasting blood sugar 12.0 mmol / l, after a meal - 15 mmol / l, glycated
hemoglobin - 13.2%. Other laboratory parameters within normal limits. Choose treatment
tactics:
A. Add to the glibenclamide biguanides.
B. Against the background of this antipyretic therapy, appoint gliniids (repaglinide).
C. Cancel glibenclamide and designate III generation sulfonylureas
D. Convert insulin therapy with prolonged insulin in combination with metformin
E. Transfer to combination with NSCTG 2 inhibitors

2. Patient, 54 years old, ill with type 2 diabetes for 3 years, complains of pain, fever up to 38 0C.
Ill for 3 days. Objectively: In the nasal nose, left, a cone-shaped infiltrate with a purulent core in
the center is identified, acutely painful when touched. To prescribe treatment?
A. Anesthetic therapy.
B. The opening of infiltrate.
C. Physiotherapy treatment.
D. Active anti-inflammatory therapy in a hospital setting taking into account the appointments
of the endocrinologist.
E. Symptomatic treatment.

3. 55-year-old female patient with type 2 diabetes for 5 years. It is treated with metformin at a
dose of 850 mg three times a day. Fasting glycemia of 7.8 mmol / l, two hours after eating - 9.5
mmol / l, glycated hemoglobin - 8.3%. In ZAS - proteinuria up to 0,099% protein. On
examination - height 167 cm, body weight 88 kg. (BMI - 31.6 kg / m2). Consulted by a
cardiologist: AH II. Assign treatment.
A. Do not change therapy, but recommend that you follow diet # 9 more carefully.
B. Combine metformin with prolonged insulin
C. Translate into combination with sulfonylurea derivatives.
D. Increase the daily dose of metformin to 3000 mg per day.
E. Add NZKTG-2 inhibitors to metformin.

4. Patient 64 years, suffering from type 2 diabetes for 7 years, became infected with infectious
hepatitis A. During the last 2 years he received glibenclamide 15 mg per day, fasting glycemia -
13.6 mmol / l, HbA1c-10.1% . Determine the tactics of further treatment:
A. Transfer the patient to insulin therapy.
B. Replace glibenclamide with glimepiride.
C. Additionally, the appointment of biguanides.
D. Increase the dose of glibenclamide to 20 mg per day.
E. Additionally appoint thiazolidinediones.

5. Patient 52 years of complaints did not express, accidentally during examination revealed type
2 diabetes. Objectively: BMI is 26.8 kg / m2. Pulse - 76 / min, rhythmic, blood pressure -
160/90 mm Hg. Art. Fasting glycemia - 7.1 mmol / l. glucose content in daily urine - 1%;
diuresis - 2.5 l. HbA1c-7.2%. AST-31 units / l, ALT-34 units / l. SKF-92 mm / min / 1.73 m2.
What is the primary treatment tactic?
A. To appoint to the patient only diet therapy.
B. To appoint metformin
C. Prescribe glybeclamide.
D. To appoint dapagliflozin.
E. Prescribe insulin.

6. Patient 56 years old, suffering from type 2 diabetes for 5 years. Adheres to the diet, takes
metformin 1000 mg 2 g / d and gliclazide at a dose of 60 mg / day. Examination results: НbA1c-
7.0%, blood sugar-5.9 mmol / l. The patient is scheduled planned laparoscopic removal of the
gall bladder. What should be the tactics for diabetes treatment?
A. Leave unchanged.
B. Replace gliclazide with pioglitazone.
C. Replace gliclazide with short-acting insulin
D. Transfer to short-acting insulin preparations.
E. Transfer to base-bolus insulin therapy.

7. Patient 52 years of complaints did not express, accidentally during examination revealed type
2 diabetes. Objectively: BMI is 26.8 kg / m2. Pulse - 76 / min, rhythmic, blood pressure -
160/90 mm Hg. Art. Fasting glycemia - 7.1 mmol / l. glucose content in daily urine - 1%;
diuresis - 2.5 l. HbA1c-7.2%. AST-31 units / l, ALT-34 units / l. SKF-92 mm / min / 1.73 m2.
What is the primary treatment tactic?
A. To appoint to the patient only diet therapy.
B. To appoint metformin
C. Prescribe glybeclamide.
D. To appoint dapagliflozin.
E. Prescribe insulin.
8. Patient 56 years old, suffering from type 2 diabetes for 5 years. Adheres to the diet, takes
metformin 1000 mg 2 g / d and gliclazide at a dose of 60 mg / day. Examination results: НbA1c-
7.0%, blood sugar-5.9 mmol / l. The patient is scheduled planned laparoscopic removal of the
gall bladder. What should be the tactics for diabetes treatment?
A. Leave unchanged.
B. Replace gliclazide with pioglitazone.
C. Replace gliclazide with short-acting insulin
D. Transfer to short-acting insulin preparations.
E. Transfer to base-bolus insulin therapy.
9. In the patient, 50 years, accidentally during the prophylactic examination revealed fasting
glycemia - 7.2 mmol / l, post-radial - 8.0 mmol / l, glucosuria - 0.5 g / l. HbA1c-6.6%. Height
169 cm, body weight 72 kg. Distinctive initial tactics for treating the patient?
A. Diet therapy
B. Biguanides
C. Derivatives of sulfonylureas.
D. Thiazolidinediones
E. Insulin
10. A 50 year old woman went to a family doctor for an increase in blood pressure. He does not
take any drugs. Besides, I noted that over the last six months the body weight has increased by
10 kg. Objectively: BMI 34 kg / m2, heart rate 76 beats / min, blood pressure 145/75 mm. Hg
Art .; heart - the borders of the heart are shifted to the left. Fasting glycemia - 8.0 mmol / l,
glycated hemoglobin - 7.2%. Which hypoglycemic drug should be assigned first.
A. Gliclazide
B. Metformin
C. Dapogliflozin
D. Insulin is glyculin
E. Detulin insulin

1. Female 49 years old, suffering from type 2 diabetes 6 years. It is treated with metformin at a
dose of 2000 mg per day. Glycemia on an empty stomach 7.2 mmol / l, two hours after a meal -
12.5 mmol / l, glycated hemoglobin - 7.9%. In ZAS, proteinuria is up to 0.033% of protein. On
examination - height 162 cm, body weight 102 kg. (BMI - 39.2 kg / m2). Make an adjustment
to diabetes.
A. Do not change therapy, but recommend that you follow diet # 9 more carefully.
B. Combine metformin with thiosolidinediones
C. Add to the metformin agonist GPP-1.
D. Translate into combination with sulfonylurea derivatives.
E. Increase the daily dose of metformin to 3000 mg per day.
2. Patient 64 years old, suffering from type 2 diabetes, osteomyelitis developed after an open
fracture of the lower extremity. Takes metformin 1500 mg / day. Fasting glycemia 8.8 mmol / l,
postprandial - 12.6 mmol / l, glucosuria 3%. Determine the endocrinologist's tactics for further
treatment of the patient.
A. Increase metformin dose.
B. Additionally appoint gliclazide MR.
C. Additionally appoint glimepiride.
D. Transfer the patient to insulin therapy.
E. None of the following.

3. Patient, 54 years old, was diagnosed with type 2 diabetes for the first time. Glycemia during
the day 8.8 - 12.9 mmol / l, glucosuria 3%, HbA1c content - 9.9%. Make a diagnosis.
A. Type 1 diabetes, first discovered in Art. decompensation.
B. Type 2 diabetes, first discovered in Art. decompensation.
C. Type 2 diabetes, lightweight in Art. subcompensation.
D. Type 2 diabetes, moderate in Art. decompensation.
E. Type 2 diabetes, moderate in Art. decompensation.

4. Patient 44 years old, suffering from type 2 diabetes mellitus 4 years, hospitalized, need to
undergo surgery for g. Appendicitis. Takes gliclazide 60 mg / day. Fasting glycemia - 6.8 mmol
/ l, daily glucosuria 1%. HbA1c-6.9%. What should be the tactics for diabetes treatment?
A. Do not change treatment.
B. Replace gliclazide with glimepiride.
C. Additionally appoint Metformin.
D. Transfer to multiple short-acting insulin injections.
E. Leave gliclazide and prescribe medium-acting insulin.

5. In a patient of 58 years, diabetes mellitus 5 years, from the beginning of the disease receives
glyclazide MR 60 mg / day, after trauma developed phlegmon of the right foot. Which treatment
option should be used in this case?
A. Add to gliclazide MR Metformin.
B. Add insulin to gliclazide MR.
C. Add to glyclazide MR acarbose.
D. Replace gliclazide MR with metformin
E. Replace gliclazide MR with insulin therapy.

6. Woman 58 years old, suffering from diabetes for 15 years. Uses glimepiride at a dose of 4
mg per day. Strictly adheres to the diet regime. History - coronary heart disease, angina pectoris
II FC. In the laboratory, it was diagnosed: fasting glycemia 8.2 mmol / l, postprandial - 9.3
mmol / l, glycated hemoglobin 8.5%. Physical examination - BMI 24 kg / m2. Perform therapy
correction.
A. Increase the daily dose of glimepiride to 6 mg and add repaglinide.
B. Glimepiride is combined with metformin at a dose of 2000 mg per day.
C. Convert to metformin in combination with emfagliflozin.
D. Dietary regimen, insulin therapy.
E. Add DPP-4 inhibitors to treatment.

7. Patient 64 years of type 2 diabetes is treated with metformin at a dose of 3000 mg per day.
Fasting glycemia 7.8 mmol / l, 10.5 mmol / l two hours after meals. HbA1c - 7.9%. Consulted
by an optometrist. Conclusion - "Diabetic proliferative retinopathy of the retina". What are the
tactics of further diabetes treatment?
A. Do not change therapy.
B. Increase the daily dose of metformin to 4000 mg per day.
C. Only translate into insulin therapy.
D. Convert to metformin with insulin
E. Convert to Metformin with Glycladide

8. A 69-year-old woman has type 2 diabetes for about 20 years. Uses glimepiride 4 mg daily.
Does not adhere to the diet regime, longer treatment time did not go. Diabetic kidney disease,
stage 4 was diagnosed with careful burden; diabetic proliferative retinopathy. HbA1c - 9.9%.
Perform therapy correction.
A. Dietary regimen, increase daily dose of glimepiride to 6mg.
B. Dietary regimen, add Metformin 1000 mg 2 times a day.
C. Dietary regimen, add pioglitazone 15 mg 2 times a day.
D. Dietary regimen, add short-acting insulin.
E. Dietary regimen, basic bolus insulin therapy.

9. Patient, 56 years old, suffering from type 2 diabetes for 10 years, constantly taking
glibenclamide 5 mg 3 times a day, recently dramatically lost weight, for three years did not
consult an endocrinologist. Examination on the right leg revealed blackening of the nail, cold
foot, pulsation weakened. Fasting blood sugar is 15.0 mmol / l, urine sugar is 2.0%. Choose
patient treatment tactics:
A. Add to the antihypertensive therapy biguanides.
B. Against the background of this antipyretic therapy, prescribe vasodilators.
C. Consult a surgeon for immediate surgical treatment of the foot.
D. Transfer to insulin therapy, consult a surgeon to determine the need for surgical treatment of
the foot.
E. Against the background of this antipyretic therapy, perform angioplasty treatment of the
lower extremity
10. Patient 56 years old, suffering from type 2 diabetes 1 year, BMI 34.7 kg. Complains of dry
mouth, thirst, nocturia. Glycemia 8.4-10.1 mmol / L. HbA1c - 7.4%. Sugar therapy is not
applicable. Adheres to diet regimen according to table 9. No liver and kidney disease were
detected. What is your further tactic?
A. Metformin 850 mg 2 times a day
B. Gliclazide MR 60 mg in the morning.
C. Dapogliflozin 10 mg.
D. Insulin therapy.
E. No correction needed.

1. The patient at the age of 50 has no complaints of dry mouth, thirst, but notes increased
appetite. After the glucose tolerance test, the glucose content in the blood after 2 hours of
loading was 11.1 mmol / l. BMI - 29.6 kg / m2, HbA1c - 6.6%. What diagnosis does this
picture point to?
A. Impairment of glucose tolerance
B. Diabetes mellitus type 1
C. Type 2 diabetes mellitus
D. Diabetes mellitus
E. Gestational diabetes

2. In a patient of 56 years, suffering from diabetes mellitus with obesity 2 in 8 years, the
examination revealed proteinuria 0.132 g / l, urine acetone (+), fasting glycemia 16.32 mmol / l,
HbA1c - 9.8%. It is treated with glimepiride at a dose of 6 mg per day. Is it necessary to insist
on the need for insulin therapy?
A. No, because there is a risk of further weight gain.
B. No, you need to transfer it to pioglitazone.
C. No, the combination of metformin glyclazide MR should be used
D. Yes, you should insist on insulin therapy.
E. No, only Metformin should be prescribed.

3. Patient 47 years, 9 years old suffering from type 2 diabetes, moderate. BMI-25.4 kg / m2. For
the last year has been using glimepiride 4 mg, metformin 2000 mg, dapagliflozin 10 mg. HbA1c
- 8.9%. Your tactics for further diabetes treatment:
A. Dietary calorie restriction.
B. Translate to combination of glimepiride, metformin and gliclazide MR.
C. Add to current therapy pioglitazone 30 mg.
D. Replace glimepiride and dapagliflozin with medium-duration insulin.
E. Replace glimepiride and dapagliflozin with 0.6 mg liraglutide.

4. Patient 64 years old, suffering from type 2 diabetes mellitus. Receives 15 mg glibenclamide
and 2000 mg metformin daily. After physical activity I felt weakness, dizziness, shortness of
breath, slight pain in the heart area. A / T 70/30 mm Hg Pulse rate - 110 beats / min. According
to the ECG: domed rise of the CT segment, depression of the CT segment in the reciprocal zone,
appearance of the QS tooth. The patient was admitted to the infarct ward. What type of
antipyretic therapy is needed for the patient?
A. Translate into insulin therapy
B. Leave previous therapy
C. Cancel metformin
D. Cancel glibenclamide
E. To appoint emfagliflozin.

5. Patient N. for 23 years was first ill with type 1 diabetes. Which method of diabetes treatment
will not be included in the treatment regimen?
A. Diet.
B. Phytotherapy.
C. Exercise mode.
D. Oral antipyretics
E. Insulin therapy.

6. Woman 57 years old, with obesity 3 tbsp. diagnosed with diabetes two months ago.
endocrinologist patients were recommended a sub-calorie diet and metered exercise. fasting
glycemia level of 9.2 mmol / l. What type of antipyretic drug can a patient recommend?
A. Metformin
B. Glibenclamide
C. Alpha-glucosidase inhibitors
D. NSCTG inhibitors 2
E. CCI agonists 1.

7. The patient, 46 years old, decided to do regular physical exercises for the sake of reducing the
dose of antihypertensive drugs. The examination glycemic profile: 13.1 - 15.4-17.2 - 16.0 mmol
/ l. What do you recommend for the patient?
A. Start intense physical activity.
B. First, achieve diabetes compensation and then start exercising.
C. Start physical activity with low intensity and gradually increase them.
D. Start intense physical activity and gradually reduce them as glycemia normalizes.
E. Exercise every other day.

8. Patient R., 32, had diabetes for the first time, HbA1c - 8.1%. He stubbornly refuses insulin
therapy, hopes to compensate for diabetes with herbal remedies. What herbal raw materials
would you recommend to the patient as monotherapy?
A. Bean husks.
B. Leaves of blueberries.
C. Strawberry grass.
D. Walnut leaves.
E. None of the above, as prescribing monotherapy with herbal remedies is ineffective.

9. Evaluate glucose tolerance test results: Fasting 7.0 mmol / l after 1 hour. - 13,2 mmol / l, after
2 years. - 11.1 mmol / l.
A. Normal test.
B. Clear diabetes.
C. Re-examination is required.
D. Impaired glucose tolerance.
E. Impaired fasting glycemia

10. The absolute contraindications for the treatment of diabetes with sulfonylureas are:
A. Pregnancy.
B. Hemorrhoids.
C. Psoriasis.
D. CHD: postinfarction cardiosclerosis.
E. Prostate adenoma II.

1. Patient A, 58, has been suffering from type 2 diabetes for 17 years. Due to the presence of
proliferative retinopathy, he receives a combination of insulin (Protafan and Actropid, in the
base-bolus mode) at a dose of 78 U / day. Complains about episodes of hypoglycemia,
especially at night. Over the last 6 months, the patient's body weight has increased by 10 kg
(currently 85 kg). Fasting blood sugar on a fasting heart 9,3 mmol / l, glycated hemoglobin -
8,7%. As the insulin dose increases, the blood sugar levels do not change significantly. Make
the right diagnosis
A. Type 2 diabetes, severe, compensated, insulin allergic.
B. Type 2 diabetes, severe, in a state of subcompensation, insulin resistance.
C. Type 2 diabetes is a severe form of decompensation, a Somoji phenomenon.
D. Type 2 diabetes of medium severity, the phenomenon of morning dawn.
E. Type 2 diabetes mellitus, decompensated.

2. In a 59-year-old patient with type 2 diabetes mellitus, 17 years, peripheral polyneuropathy


progresses rapidly. HbA1c - 7.1%. Which mode of therapy should the patient advise?
A. Combine several antipyretic drugs of different groups.
B. Combine glibenclamide with biguanides.
C. Combine oral antipyretics with herbal medicine.
D. The patient's condition is an indication for insulin therapy.
E. Group B and a-lipoic acid drugs should be added to current antipyretic therapy.

3. Patient 72 years old with type 2 diabetes, treated with glibenclamide at a dose of 20 mg per
day. Frequent elevations of blood pressure, persistent cardialgia, especially at night, sweating
are disturbed. What is most likely associated with this condition of the patient?
A. With unstable angina.
B. With late menopausal neurosis.
C. With allergies to glibenclamide.
D. With hypoglycemia.
E. With angina pectoris.

4. In a boy of 16 years, the examination revealed fasting glycemia 6.68 mmol / l. What actions
of the doctor will be appropriate in this case?
A. Undoubtedly diabetes, prescribe treatment with insulin.
B. Diabetes is questionable, to carry out the test for glucose tolerance.
C. Suggest 2 weeks to abstain from simple carbohydrates and then repeat the fasting glucose
again.
D. For stimulation of beta-cells to appoint glibenklamid 2.5 mg every morning.
E. Do nothing, repeat the survey in a year.

5. Patient 44 years old, suffering from type 2 diabetes in combination with obesity 1 tbsp. asks
for advice on optimal diet to reduce body weight. What do you recommend?
A. Low calorie diet 1 time a day.
B. Low calorie diet 2 times a day.
C. Low calorie diet 3-5 times a day.
D. One unloading day per week.
E. Two days off per week.

6. Patient 44 years old, took prednisolone on the scheme for asthmatic status in chronic
bronchitis. After that, a thirst appeared, increased appetite. The diagnosed hyperglycemia is
11.2 mmol / l. How to evaluate the clinical picture?
A. Kidney diabetes
B. Functional impairment of carbohydrate metabolism
C. Type 1 diabetes.
D. Type 2 diabetes
E. Secondary diabetes mellitus

7. A 58 year old woman turned to an endocrinologist. From the anamnesis known to be ill with
type 2 diabetes for 2 years, took metformin 2 g per day. Objectively BMI 28.6 kg / m2,
moderate skin, internal organs without features. Laboratory: fasting glycemia - 10.2 mmol / l,
glycated hemoglobin 8.5%. Following the examination, a type 2 sodium glucose cotransporter
(SGLT2) inhibitor was further administered. What drug did the doctor choose:
A. Sitogliptin (Januvia)
B. Gliclazide (Diabetone)
C. Dagogliflozin (Forksiga)
D. Glimepiride (Amaryl)
E. Pioglitazone (Glutazone).

8. Patient 50 years old, suffering from diabetes for 12 years. Receives glimepiride 4 mg in the
morning. Complains of thirst, weight loss, loss of vision, leg pain. Objectively: height - 168 cm,
body weight - 66 kg Dry skin. HELL - 160/100 mmHg Single point hemorrhages on the fundus.
The pulsation on the arteries of the feet is weakened. Fasting blood glucose - 15 mmol / l, urine
glucose - 2%. HbA1c - 9.6%. What are the tactics of drug therapy?
A. Transfer the patient to insulin therapy
B. Increase the dose of glimepiride
C. Reduce the dose of glimepiride
D. Transfer the patient to biguanides
E. Transfer patient to alpha-glucosidase inhibitors

9. The patient misuses alcohol and suffers from type 2 diabetes. Which of the following is
contraindicated in this case?
A. Pioglitazone
B. Metformin
C. Sitogliptin
D. Liraglutide
E. Dagogliflozin

10. A 62-year-old patient complains of a change in sugar levels and frequent hypoglycemic
conditions, notes an increase in appetite and weight gain. From the anamnesis it is known that a
patient with type 2 diabetes 3 years, uses metformin 2 g / day, a month ago, without consulting a
doctor began to take glimepiride 8 mg / day. HbA1c-6.4%. Choose the right treatment tactic?
A. Lower the dose of metformin
B. Increase metformin dose
C. Cancel metformin
D. Lower the dose of glimepiride
E. Cancel glimepiride.

1. In the patient, 50 years, accidentally during the prophylactic examination revealed fasting
glycemia - 7.2 mmol / l, post-radial - 8.0 mmol / l, glucosuria - 0.5 g / l. HbA1c-6.6%. Height
169 cm, body weight 72 kg. Distinctive initial tactics for treating the patient?
A. Diet therapy
B. Biguanides
C. Derivatives of sulfonylureas.
D. Thiazolidinediones
E. Insulin

2. 64-year-old woman suffering from type 2 diabetes. It is treated with metformin at a dose of
3000 mg per day. Fasting glycemia 7.8 mmol / l, 10.5 mmol / l two hours after meals.
Consulted by an optometrist. Conclusion - "Diabetic proliferative retinopathy of the retina".
What is the further tactic of treating the patient?
A. Do not change therapy.
B. Immediately switch to insulin therapy.
C. Translate into combination with sulfonylurea derivatives.
D. Transfer to combination with NSCTG 2 inhibitors
E. Translate to combination with GPP-1 agonists.

3. In which of the following conditions, in patients with type 2 diabetes, treatment with
antipyretic drugs of a group of NSCTG 2 inhibitors is indicated?
Type 2 diabetes, a severe form of decompensation. Diabetic nephropathy (GFR of 56 mm / h /
1,73m2)
A. Type 2 diabetes, first discovered in a state of subcompensation.
B. Diabetes mellitus type 2, severe form of subcompensation. Proliferative diabetic retinopathy.
C. Diabetes mellitus type 2, moderate in compensation. Coronary heart disease: postinfarction
cardiosclerosis, NK IIa.
D. Diabetes mellitus type 2, moderate in compensation.
E. Scheduled surgery - cholecystectomy.

4. A 55-year-old patient complained of bloating and diarrhea. From the anamnesis it is known
that he has been suffering from type 2 diabetes for about two years. Since diagnosis, he has been
taking glutazone at a dose of 30 mg per day. Two weeks ago, the endocrinologist prescribed an
additional drug from the group of α-glucosidase inhibitors. What drug caused the
gastrointestinal disorders?
A. Sitogliptin (Januvia)
B. Glimepiride (Amaryl)
C. Glipizide
D. Acarbose (Glucobay)
E. Dagogliflozin (Forksiga).
5. In a patient of 58 years, diabetes mellitus 5 years, from the beginning of the disease receives
glyclazide MR 60 mg / day, after trauma developed phlegmon of the right foot. Which treatment
option should be used in this case?
A. Add to gliclazide MR Metformin.
B. Add insulin to gliclazide MR.
C. Add to glyclazide MR acarbose.
D. Replace gliclazide MR with metformin
E. Replace gliclazide MR with insulin therapy.

6. A 69-year-old woman has type 2 diabetes for about 20 years. Uses glimepiride 4 mg daily.
Does not adhere to the diet regime, longer treatment time did not go. Diabetic kidney disease,
stage 4 was diagnosed with careful burden; diabetic proliferative retinopathy. HbA1c - 9.9%.
Perform therapy correction.
A. Dietary regimen, increase daily dose of glimepiride to 6mg.
B. Dietary regimen, add Metformin 1000 mg 2 times a day.
C. Dietary regimen, add pioglitazone 15 mg 2 times a day.
D. Dietary regimen, add short-acting insulin.
E. Dietary regimen, basic bolus insulin therapy.

7. Patient, 56 years old, suffering from type 2 diabetes for 10 years, constantly taking
glibenclamide at 5 mg 3 times a day, recently dramatically lost weight, for three years did not
consult an endocrinologist. Examination on the right leg revealed blackening of the nail, cold
foot, pulsation weakened. Fasting blood sugar is 15.0 mmol / l, urine sugar is 2.0%. Choose
patient treatment tactics:
A. Add to the antihypertensive therapy biguanides.
B. Against the background of this antipyretic therapy, prescribe vasodilators.
C. Consult a surgeon for immediate surgical treatment of the foot.
D. Transfer to insulin therapy, consult a surgeon to determine the need for surgical treatment of
the foot.
E. Against the background of this antipyretic therapy, perform angioplasty treatment of the
lower extremity

8. The absolute contraindications for the treatment of diabetes with sulfonylureas are:
A. Pregnancy.
B. Hemorrhoids.
C. Psoriasis.
D. CHD: postinfarction cardiosclerosis.
E. Prostate adenoma II.

9. Patient 47 years, 9 years old suffering from type 2 diabetes, moderate. BMI-25.4 kg / m2. For
the last year has been using glimepiride 4 mg, metformin 2000 mg, dapagliflozin 10 mg. HbA1c
- 8.9%. Your tactics for further diabetes treatment:
A. Dietary calorie restriction.
B. Translate to combination of glimepiride, metformin and gliclazide MR.
C. Add to current therapy pioglitazone 30 mg.
D. Replace glimepiride and dapagliflozin with medium-duration insulin.
E. Replace glimepiride and dapagliflozin with 0.6 mg liraglutide.

10. In a patient of 56 years, suffering from diabetes mellitus with obesity 2 in 8 years, the
examination revealed proteinuria 0.132 g / l, urine acetone (+), fasting glycemia 16.32 mmol / l,
HbA1c - 9.8%. It is treated with glimepiride at a dose of 6 mg per day. Is it necessary to insist
on the need for insulin therapy?
A. No, because there is a risk of further weight gain.
B. No, you need to transfer it to pioglitazone.
C. No, the combination of metformin glyclazide MR should be used
D. Yes, you should insist on insulin therapy.
E. No, only Metformin should be prescribed.

1. Patient 36 years, type 1 diabetes sick 17 years. Receives insulin 38 IU / day. The course of
the disease is stable. Microalbuminuria up to 180 mg / day has been observed for the last 3
months. Which of these drugs should be prescribed to a patient in complex treatment?
A. Beta blockers
B. Angiotensin converting enzyme inhibitors
C. Diuretics
D. Calcium antagonists
E. Alpha blockers.

2. Patient, 65, has been suffering from type 1 diabetes for 17 years. After the injury, a gangrene
of 1 finger of the left foot developed. Which examination should be performed before resolving
the issue of finger amputation?
A. Doppler imaging of bone index
B. Duplex scan of the limb arteries
C. Thermography
D. Determination of tactile and pain sensitivity
E. Capillaroscopy.

3. Sick 35 years old, suffering from type 1 diabetes for 15 years, abusing alcohol. It is treated
regularly. After drinking, there was a sharp weakness, trembling in the body, sweating.
Objectively: excited, heart rate - 98 / min, blood pressure - 120/70 mm Hg. Art. What causes
the worsening of the patient's condition?
A. With withdrawal syndrome.
B. Hypoglycemia.
C. Hyperglycemia.
D. Sympathoadrenal crisis.
E. Thyrotoxicosis.

4. The patient is suffering from type 1 diabetes and is using a baseline bolus regimen of insulin
therapy using Actropid and Protafan. In the last week he started training in the gym. After the
last session, there was aggression, anxiety, tachycardia and sweating. What is the reason for the
development of this condition and what recommendations can you give to the patient?
A. Hypoglycemia, mild. It is necessary to use 10-20 g. of "fast" carbohydrates.
B. Hypoglycemia, moderate. It is necessary to use 10-20 g. of "fast" carbohydrates.
C. Hypoglycemia, mild. It is necessary to use 10-20 g. of "long" carbohydrates.
D. Physiological response to exercise, the symptoms will go away on their own.
E. Hypoglycemia, moderate. You should use 10-20 g. of "long" and 10-20 g. of carbohydrates.

5. The patient undergoes a routine examination by an endocrinologist. A man has type 1


diabetes for 18 years, the glycemic control is suboptimal. Complains of tachycardia both during
exercise and at rest in the morning and after eating, dizziness after changing the position of the
body. The patient was examined by a cardiologist who recorded QTc 460 ms and abnormal
blood pressure rhythm, namely nocturnal hypertension. Which symptom is a prerequisite for
screening for the development of cardiovascular form of diabetic neuropathy?
A. Orthostatic hypotension.
B. Tachycardia of rest.
C. QTc prolonged.
D. Violation of the daily index of blood pressure at DMAT.
E. All of the above.

6. In a patient of 49 years, who has been suffering from type 2 diabetes for 6 years, there have
been burning feet pain in recent months. Objectively: slimming muscles are noted, skin is dry,
pale, nails are brittle. There are no reflexes on the legs, sensitivity is reduced by the type of
"socks". What disease should you think about?
A. Neuropathy of the tibial nerve
B. Diabetic distal polyneuropathy
C. Neuropathy of the sciatic nerve
D. Lower limb endarteritis
E. Lumbosacral radiculitis.

7. Patient, 60 years old, suffering from type 2 diabetes for 3 years, after drinking alcohol
complains of abdominal pain, muscle, nausea, vomiting. For diabetes, Metformin uses 850 mg 2
times a day. Objectively: symptoms of dehydration, Kussmaul's breathing, blood pressure of
90/50 mm Hg, anuria, temperature 35.9 C, glycemia - 12.9 mmol / l, acetonuria are absent.
blood pH 6.8, lactic acid content 2.7 mmol / l (norm 0.62-1.3 mmol / l). Make the right
diagnosis?
A. Uremic coma.
B. Ketoacidotic coma.
C. Lactic acid coma
D. Brain Coma.
E. Hyperosmolar coma.

8. Patient 56 years old, complaints of abdominal pain, nausea, vomiting, muscle pain.
Objectively, there are clear symptoms of dehydration. Kussmaul's breathing, AO - 90/50 mm.
Hg art., anuria, body t - 35.9 ºC, glycemia - 12.9 mmol / l, no acetonuria, blood PN - 6.8, lactate
content -1.7 mmol / l (N - 0.62 -1.3 mmol / l). Possible diagnosis?
A. Uremic coma
B. Ketoacidotic coma
C. Brain Coma
D. Hyperosmolar coma
E. Lactatacidemic coma.

9. What form of diabetic neuropathy is characterized by acute pain, preesthesia, hyperesthesia


and is completely reversible?
A. Autonomic neuropathy.
B. Acute sensory diabetic neuropathy.
C. Chronic sensory-motor neuropathy.
D. Proximal motor neuropathy.
E. Chronic inflammatory demyelinating polyneuropathy

10. Which of the following diseases characterizes the following changes in the fundus: uneven
caliber of retinal veins, venous loops, soft and hard exudates, pre- and subretinal hemorrhages?
A. Initial diabetic retinopathy.
B. Pre-proliferative diabetic retinopathy.
C. Proliferative diabetic retinopathy.
D. Diabetic maculopathy.
E. Secondary glaucoma.

1. When conducting a screening study on the probable development of orthostatic hypotension in


an asymptomatic patient with type 2 diabetes, it was established that the level of CAT drop is 30
mm Hg, DAT - 11 mm Hg. There are no other causes of orthostatic hypotension in history.
Cardiovascular autonomic neuropathy, severe, was diagnosed. Prescribe therapy for orthostatic
hypotension.
A. Erythropoietin.
B. Midodrin.
C. Desmopressin acetate.
D. Analogues of somatostatin.
E. Non-selective β-blockers.

2. The buyer of 18 years went to the pharmacy with the intention to buy medicine for shivering
and sweating, told that he had diabetes and was taking insulin. Aggressively behaved, refused
the sweet tea offered by the pharmacist, lost consciousness delivered to the emergency room.
What will be the treatment tactics?
A. Introduction of 40 ml of 40% glucose solution in / in
B. Introduction of 50 ml 5% glucose solution in / in
C. Introduction of 20 IU of insulin subcutaneously
D. Introduction of 20 IU of insulin into / in
E. Introduction of 500 ml of 0.9% sodium chloride solution.

3. A girl of 10 years was taken to the hospital unconscious. Suffers type 1 diabetes. Is on basic
bolus insulin therapy. In the morning, going to school, after injecting insulin, I ate less than
usual. After 2 hours, there was anxiety, tremor. She soon fainted. The hospital was diagnosed
with hypoglycemic coma. What is the medical tactic for this baby?
A. Introduce / m 1% solution of adrenaline.
B. Take blood for the sugar content and wait for answers.
C. Adjust the dropwise administration of 10% glucose solution.
D. Take blood for sugar and immediately inject 20.0-40.0 ml of 40% glucose solution into a jet
stream.
E. Inject 20.0 ml of 40% glucose solution into a jet.

4. The patient, 29, was admitted to the cardiology department. Objectively: the skin is dry, the
turgor is reduced, the extremities are cold, cyanotic, Kussmaul's breathing. Pulse 122 beats /
min, arrhythmic, weak filling and tension, blood pressure 60/35 mm RT. On ECG phenomena of
atrial fibrillation. Glycemia 23 mmol / l, pH 7.1. Acetone in urine (++++). Make the right
diagnosis.
A. Ketoacidotic coma, renal variant.
B. Uremic coma.
C. Myocardial infarction.
D. Ketoacidotic coma, colaptoid variant.
E. Hyperosmolar coma.

5. When examined by an ophthalmologist with type 2 diabetes, the proliferative stage of diabetic
retinopathy was diagnosed. What changes in the fundus are most characteristic of this stage?
A. Lens opacities.
B. Microaneurysms, hard and soft exudates.
C. Microaneurysms on the retina ..
D. Processes of neovascularization on the retina and optic disc.
E. Venous loops, subretinal hemorrhages
6. Woman 65 years old with type 2 diabetes, 20 years. She was treated with oral antipyretic
drugs. Only the last year on insulin therapy. HbA1c-11.7%. At present, abdominal pain,
flatulence, and excrement are rare, incontinence. The skin is dry, with a slight
hyperpigmentation. The language is red, "geographical". The abdomen is swollen, sensitive to
palpation. The lower edge of the liver 1 cm protrudes from the edge of the costal arch. Make a
preliminary diagnosis.
A. Chronic cholecystitis.
B. Chronic viral hepatitis.
C. Crohn's disease.
D. Gastric ulcer.
E. Diabetic enteropathy.

7. Patient 37 years old, was admitted to the intensive care unit. The overall condition is very
bad. Sopor. The skin is gray-yellow in color, the turgor is reduced. The pulse is frequent,
intense. HELL - 160/110 mm Hg. Art. Muscle tone increased. Hyperreflexia. The smell of
ammonia in the air. Previous diagnosis?
A. Alcohol coma.
B. Brain Coma.
C. Uremic coma.
D. Hyperglycemic coma.
E. Hypoglycemic coma.

8. Female 22 years, first pregnancy, 36 weeks. Has diabetes type 1 diabetes for 9 years. At
another examination, the woman complains of a sharp deterioration of vision, fog before the
eyes, headache. On examination of the ophthalmologist: on the fundus: hemorrhages of different
ages, degenerative changes, connective tissue strands in the course of retinal vessels. What are
the tactics of childbirth in this case?
A. Cesarean section in urgent order
B. Planned cesarean section
C. Preparation of childbirth paths within 2-3 days, excitation of childbirth with oxytocin
D. Amniotomy followed by oxytocin and enzaprost
E. Treatment of the underlying disease before the development of spontaneous labor.

9. Patient T., 26, is in the intensive care unit in association with a ketoacidotic coma. The
consciousness is blurred, the tone of the eyeballs is reduced, the blood pressure is 90/60 mm Hg,
the heart rate is 130 beats / min. Glycemia 25 mmol / l, pH 7.1. Content of ketone bodies
(++++). What is your initial tactic?
A. Introduction of 500 ml of 5% glucose solution.
B. Introduction of 4% sodium bicarbonate 2.5 ml / kg.
C. The introduction of 10-20 of insulin short-acting in / in jet, and then drip at a rate of 0.1 U /
kg / h to eliminate ketoacidosis.
D. The introduction of 10-20 insulin short-acting insulin / jet, and then drip at the rate of 0.05
U / kg / h to eliminate ketoacidosis.
E. The introduction of 500 ml of 0.9% sodium chloride in / dropwise.

10. Patient, 36, found on the street unconscious. The patient has a card of a patient with
diabetes. The smell of alcohol from your mouth. Skin moist, warm, blood pressure - 145/90 mm
Hg, spasmodic muscle twitching. Breathing superficial, tone of eyeballs preserved, pupils
dilated, hyperreflexia. What is your tactic?
A. Insulin short-acting 10 IU subcutaneously.
B. Insulin short-acting 10 IU intravenously.
C. The introduction of 40-80 ml of 40% glucose solution.
D. Introduction of 500 ml of 5% glucose solution intravenously.
E. Administration of 500 ml of 0.9% sodium chloride intravenously.

1. The patient undergoes routine inspection by an endocrinologist. He recently read an article


regarding varieties of diabetic neuropathy. He became interested in this information because I
recently got sick with diabetes. When you need to carry out a screening on the potential
development of cardiovascular form of Autonomous neuropathy in asymptomatic patients with
diabetes?
A. In type 1 diabetes after 5 years since diagnosis, further 1 once a year / when type 2 diabetes
since diagnosis in the future 1 time in 6 months.
B. With type 1 diabetes after 5 years since diagnosis, further 1 once a year/ when type 2
diabetes since diagnosis in the future 1 time a year.
C. With type 1 diabetes after 5 years since diagnosis in the future 1 every 6 months / when type
2 diabetes since diagnosis in the future 1 time a year.
D. In type 1 diabetes after 3 years since diagnosis, further 1 once a year / when type 2 diabetes
since diagnosis in the future 1 time in 6 months.
E. Exclusively patients with diabetes type 1 and 2 with a history of poor glycemic control.

2. The patient undergoes routine inspection by an endocrinologist. He recently read an article


regarding varieties of diabetic neuropathy. He became interested in this information because I
recently got sick with diabetes type 2 diabetes. The doctor sent the patient to conduct cardio-
vascular tests to exclude cardiovascular form of Autonomous neuropathy.
What is the minimum result of cardio-vascular tests are needed to confirm the diagnosis of
reliable cardiovascular autonomic neuropathy?
A. at least two excited results cardio-vascular tests / two edge, the third is violated.
B. at least four excited results cardio-vascular tests/ or four excited, the fifth edge.
C. at least one impaired result of cardio-vascular tests / two edge.
D. at least one result of disturbed cardio-vascular tests.
E. the Presence of orthostatic hypotension.

3. At the time of examination, the patient is unconscious, the skin is moist, cold, pale.
Kussmaul's breath, the smell of acetone in the air. Positive symptoms of peritoneal irritation.
Blood sugar is 28 mmol / l. What immediate action should you take first?
A. IV glucose infusion with insulin.
B. IV a short-acting insulin infusion + saline sodium chloride.
C. Insulin administration of prolonged action.
D. IV infusion of neohemodesis with glutamic acid.
E. IV infusion of saline sodium chloride.

4. The patient, 29, has been suffering from type 1 diabetes for several years and has been
admitted to the cardiology department. Objectively: cold cyanotic limbs, Kussmaul's breathing
changes superficial, pulse filamentous, blood pressure 60/35 mm Hg. The phenomena of
vascular collapse are clearly pronounced - sleeping veins, especially on the neck. On ECG
phenomena of atrial fibrillation. Glycemia 23 mmol / l, pH 7.1. Acetone in urine (++++). Make
the right diagnosis?
A. Renal form of ketoacidotic coma.
B. Uremic coma.
C. Myocardial infarction.
D. Lactic acid coma.
E. Cardiovascular form of ketoacidotic coma.

5. The patient is admitted to the ward without consciousness. Has been suffering from type 1
diabetes for 5 years. Receives prolonged insulin at a dose of 24 IU in the morning and 18 IU in
the evening. Suddenly lost consciousness. The skin is moist, the muscle tone of the limbs is
elevated. Pulse - 96 / min, blood pressure - 145/85 mm Hg. Art. Heart sounds of normal
volume. Breathing rhythmic. Wet tongue. What kind of therapy is needed first?
A. Insulin short-acting intravenously.
B. Administration of hydrocortisone intravenously.
C. Introduction of 40% glucose solution intravenously.
D. Introduction of a 4% solution of sodium bicarbonate intravenously.
E. Adrenaline administration subcutaneously.

6. Sick 64 years old, suffering from type 2 diabetes, treated with metformin at a dose of 3000 mg
per day. Fasting glycemia-7.8 mmol / l, two hours after meals - 10.5 mmol / l. HbA1c-9.4%.
Creatinine and urea levels are within normal limits. Consulted by an optometrist. Conclusion -
"Diabetic proliferative retinopathy of the retina". What is the further method of treatment of the
patient?
A. Do not change therapy.
B. Increase the daily dose of metformin to 4000 mg per day.
C. Insulin therapy
D. Translate into combination with sulfonylurea derivatives.
E. Translate to combination with thiazolidinediones.

7. Child 7 years. From the age of 2, she was diagnosed with type 1 diabetes, receiving routine
insulin therapy. In the morning after the injection of insulin until 13.00 hours the child ate
nothing. At 3pm, the baby had cold sweat, pallor, tongue and hand tremors. What is the cause
of this condition?
A. Hyperglycemia.
B. Hypoxia.
C. Hypoglycemia.
D. Thyrotoxicosis.
E. Hypercalcemia.

8. Sick 40 years old, suffering from type 1 diabetes for 28 years. During the last year, the insulin
dose decreased by 14 units. In the analysis of urine: protein - 1.7 g / l, sugar - 0.8%, many
erythrocytes, cylinders. In the blood test - iron deficiency anemia. These signs are a
manifestation of:
A. Insulin resistance.
B. Decompensation of diabetes.
C. Chronic diabetic kidney disease
D. Acute pyelonephritis.
E. Chronic insulin overdose syndrome.

9. Patient 64 years old, complains of chills, fever up to 380C, frequent urination, pain in the
lumbar region. Blood sugar - 11.2 mmol / l. In the analysis of urine: specific gravity - 1019,
protein - 0.1 g / l, positive reaction for acetone, L - 30-40 pp / s, Eer - 30-40 pp / s. What is the
most likely complication of diabetes?
A. Diabetic glomerulosclerosis.
B. Acute glomerulonephritis.
C. Acute pyelonephritis.
D. Myocardial infarction.
E. Chronic renal failure.

10. A 25-year-old man who has been suffering from type 1 diabetes for 8 years has developed a
coma. Objectively: the skin is dry, the turgor is reduced, Kussmaul's breathing, blood pressure is
105/60 mm Hg. Art., pulse - 116 / min, the smell of acetone in the air. What kind of coma is
suspected?
A. Hypoglycemic.
B. Hyperosmolar.
C. Ketoacidotic.
D. Lactic acid.
E. Brain.

1. A patient of 36 years complains of weakness, loss of appetite, swelling of the face and legs.
Diabetes mellitus type 1 16 years old, receives insulin 34-46 IU / day. The face is gray, swollen,
swelling on the legs. HELL 210/110 mm Hg. Art., pulse 110 / min, rhythmic. Glycemic
profile: 9-12-10 mmol / l. Ann. urine: Pit. weight 1022, sugar 3%, protein-1,32 g / l, lake.- 3-5
in p / s. What complication did the patient experience?
A. Chronic pyelonephritis
B. Amyloidosis of the kidneys
C. Chronic diabetic kidney disease
D. Nephrotic syndrome
E. Glomerulonephritis

2. Patient 40, suffering from type 1 diabetes for 13 years, consulted for severe tachycardia,
which was not adjusted by beta-blockers, calcium antagonists. The patient also observed
instability of blood pressure (90/60 mm Hg in the morning, 180-200 / 100-110 mm Hg in the
evening), orthostatic fall in blood pressure. Make the right diagnosis.
A. Myocarditis.
B. Diabetic cardiomyopathy.
C. Diabetic autonomic neuropathy of the heart.
D. Symptomatic hypertension.
E. Diabetic macroangiopathy of coronary vessels.

3. Patient 63 years old, suffering from type 1 diabetes. Against the background of acute
disturbance of cerebral circulation developed high hyperglycemia, sharp dehydration,
hyperchloremia, hypernatremia. Ketonemia and acetonuria are absent. Make the correct
diagnosis:
A. Hyperlactacidemic coma.
B. Acute renal failure.
C. Hyperketonemic diabetic coma.
D. Hyperosmolar diabetic coma.
E. Chronic renal failure.

4. A patient of 27 years has been suffering from severe type 1 diabetes for 10 years. After the
operation of the cesarean section, she lost consciousness, Kusmaul's breathing, pale skin,
hypothermia, hypotension, the smell of acetone from the mouth, anuria, blood sugar - 19 mmol /
l, plasma osmolality - 180 mosmol / l. What is the cause of the deterioration of the patient's
condition?
A. An attack of eclampsia.
B. Acute disorders of cerebral circulation.
C. Ketoacidotic coma.
D. Embolism by amniotic fluid.
E. Air embolism.

5. The girl, 12 years old, went to the hospital from school, where during the lessons she became
unconscious, hallucinations began, then convulsions. Has been suffering from type 1 diabetes
for several years. Objectively: the girl is unconscious, no seizures, no meningeal signs. There
are numerous marks on the skin of the hands and thighs after the injection. Blood glucose is 1.6
mmol / l. In what condition is the girl?
A. Hyperosmolar coma.
B. Hyperglycemic coma.
C. Hypoglycemic coma.
D. Lactic acid coma.
E. Adrenal insufficiency.

6. An 8-year-old girl suffering from type 1 diabetes for three years developed a hyperglycemic
coma. What could have caused this complication?
A. Excessive insulin administration.
B. Insufficient administration of insulin.
C. Insufficient food intake.
D. Insufficient fluid intake.
E. Increased physical activity.

7. The girl of 12 years was taken to the hospital unconscious. On examination: sharply depleted,
skin pale gray, dry. Breath deep, loud. The smell of acetone from the mouth. Heart tones are
deaf, rhythmic. Pulse - 90 / min, blood pressure - 90/50 mm Hg. Art. Belly sore. Liver +3.0
cm. Which disease can be assumed in the first place?
A. Liver coma
B. Diabetic ketoacidotic coma.
C. Intestinal infection with neurotoxicosis.
D. The acetonemic state.
E. Adrenal insufficiency.

8. Patient 32 years old with newly diagnosed type 1 diabetes complains of sharp pain in both
legs, their hypersensitivity, especially in the thigh area, touch of clothing causes unbearable pain.
The pain came along with the symptoms of diabetes and significant weight loss. Make a
preliminary diagnosis?
A. Type 1 diabetes, asthenoneurotic syndrome.
B. Type 1 diabetes, chronic sensory-motor neuropathy.
C. Type 1 diabetes, acute sensory neuropathy.
D. Diabetes mellitus type 1, compression mononeuropathy of the lower extremities.
E. Type 1 diabetes, proximal motor neuropathy.

9. The patient with consciousness is transported by ambulance to the intensive care unit. Noisy
breathing, Kussmaul type, acetone odor in the mouth, dry skin, turgor lowered, facial features
sharpened, periosteal reflexes absent, eyeball tone reduced. Glycemia 25.3 mmol / l, acetonuria
(++++). What is the rate at which glycemia is reduced?
A. It is necessary to reach the level of 8-12 mmol in the first hour
B. The level of 13-14 mmol must be reached within the first hour
C. Glycemia should be reduced by 4-5 mmol / h.
D. Glycemia should be reduced by 6-7 mmol / h.
E. A level of ≤ 7.0 mmol / l must be reached during the first day.

10. Patient, 59 years old, suffering from diabetes for 11 years, daily dose of insulin is 62 IU +
metformin 2000 mg, the traditional mode. There are frequent hypoglycemic and ketoacidotic
conditions in history. HbA1c 12.1%. At ophthalmoscopy: on the fundus, numerous
hemorrhages, neovascularization of the optic disc, fibroids. Assign appropriate therapy.
A. Basis-bolus regimen of insulin therapy. Laser coagulation of the retina.
B. To therapy add analogues of GPP-1. Laser coagulation of the retina.
C. Transfer patient to intermediate-acting insulin. Laser coagulation of the retina.
D. Basis-bolus regimen of insulin therapy. Phacoemulsification.
E. None of the above.

1. Patient 64 years old, complains of chills, fever up to 380C, frequent urination, pain in the
lumbar region. Blood sugar - 11.2 mmol / l. In the analysis of urine: specific gravity - 1019,
protein - 0.1 g / l, positive reaction for acetone, L - 30-40 pp / s, Eer - 30-40 pp / s. What is the
most likely complication of diabetes?
A. Diabetic glomerulosclerosis.
B. Acute glomerulonephritis.
C. Acute pyelonephritis.
D. Myocardial infarction.
E. Chronic renal failure.

2. Patient 63 years old, suffering from type 1 diabetes. Against the background of acute
disturbance of cerebral circulation developed high hyperglycemia, sharp dehydration,
hyperchloremia, hypernatremia. Ketonemia and acetonuria are absent. Make the correct
diagnosis:
A. Hyperlactacidemic coma.
B. Acute renal failure.
C. Hyperketonemic diabetic coma.
D. Hyperosmolar diabetic coma.
E. Chronic renal failure.

3. A 25-year-old man who has been suffering from type 1 diabetes for 8 years has developed a
coma. Objectively: the skin is dry, the turgor is reduced, Kussmaul's breathing, blood pressure is
105/60 mm Hg. Art., pulse - 116 / min, the smell of acetone in the air. What kind of coma is
suspected?
A. Hypoglycemic.
B. Hyperosmolar.
C. Ketoacidotic.
D. Lactic acid.
E. Brain.

4. A girl of 10 years was taken to the hospital unconscious. Suffers type 1 diabetes. Is on basic
bolus insulin therapy. In the morning, going to school, after injecting insulin, I ate less than
usual. After 2 hours, there was anxiety, tremor. She soon fainted. The hospital was diagnosed
with hypoglycemic coma. What is the medical tactic for this baby?
A. Introduce / m 1% solution of adrenaline.
B. Take blood for the sugar content and wait for answers.
C. Adjust the dropwise administration of 10% glucose solution.
D. Take blood for sugar and immediately inject 20.0-40.0 ml of 40% glucose solution into a jet
stream.
E. Inject 20.0 ml of 40% glucose solution into a jet.

5. When examined by an ophthalmologist with type 2 diabetes, the proliferative stage of diabetic
retinopathy was diagnosed. What changes in the fundus are most characteristic of this stage?
A. Lens opacities.
B. Microaneurysms, hard and soft exudates.
C. Microaneurysms on the retina ..
D. Processes of neovascularization on the retina and optic disc.
E. Venous loops, subretinal hemorrhages.

6. In a patient with type 1 diabetes, the examination revealed a level of HbA1c 10.2%, UACR
4.3 mg / mmol. Identify the main areas of therapy correction:
A. Lifestyle modification, self-control training, insulin therapy correction (baseline-bolus
regimen), kidney transplantation.
B. Lifestyle modification, self-control training, insulin therapy correction (base-bolus regimen),
peritoneal dialysis.
C. Lifestyle modification, self-control training, insulin therapy correction (base-bolus regimen),
ACE inhibitors.
D. Lifestyle modification, self-control training, insulin therapy correction (traditional mode),
angiotensin-II receptor inhibitors.
E. Lifestyle modification, self-control training, insulin therapy correction (basic-bolus
regimen), limit protein and salt intake.

7. The patient has been suffering from diabetes for 20 years. In the last 2 years there have been
hypoglycemia, the need for insulin has decreased by 16 IU per day. In the general urine
analysis, protein is 1.2 g / l, single leukocytes and altered red blood cells. These signs can be
manifestation of:
A. Concomitant pyelonephritis
B. Concomitant glomerulonephritis
C. Development of amyloidosis
D. Development of glomerulosclerosis
E. Unsatisfactory compensation for the disease

8. Sick 64 years old, suffering from type 2 diabetes, treated with metformin at a dose of 3000
mg per day. Fasting glycemia-7.8 mmol / l, two hours after meals - 10.5 mmol / l. HbA1c-
9.4%. Creatinine and urea levels are within normal limits. Consulted by an optometrist.
Conclusion - "Diabetic proliferative retinopathy of the retina". What is the further method of
treatment of the patient?
A. Do not change therapy.
B. Increase the daily dose of metformin to 4000 mg per day.
C. Insulin therapy
D. Translate into combination with sulfonylurea derivatives.
E. Translate to combination with thiazolidinediones.

9. The girl of 12 years was taken to the hospital unconscious. On examination: sharply depleted,
skin pale gray, dry. Breath deep, loud. The smell of acetone from the mouth. Heart tones are
deaf, rhythmic. Pulse - 90 / min, blood pressure - 90/50 mm Hg. Art. Belly sore. Liver +3.0
cm. Which disease can be assumed in the first place?
A. Liver coma
B. Diabetic ketoacidotic coma.
C. Intestinal infection with neurotoxicosis.
D. The acetonemic state.
E. Adrenal insufficiency.

10. Which of the following diseases characterizes the following changes in the fundus: uneven
caliber of retinal veins, venous loops, soft and hard exudates, pre- and subretinal hemorrhages?
A. Initial diabetic retinopathy.
B. Pre-proliferative diabetic retinopathy.
C. Proliferative diabetic retinopathy.
D. Diabetic maculopathy.
E. Secondary glaucoma.

Female 35 years of age, taking daily levothyroxine 50 mcg / day due to hypothyroidism on the
basis of chronic autoimmune thyroiditis. In the control examination: no complaints, objectively -
without pathological abnormalities, MT = 58 kg; TSH = 7.2 mIU / l, dT4 = 1.26 ng / dL, dT3 =
3.08 ng / dL. Your suggestion for therapeutic tactics at the moment:
A. No correction is required.
B. Reduce the dose to 25 mcg / day.
C. Increase the dose to 75 mcg / day.
D. Leave dose, add selenium preparation.
E. Correction after determination of lipidogram.

2. Patient V., 3 years old, was admitted to hospital for congenital hypothyroidism. Which of the
following drugs should be prescribed?
A. Levothyroxine
B. Somatotropic
C. Potassium iodide
D. Thiamazole
E. Prednisone.

3. Patient V., 3 years old, was admitted to the hospital for delayed physical and mental
development. Objectively: height 52 cm, body weight 13 kg. Expressed skin dryness, hair thin
and brittle. The skull is large, the crown is not closed. The tongue protrudes from the oral
cavity. Heart rate 56 beats / min Heart tones are muted. The thyroid gland is not enlarged.
Mental and physical development lags behind the passport. Make a preliminary diagnosis.
A. Subacute thyroiditis
B. Congenital hypothyroidism
C. Autoimmune thyroidin, hypothyroid phase.
D. Endemic goiter
E. Sporadic goiter.

From anamnesis of the patient M., 46 years old, it is known about chronic thyroiditis. Recently,
she has been worried about frostbite, drowsiness, drowsiness, constipation, memory loss.
Objectively: the skin is pale, dry, cold and swollen, the loss of eyebrows and hair on the temples.
HELL 100/70 mm Hg. bradycardia. What is the most likely diagnosis?
A. Endemic goiter, euthyroid condition
B. Chronic fibrous thyroiditis
C. Autoimmune thyroiditis, euthyroid condition
D. Secondary hypothyroidism
E. Autoimmune thyroiditis, hypothyroidism.

5. The patient is diagnosed with acute purulent thyroiditis at the stage of abscission. Which of
the following treatments is appropriate?
A. Surgical treatment
B. Thiamazole
C. Radioiodine therapy
D. Prednisone
E. Potassium iodide.
In the complex treatment of thyrotoxic crisis, the following drugs are prescribed, except:
A. Anaprilin
B. The Lugol solution
C. Adrenaline
D. Cordyamine
E. 40% glucose solution.

7. Patient L., 42 years. Operated for the diffuse toxic goiter of the third century. A month after
surgery, the patient had convulsive contractions of the muscles of the extremities. Objectively:
Pulse 76 beats / min., Blood pressure 136/80, symptoms of Chvostek, Trusso positive. What is
the most likely diagnosis?
A. Postoperative hypothyroidism
C. Recurrence of goiter
C. Vegetative-vascular dystonia
D. Postoperative hypoparathyroidism
E. None of the above.

8. The boy is 10 days, sluggish, motionless, constantly drowsy, suffering from constipation. He
rarely screams, his voice rough. Objectively: the language is large; the skin is dry, yellowish in
color, cold. Bradycardia, hypotension. The thyroid gland is not palpable. Karyotype 46XY.
Establish a probable diagnosis.
A. Down disease
B. Congenital hypothyroidism *
C. Hemolytic disease of the newborn
D. Iron deficiency anemia
E. Endemic goiter.

A woman has 35 years of complaints of frostbite, adynamia, inhibition, drowsiness. Objectively:


the skin is pale, dry, cold and swollen, the loss of eyebrows and hair on the temples. HELL
100/70 mm Hg. bradycardia, fluid in the pericardium cavity. On ECG bradycardia, the low
voltage of the QRS complexes and the teeth of P and T. The thyroid gland is not enlarged.
Which diagnosis is most likely?
A. Autoimmune thyroiditis, euthyroid state
B. Endemic goiter
C. Subacute thyroiditis
D. Hypothyroidism
E. Cardiovascular failure.

10. Patient O., 42, complains of emotional lability, tachycardia during agitation, hyperhidrosis
of the palms. Thyroid enlarged to IV century, moderately compacted and heterogeneous, lobe
surface. Ultrasound: echogenicity of the gland is reduced, the structure is heterogeneous. TSH -
2.1 mIU / l, free T4 - 13.6 pmol / l. What is the most likely diagnosis?
A. Primary hypothyroidism
B. Secondary hypothyroidism
C. Primary hyperthyroidism
D. Secondary hyperthyroidism
E. Autoimmune thyroiditis, euthyroid state.

Patient 38 years old complained of weight gain, frostbite, dry skin, drowsiness, difficulty
concentrating. Objectively: height 165 cm, weight 78 kg, female phenotype, t = 35,8C, HR = 58
/ min, blood pressure = 105/60 mm Hg Other internal organs unchanged. Thyroid 1 st.,
Diffusely compacted. The galactorrhea of the 1st century Laboratory study found an increase in
TSH and prolactin levels, a decrease in T4. What is the probable cause of the galactorrhea?
A. Primary hypothyroidism.
B. Secondary hypothyroidism.
C. Tertiary hypothyroidism.
D.Hippopituitarism.
E. Prolactinoma.

2 Patient hospitalized after influenza. From anamnesis (according to relatives) it is known about
hypothyroidism, the last 6 months have not been treated. On examination, consciousness is
absent, tendon reflexes are lowered, body temperature is 35.50 C, breathing 10 per 1 min,
superficial. HELL 80/50 mm Hg. Art. Pulse 48 beats / min, fluid in the cavity of the
pericardium. ECG low voltage QRS complexes and teeth P and T. What is the most likely
diagnosis?
A. Acute disorders of cerebral circulation
B. Myocardial infarction.
C. Acute cardiovascular failure
D. Hypothyroid coma
E. Chronic adrenal insufficiency.

In a patient of 26 years with postoperative hypothyroidism who received L - thyroxine 100 μg


twice a day, tachycardia, sweating, irritability, and sleep disorders appeared. Determine the
tactics of further treatment.
A. Reduce the dose of thyroxine
B. Add thiamazole
C. Assign beta blockers
D. Increase the dose of thyroxine
E. Prescribe sedatives.

4. In the patient K., 46 years, six months after subtotal resection of the thyroid gland appeared
general weakness, apathy, drowsiness, hair loss, dry skin, constipation. Pulse 60 beats / min.,
Blood pressure 130/80 mm Hg. Art. What is the most likely diagnosis?
A. Iodine deficiency state
B. Subacute thyroiditis, hypothyroid stage
C. Autoimmune thyroiditis
D. Postoperative hypothyroidism
E. Hyperthyroidism.

Patient S., 25, has no complaints. The examination revealed an increase in the thyroid gland. He
resides in the Skole district. Objectively: the thyroid gland enlarged to the 2nd century., Of
uniform consistency, elastic, not painful. Total T4 - 80 mmol / l, T3 - 2.3 mmol / l, TSH - 3.6
mmO / l.
A. Diffuse toxic goiter
B. Subacute thyroiditis
C. Autoimmune thyroiditis, euthyroid state
D. Endemic goiter, euthyroid state
E. Endemic goiter, hypothyroidism.

6. Patient K., age 52, complains of weight gain, weakness, discomfort in the neck. Objectively:
dry skin, moderate swelling of the face and extremities. Pulse rate of 60 beats / min. The
thyroid gland enlarged to the second century, is heterogeneous, not painful. Antibodies against
thyroid peroxidase 300 IU / ml. Hormone levels: total T4 - 40 nmol / l, T3 - 0.68 nmol / l, TSH -
12.4 mIU / l. What is your diagnosis?
A. Endemic goiter
B. Subacute thyroiditis
C. Autoimmune thyroiditis, hypothyroidism
D. Autoimmune thyroiditis, euthyroid state
E. Fibrous thyroiditis.

Patient C., 42 years old, found an increase in thyroid gland of II., Painful on palpation, pain
radiates into the lower jaw, body temperature 37-38 ° C, a week ago suffered angina. Most
likely that the patient:
A. Diffuse toxic goiter
B. Toxic thyroid adenoma
C. Subacute thyroiditis
D. Autoimmune thyroiditis
E. Riedel's goiter

Patient S., 18, has no complaints. The examination revealed an increase in the thyroid gland. He
resides in the Skole district. Objectively: thyroid gland enlarged to the 2nd century., Of uniform
consistency, elastic, not painful, sensitive to palpation. Total T4 - 80 mmol / l, TSH - 1,8 mmO /
l. Assign therapy.
A. L-thyroxine
B. Thiamazole
C. Potassium iodide
D. Combination of L-thyroxine and potassium iodide
E. None of the following.

9. The main reason for thyroid enlargement in Hashimoto thyroiditis is:


A. Thyrocyte hyperplasia.
B. Formation of fibrous tissue.
C. TSH stimulation.
D. Lymphoid infiltration of the gland.
E. Influence of antimicrosomal antibodies.

10. An “accumulation defect” on a thyroid scan may be a sign of:


A. Subacute thyroiditis.
B. Malignant tumor.
C. Riddle's thyroiditis.
D. Acute purulent thyroiditis.
E. Any underlying pathology}

A resident of the mountain district of Ivano-Frankivsk region, 23 years old, complains of


miscarriage (in history - 2 miscarriages). Palpatory thyroid gland I., Soft, homogeneous; on
ultrasound - without pathological changes. There are no clinical signs of thyroid dysfunction.
Laboratory: TSH = 6.2 mIU / L, BT4 = 1.46 ng / dL, anti-TPO = 20 IU / ml. What is the
probable cause of this situation?
A. Hashimoto thyroiditis.
B. Iodine deficiency subclinical hypothyroidism.
C. Thyrotoxicosis.
D. Manifest hypothyroidism.
E. Stressful condition.

2. Clinical manifestations of Riddle thyroiditis may be:


A. Bradycardia.
B. Tachycardia.
C. Leukocytosis.
D. Weight loss.
E. Swallowing disorders.

3. Woman 30 years old, complains of dry skin, memory loss. On examination: tongue enlarged,
reflexes slowed down, swelling of the face, dry, cold to the touch skin. These symptoms are
characteristic of:
A. Euthyroid goiter
B. Manifest hypothyroidism
C. Latent hypothyroidism
D. Iodine deficiency state
E. Autoimmune thyroiditis.

Patient V., 28 years, within 3 months notes swelling of the anterior surface of the neck. The
thyroid gland enlarged to the IV century., Moderately compacted, heterogeneous, not painful.
Heart rate 70 beats / min. AO 120/60. Laboratory revealed high levels of antibodies to
thyroglobulin and thyroid peroxidase. Thyroid ultrasound: echogenicity reduced, structure
heterogeneous. What is your diagnosis?
A. Euthyroid goiter
B. Primary hypothyroidism
C. Subacute thyroiditis
D. Autoimmune thyroiditis
E. None of the following.

Riedel thyroiditis should first be differentiated from:


{= Thyroid cancer.
~ Hashimoto thyroiditis.
~ Diffuse toxic goiter
~ Subacute thyroiditis de Kerven.
~ Toxic thyroid adenoma}

6. Patient S., 52, complains of weight gain, weakness, constipation, and memory impairment.
Objectively: dry skin, moderate swelling of the face and extremities. Pulse 60 beats / min.,
Blood pressure 110/60 mm Hg. Art. Ultrasound: thyroid gland of reduced size, echogenicity is
reduced, structure is uneven. Antibodies to thyroglobulin - 150 IU / ml, thyroid peroxidase - 400
IU / ml. Most likely that the patient:
A. Euthyroid goiter
B. Autoimmune thyroiditis, euthyroidism
C. Subacute thyroiditis
D. Iodine deficiency hypothyroidism
E. Autoimmune thyroiditis, hypothyroidism.

Assess the results of the laboratory examination of the patient: total T4 - 40 nmol / l, T3 - 0.68
nmol / l, TSH - 12.4 mmol / l:
A. Secondary hypothyroidism
B. Primary hypothyroidism
C. Euthyroid state
D. Primary hyperthyroidism
E. Secondary hyperthyroidism.

8. Patient K., 47 years old, resident of Prykarpattya, appealed about the appearance of a circular
formation on the neck 3 months ago. Worked in the Chernobyl zone in the period 1987-1988.
Objectively: the left part of the thyroid gland is not enlarged, in the lower pole of the right part is
nodular formation of an oval form of size 3.2x2.5 cm, dense consistency, with uneven hilly
surface, not painful , moving. Ultrasound: Almost all the right part is occupied by a formation
with reduced inhomogeneous echogenicity and fuzzy contour, containing two small calcinates.
What is your previous diagnosis?
A. Thyroid cancer
B. Autoimmune thyroiditis
C. Subacute thyroiditis
D. Endemic goiter
E. Sporadic goiter.

Patient K, addressed about the appearance of a circular formation on the neck 3 months ago on
the right. Well-being is good. Objectively: The left lobe of the thyroid gland is not enlarged; in
the projection of the right lobe, an oval-shaped 3,2 × 2,5 cm node is thick, with a dense
consistency, limited mobility, not painful. Suggest optimal tactics.
A. Surgical treatment
B. Potassium iodide
C. Radioiodine therapy
D. Active surveillance
E. L – thyroxine}

10. In the examination of a woman of 50 years, nodular goiter of the II century was found. T3
and T4 levels are slightly elevated in the blood. Ultrasound: the lower pole of the left lobe is 2.5
× 2.0 cm in height, with reduced echogenicity and a fuzzy outline, and contains two small
calcines. What examination method is indicated for the specified diagnosis?
A. Transdermal thin needle biopsy
B. Determination of auto antibody content
C. Determination of TSH content
D. Determination of thyroglobulin content
E. Scintigraphy

In a woman, 35 years old, palpation of the thyroid gland in the left lobe revealed a formation of
Ø 1.5 cm, moderately dense consistency, indistinctly delineated, sensitive to palpation. In
ultrasound examination, it has reduced echogenicity, heterogeneous structure, with fuzzy
contours. Which examination is most appropriate for clarifying the diagnosis?
A. Determination of TSH level.
B. Determination of TSH and BT4 levels.
C. Fine needle aspiration biopsy.
D. Determination of thyroglobulin content.
E. Other (specify).

2. The most common cause of primary hypothyroidism in adults is:


A. Hashimoto thyroiditis.
B. Subacute thyroiditis de Kerven.
C. Iodine deficiency in the environment.
D. Treatment with lithium carbonate.
E. Thyroid tumors.

In the presence of enlargement of the thyroid gland III degree and overt hypothyroidism for
the following examinations are needed to establish a diagnosis:
A. Determination of TSH level in the blood.
B. Determination of T4 level in the blood.
C. Thyroid scan with I131.
D. Ultrasound examination of the thyroid gland.
E. Determination of the level of autoantibodies to thyroperoxidase.

4. In the treatment of hypochromic anemia for hypothyroidism, the most essential is:
A. Purpose of iron preparations.
B. Purpose of cyancobalamin.
C. Prescription of thyroid replacement therapy.
D. Small doses of androgens.
E. Correction of autoimmune shifts.

5. Choose an appointment that is not used in the treatment of hypothyroid coma:


A. Droperidol 0.25-0.5 ml intramuscularly every 12 hours.
B. L-thyroxine or triiodothyronine through a gastric probe of 50 μg every 6 hours.
C. Passive warming of the patient.
D. Intravenous infusion of 100 mg prednisolone in 150 ml of 0.9% sodium chloride solution.
E. Intravenous infusion of 125 mg of hydrocortisone in 250 mg of 5% glucose solution.
In a patient with Hashimoto's autoimmune thyroiditis, the examination revealed a normal level of
T4 in the blood, a level of TSH - at the upper limit of normal. What additional examinations
need to be done to evaluate the functional status of the thyroid gland?
A. Triiodothyronine test.
B. Accumulation of I131 by the thyroid gland.
C. Definition of the main exchange.
D. Tyroliberin test.
E. Test with suppression of TSH.

7. Patient K., 26 years. For 2-3 years complains of headaches, frostbite, apathy, decreased
appetite, constipation, impotence. Objectively: the skin is pale, cold, dry. Slight hair on the face
and torso. It is unique. The thyroid gland does not palpate. Pulse 60 per minute, blood pressure
80/50 mm Hg. Art. Heart tones are weakened. Additional data: 17XC - 18 µmol / l, 17XCS -
11 µmol / l. Radiography of the skull: Turkish saddle 2.2x1.6 cm, its contours are clear. TSH -
30 ml / ml. Previous diagnosis?
A. Secondary hypothyroidism.
B. Primary hypocorticism.
C. Primary hypogonadism.
D. Hypopituitarism.
E. Primary hypothyroidism, symptomatic hypogonadism.

The 56-year-old patient turned to the clinic complaining of growing weakness, drowsiness,
weight gain, and swelling on her feet during the year. The examination draws attention to the
pasty face, legs and feet, dry skin. The borders of the heart are shifted to the left and right by 2
cm, the heart tones are deaf. HR 60 beats and 1 min, the rhythm is correct. HELL 150/90 mm
Hg. Art. Blood test is normal. ECG: sinus rhythm, bradycardia, decreased voltage of all teeth.
Previous diagnosis?
A. CHD, atherosclerotic cardiosclerosis, HNA PA.
B. Dilated cardiomyopathy, CNC II A.
C. Obesity. Dysmetabolic myocardiopathy, CNC II A.
D. Hypothyroidism. Myocardiodystrophy of CNC II A.
E. None of these diagnoses.

9. Goiter in autoimmune thyroiditis has the following characteristic:


A. Immobile, soldered to soft tissues.
B. Seal unevenly.
C. The surface of the tubercle.
D. It is more often diffuse.
E. Sharply painful on palpation.

10. What are the characteristics that distinguish exacerbation of Hashim's autoimmune
thyroiditis from de Kerven's subacute thyroiditis:
A. The presence of "goiter" in the anamnesis.
B. Temperature rise.
C. Symmetric enlargement of the thyroid gland.
D. Increase in ESR.
E. Absence of signs of thyrotoxicosis.

Patient R., 44 years. Complains of weakness, periodic pain in the heart, palpitations, drowsiness,
irritability. Has been ill for 3 years. It was treated without effect in a cardiologist and a
neurologist. Objectively: height 166 cm, body weight 70 kg. Normal skin, moist. The thyroid
gland is diffusely enlarged to the 2nd degree, elastic-elastic consistency, with a smooth surface,
movable. Pulse 96 per minute, blood pressure 140/70 mm Hg. Art. Heart sounds loud, systolic
murmur over the top. Additional data: general blood and urine analysis without pathology.
Thyroid hormone levels are moderately elevated. Ultrasound: evenly reduced echogenicity.
Make a preliminary diagnosis:
A. Diffuse toxic goiter II. Art., Manifest thyrotoxicosis.
B. Autoimmune thyroiditis.
C. Thyroid cancer.
D. Riedel's goiter.
E. Diffuse euthyroid goiter grade II

Patient N., 68 years. Complains of irritability, palpitations, insomnia, weight loss of 8 kg during
the month. Objectively: the thyroid gland enlarged to the second century, not painful;
exophthalmos, glitter, eye tremor. Results of laboratory tests. Total T4 - 200 nmol / l, T3 - 4
nmol / l, TSH - 0.1 mmO / l. What is the previous diagnosis?
A. Diffuse toxic goiter
B. Neurasthenia
C. Autoimmune thyroiditis
D. Hypothyroidism
E. Hypoparathyroidism

The patient is diagnosed with thyrotoxicosis. Pulse 116 for 1 min, atrial fibrillation. Pulse
deficit is 30 for 1 min. Which of the following antiarrhythmic drugs is contraindicated?
A. Amiodarone
B. Lidocaine
C. Propranolol
D. Verapamil
E. Aymalin

Patient M., 33 years. Complains of irritability, palpitations, sweating, general weakness,


shortness of breath. Weight lost 7 kg. Objectively: height 168 cm, body weight 58 kg. The skin
is moist. The thyroid gland is enlarged by all departments. There is a glint, a slight bilateral
exophthalmos, tremor of the fingers. Pulse 120 for 1 min., BP 150/60 mm Hg. Art. What is the
previous diagnosis?
A. Diffuse toxic goiter
B. Toxic adenoma of the thyroid gland
C. Chronic fibrous thyroiditis
D. Autoimmune thyroiditis, euthyroid state
E. Neurasthenia

In what disease of the thyroid gland do the grounds for percutaneous thin-needle biopsy most
often arise?
A. Nodular nontoxic goiter.
B. Diffuse toxic goiter.
C. Thyrotoxic adenoma.
D. Autoimmune thyroiditis.
E. Subacute thyroiditis

Patient S., 45 years. Has been suffering from diffuse toxic goiter for 5 years. Which of the
prescribed drugs has anti-thyroid effects?
A. Mercazolyl
B. Prednisolone
C. B-Vitamin Complex
D. Phytosedative complex
E. Anaprilin
Patient S., 12 years old suffering from diffuse toxic goiter. A thyrotoxic crisis has developed due
to an appendectomy. To derive from this critical condition used:
A. Thiamazole
B. Captopril
C. Glucagon
D. Levothyroxine
E. Nifedipine.

Evaluate the results of thyroid hormone examination: total T4 - 40 nmol / l, T3 - 1.68 nmol / l,
TSH - 12.4 mIU / l.
A. Manifest hypothyroidism
B. Latent hypothyroidism
C. Euthyroidism
D. Latent hyperthyroidism
E. Manifest hyperthyroidism.

Which of the following may cause hypocalcaemia


A. Furosemide
B. Tomatoron
C. Digoxin
D. Calcitriol
E. Vitamin D3

At admission to an endocrinologist ill K., 55, complains of limb numbness, tingling and muscle
twitching, irritability and frequent depression. What disease can be suspected in a patient?
A. Epilepsy Hypoparathyroidism
B. Hypothyroidism
C. Spasmophilia
D. Epilepsy
E. Hyperparathyroidism

Woman 25 years old, emotionally labile, on examination - tremor of fingers, skin moist, warm,
apical impulse increased, atrial fibrillation. Temperature 37.20 C, heart rate 96 beats / min.,
Blood pressure 170/70 mm Hg. Art. Thyroid gland II. The patient's pathogenetic mechanisms
are at the heart of impaired cardiovascular function, except:
A. The effect of thyroid stimulating antibodies
B. Increasing the sensitivity of receptors to catecholamines
C. Increasing catabolism of protein substrates
D. Changes in the tone of the autonomic nervous system
E. Influence of excess thyroid hormones on the myocardium

Patient with diffuse toxic goiter, receiving drug therapy. Over the last three days the condition
has worsened: sore throat, stomatitis, hyperthermia. Which of the following drugs could cause
complications?
A. Mercazolyl
B. Cocarboxylase
C. Verapamil
D. Disputes
E. Corvalol

The patient was diagnosed with diffuse toxic goiter of the third century. of moderate severity in
the stage of decompensation. What skin changes are characteristic of this condition?
A. Skin moisture
B. Dry skin
C. Depigmentation
D. Hemorrhagic rash
E. Papular rash

The patient complains of irritability, palpitations, weakness, tremor of extremities, disturbance of


falling asleep. Pulse 116 beats / min., Blood pressure 160/60 ml RT. Art. Weight loss up to
20%. The total T4 level is 180 nmol / l, T3 is 4 nmol / l. Thyroid enlarged to the 3rd century. at
the expense of all departments, elastic, not painful. Your previous diagnosis.
A. Diffuse toxic goiter, medium weight.
B. Diffuse toxic goiter, severe form
C. Chronic fibrous thyroiditis
D. Diffuse toxic goiter, light in shape
E. Toxic adenoma of the thyroid gland

Which of the following hormonal parameters is considered the most sensitive test for the
evaluation of functional disorders of the thyroid gland?
A. TSH
B. Total T4
C. Free T4
D. Total or free T3
E. Definition of T4 and T3 simultaneously

In the patient M., 35 years old, there are palpitations, shortness of breath, increased sweating,
weight loss, hypertension, weight loss of up to 30%. Pulse 130 beats / min., Blood pressure
170/70 mm Hg. Art. The level of total T4 is 220 nmol / l, T3 is 7 nmol / l. The thyroid gland is
enlarged to the second century. It is not painful, elastic. What disease can you think of?
A. Diffuse toxic goiter, severe
B. Diffuse toxic goiter, lightweight
C. Diffuse toxic goiter, medium to severe
D. Autoimmune thyroiditis
E. Toxic thyroid adenoma

Woman 25 years old, emotionally labile, on examination - tremor of fingers, skin moist, warm,
apical impulse increased, atrial fibrillation. Temperature 37.20 C, heart rate 96 beats / min.,
Blood pressure 170/70 mm Hg. Art. Thyroid gland III. The patient's pathogenetic mechanisms
are at the heart of impaired cardiovascular function, except:
A. The effect of thyroid stimulating antibodies
B. Increasing the sensitivity of receptors to catecholamines
C. Increasing catabolism of protein substrates
D. Changes in the tone of the autonomic nervous system
E. Influence of excess thyroid hormones on the myocardium

In a patient of 28 years after surgical treatment of diffuse toxic goiter seizures of chewing
muscles, hands with predominance of flexor tone are observed. Seizures are painful,
symmetrical. On examination positive symptoms of Hvostek, Trusso. What is the likely
diagnosis?
A. Hypoparathyroidism
B. Thyreotoxic crisis
C. Epilepsy
D. Ruler
E. Spasmophilia
In a woman of 25 years in the postpartum period nausea, vomiting, psychomotor agitation,
hyperthermia up to 39,20 C, tachyarrhythmia, increase in blood pressure up to 180/80 mm Hg
appeared. Art. From the anamnesis it is known that the patient was treated for a long time with
mercazolyl. What is the most likely diagnosis?
A. Thyreotoxic crisis
B. Neurosthenia
C. Hypertensive crisis
D. Acute inflammatory process
E. Eclampsia

What are the main ECG criteria for hypercalcemia?


A. QT interval shortening, QRS complex expansion, T inversion
B. Shortening of QT interval and QRS complex, inversion of T tooth
C. Expansion of QT and QRS complex, T tooth inversion
D. Expansion of QT, reduction of QRS complex, tooth T on the line
E. Shortening of the QT interval, expansion of the QRS complex, high T teeth in the chest
abductions

The patient is operated on about a diffuse toxic goiter. Before the operation, the heart rate is 96
beats / minute, blood pressure 125/70 mm Hg. Art. On the second day the condition worsened
sharply: the patient was agitated, the consciousness was darkened, palpitations, extreme
sweating, vomiting. Pulse 166 beats / minute, atrial fibrillation. HELL 80/40 mm Hg
Temperature 390 C. What is the most likely diagnosis?
A. Thyrotoxic crisis
B. Acute respiratory failure
C. Post-operative sepsis
D. Hypoparathyroidism
E. Acute cardiovascular failure

Patient R., 32, received complaints of weight loss, severe weakness, palpitations, limb tremor,
sweating. Objectively: the thyroid gland enlarged to the second century, at the expense of all
departments, elastic, not painful, blood pressure 180/60 mm Hg. Art., pulse 120 beats / min.
Bilateral ophthalmopathy of the III-IV centuries. Preliminary diagnosis:
A. Diffuse toxic goiter
B. Chronic fibrous thyroiditis
C. Subacute thyroiditis, thyrotoxic stage
D. Toxic adenoma of the thyroid gland
E. Autoimmune thyroiditis, euthyroid state

A girl of 8 years, went to the doctor with complaints of excessive sweating, palpitations, weight
loss, growth anticipation compared to her peers. The onset of illness is associated with overload
during exercise. Pulse - 120 beats / min., Blood pressure 140/60 mm Hg. Art. Hand tremor.
The thyroid gland is enlarged to the 1st century, elastic. What is the likely diagnosis?
A. Diffuse toxic goiter
B. Giantism
C. Physical fatigue
D. Autoimmune thyroiditis, euthyroid state
E. Thyrotoxic adenoma

Patient D, 24 years old, went to the doctor complaining of neck enlargement, tremor of the
hands, palpitations and increased excitability that arose from excessive insolation. The thyroid
gland of the second century, elastic, auscultating over it is heard a gross systolic murmur.
Specify the most likely diagnosis.
A. Diffuse toxic goiter
B. Chronic fibrous thyroiditis
C. Subacute thyroiditis
D. Toxic adenoma of the thyroid gland
E. Autoimmune thyroiditis, euthyroid state

Patient N., 50 years old, complains of headache, tearing, the feeling of sand in the eyes,
sluggishness, sometimes - diplopia. He became ill suddenly after suffering from the flu. He was
treated by an optometrist, but to no effect. Objectively: skin moist, heart rate 92 beats / min,
blood pressure 140/75 mm Hg. Art. Pronounced bilateral exophthalmos, conjunctivitis,
movements of the eyeballs are limited. The thyroid gland is diffusely enlarged And in the st.,
Smooth not painful. Signs of which pathology is dominant in the clinical picture?
A. Autoimmune (thyrotoxic) ophthalmopathy
B. Iodine deficiency state
C. Subacute thyroiditis
D. Autoimmune thyroiditis
E. Thyrotoxicosis

Patient S. after experiencing stress experienced general weakness, weight loss of 12 kg with
good appetite, irritability, palpitations, tremor of the fingers. Objectively: heart rate 120 beats /
minute, blood pressure 150/65 mm Hg. Art., skin moist, warm, light bilateral exophthalmos.
The thyroid gland enlarged to the second century, homogeneous, not painful. Make a
preliminary diagnosis.
A. Diffuse toxic goiter
B. Endemic goiter
C. Thyrotoxic adenoma
D. Autoimmune thyroiditis, euthyroid state
E. Subacute thyroiditis, thyrotoxic stage

Patient S., 52, complains of increased irritability, emotional lability, increased sweating,
palpitations, tremor of the fingers and the whole body. Objectively: heart rate 110 beats / min,
arrhythmic, blood pressure 150/60 mm Hg Thyroid enlarged to IV century. at the expense of an
elastic, clearly defined node with a diameter of 2 cm in the left lobe. Make a preliminary
diagnosis.
A. Thyrotoxic adenoma
B. Endemic nodular goiter
C. Diffuse toxic goiter
D. Autoimmune thyroiditis, euthyroid state
E. Subacute thyroiditis, thyrotoxic stage

A patient with suspected hypercalcemic crisis was admitted to the ICU. What should be done
first?
A. Rehydration, administration of calcitonin and bisphosphonates
B. Forced diuresis, administration of magnesium preparations
C. Rehydration, administration of glucocorticoids
D. Forced diuresis, introduction of cardiac glycosides
E. Rehydration, anticoagulants, antiarrhythmic agents

In the intensive care unit a patient with upper and lower extremity convulsions, an "obstetrician's
hand", "a horse's foot" was admitted. On examination the skin is pale, moist, pulse 80 beats /
min, blood pressure 150/100 mm. Mm. Art. It is known that three weeks ago the patient
underwent thyroidectomy. What is your previous diagnosis?
A. Hypocalcaemic crisis
B. Epilepsy
C. Hypoglycemic coma
D. Ruler
E. Acute strychnine poisoning

Patient N., 50 years old, complains of headache, lacrimation, sand in the eyes, sluggishness,
sometimes diplopia. He became ill suddenly after suffering from the flu. He was treated by an
optometrist, but to no effect. Objectively: skin moist, heart rate 92 beats / min, blood pressure
140/75 mm Hg. Art. Pronounced bilateral exophthalmos, conjunctivitis, movements of the
eyeballs are limited. The thyroid gland is diffusely enlarged And in the st., Smooth not painful.
Signs of which pathology is dominant in the clinical picture?
A. Endocrine ophthalmopathy
B. Iodine deficiency state
C. Subacute thyroiditis
D. Autoimmune thyroiditis
E. Thyrotoxicosis

A patient with suspected hypercalcaemic crisis was admitted to the ICU. What should be done
first?
A. Forced diuresis, administration of magnesium and glucocorticoids
B. Rehydration, administration of calcitonin and bisphosphonates
C. Rehydration, introduction of cardiac glycosides
D. Rehydration, anticoagulants, antiarrhythmic agents
E. Captopril.

The patient is operated on about a diffuse toxic goiter. Before the operation, the heart rate is 96
beats / minute, blood pressure 125/70 mm Hg. Art. On the second day the condition worsened
sharply: the patient was agitated, the consciousness was darkened, the heartbeat, extreme
sweating, vomiting. Pulse 166 beats / minute, atrial fibrillation. HELL 80/40 mm Hg
Temperature 390 C. What is the most likely diagnosis?
A. Post-operative sepsis
B. Acute respiratory failure
C. Thyreotoxic crisis
D. Hypoparathyroidism
E. Acute cardiovascular failure.

Patient R., 32 years old, received complaints of weight loss, severe weakness, palpitations, limb
tremor, sweating. Objectively: the thyroid gland enlarged to the second century, at the expense
of all departments, elastic, not painful, blood pressure 180/60 mm Hg. Art., pulse 120 beats /
min. Bilateral ophthalmopathy of the III-IV centuries. Preliminary diagnosis:
A. Toxic adenoma of the thyroid gland
B. Chronic fibrous thyroiditis
C. Subacute thyroiditis, thyrotoxic stage
D. Diffuse toxic goiter
E. Autoimmune thyroiditis, euthyroid state.

A girl of 8 years, went to the doctor complaining of excessive sweating, palpitations, weight loss,
growth anticipation compared to her peers. The onset of illness is associated with overload
during exercise. Pulse - 120 beats / min., Blood pressure 140/60 mm Hg. Art. Hand tremor.
The thyroid gland is enlarged to the 1st century, elastic. What is the likely diagnosis?
A. Giantism
B. Diffuse toxic goiter
C. Physical fatigue
D. Autoimmune thyroiditis, euthyroid state
E. Thyrotoxic adenoma

Patient D, 24 years old, went to the doctor complaining of neck enlargement, tremor of the
hands, palpitations and increased agitation, which arose after excessive insolation. The thyroid
gland of the second century, elastic, auscultating over it is heard a gross systolic murmur.
Specify the most likely diagnosis.
A. Toxic adenoma of the thyroid gland
B. Chronic fibrous thyroiditis
C. Subacute thyroiditis
D. Diffuse toxic goiter
E. Autoimmune thyroiditis, euthyroid state.

The clinical signs of prolonged hypoparathyroidism are the following, except:


A. Dry skin.
B. Hair thinning.
C. Depression.
D. Angina pectoris.
E. Bone pain.

Patient S., 35 years old, suffers from diffuse toxic goiter. In this disease, the following
symptoms occur, except:
A. Krause
B. Rosenbach
C. Bera
D. Grefe
E. Mobius.

A 36-year-old woman has had diffuse toxic goiter. The clinical picture of this disease is
characterized by the following symptoms, except.
A. Tachycardia
B. Hyperthermia
C. Increased excitability
D. Psychomotor inhibition
E. Systolic hypertension.

A diffuse toxic goiter was found in a patient 36 years of age. The ocular manifestations of this
disease include the following symptoms, except:
A. Kocher
B. Grefe
C. Khvostek
D. Dalrymple
E. Mobius.

In hyperparathyroidism on the R − graph of the skull reveal:


A. Isolated osteoporosis of the skull vault
B. Isolated osteoporosis of the Turkish saddle wall
C. Calcification in areas of the basal ganglia
D. Symptom of рож empty ″ Turkish saddle
E. Cystic degeneration of the skull bones (fibro-cystic osteitis).

1. A 35-year-old woman who has recently had hypertension has serum potassium level of 2.7
mEq / l, plasma aldosterone (AP) level is 55 ng% (norm 1-6). The following studies revealed:
AP after saline infusion - 54 ng% (norm 1-8), after a 4-hour walk - 32 ng% (norm 4-31); serum
18-hydroxycorticosterone level is 108 ng% (norm <30). What is the most likely diagnosis?
A. Primary hyperaldosteronism (Conn syndrome)
B. Corticosteroma
C. Androsteroma
D. Pheochromocytoma
E. Corticosteroma.

2. Which statement will be incorrect about congenital adrenal cortex hyperplasia


A. The disease develops as a result of birth defects in the enzyme 21 hydroxylase
B. The disease develops as a result of birth defects in the enzyme 3β dehydrogenase
C. The disease develops as a result of birth defects in the enzyme 11 β hydroxylase
D. The disease develops as a result of birth defects of the enzyme 17α hydroxylase
E. The disease develops as a result of birth defects in the enzyme lactate dehydrogenase

3. Which hormone is produced in the retina of the adrenal cortex?


A. Cortisone
B. Adrenaline
C. Testosterone
D. Corticosterone
E. Norepinephrine}

4. Which of the diagnostic criteria is uncharacteristic of pheochromocytoma?


A. Increased urinary excretion of vanillylmagic acid
B. Decreases in blood catecholamines
C. Increase in ESR
D. Leukocytosis, eosinophilia, erythrocytosis
E. Conduction disorders and left ventricular hypertrophy on the ECG

5. Which statement will be true for Itsen-Cushing's syndrome?


A. Cortisol and ACTH levels are elevated
B. Cortisol and ACTH levels are reduced
C. Cortisol levels are elevated, ACTH is lowered
D. Cortisol levels are elevated, ACTH is lowered
E. Cortisol levels unchanged, ACTH elevated}

6. Which of the antihypertensive drugs is the drug of choice for treating hypertension with
pheochomocytoma?
A. Alpha blockers
B. Beta blockers
C. Clonidine
D. ACE inhibitors
E. Receptor antagonists for angiotensin II

7. Which statement would be incorrect for acute adrenal insufficiency?


A. Develops in pathology of the adrenal glands
B. May accompany severe violations of other organs and systems
C. It is accompanied by hypertension
D. It is accompanied by collapse and hypotension
E. It is accompanied by a decrease in cortisol, corticosterone, aldosterone

8. Which statement will be true for adipozogenital dystrophy?


A. Cortisol, ACTH, testosterone, progesterone levels are elevated
B. ACTH cortisol, testosterone, and progesterone levels decreased
C. Cortisol levels increased, ACTH, testosterone, progesterone reduced
D. ACTH cortisol levels increased, testosterone lowered, progesterone lowered
E. Cortisol levels unchanged, ACTH of testosterone, progesterone elevated}

Patient K., aged 54, complained of severe headache, heartache, severe general muscle weakness,
seizures, thirst, rapid urination in large portions. Laboratory studies: ZAK - without features,
ZAS - alkaline reaction, proteinuria, isohypostenuria, biochemical analysis of urine -
hyperkaliuria, hyponatriuria, increased daily excretion of aldosterone. What is your diagnosis?
A. Hyperaldosteronism
B. Hypoparathyroidism
C. Diabetes mellitus
D. Addison's Disease
E. Chronic renal failure}

10. Konn's disease was suspected in a patient of 46 years. What blood test should you do to
confirm your diagnosis?
A. Blood tests for aldosterone
B. Blood tests for cortisone
C. Blood tests for calcium content
D. Research on cholesterol
E. General blood test}

1. Patient, 46 years old, for the last 2.5 years complains of high blood pressure, polyuria,
headache, general weakness, sweating, tremor. Objectively: AO = 175/100 - 210/115 mmHg, Рs
= 120 beats / min. Hypotensive drugs are not effective .. The level of Na = in the blood is 150
mmol / l. K = 2.8 mmol / L. What is the most likely diagnosis?
A. Essential hypertension.
B. Aldosteroma.
C. Pheochromocytoma.
D. Renal hypertension.
E. Cushing's disease.

2. Which statement would be incorrect for acute adrenal insufficiency?


A. Develops in pathology of the adrenal glands
B. May accompany severe violations of other organs and systems
C. It is accompanied by hypertension
D. It is accompanied by collapse and hypotension
E. It is accompanied by a decrease in cortisol, corticosterone, aldosterone

3. Which statement will be true for Itsen-Cushing's syndrome?


A. Cortisol and ACTH levels are elevated
B. Cortisol and ACTH levels are reduced
C. Cortisol levels are elevated, ACTH is lowered
D. Cortisol levels are elevated, ACTH is lowered
E. Cortisol levels unchanged, ACTH elevated}

4. Which of the following would be incorrect about congenital adrenal cortex hyperplasia
A. Boys have delayed physical and sexual development
B. Manifestations of boys are premature puberty
C. The manifestations of girls are false hermaphroditism
D. The development of hypertension is possible
E. Blood levels of corticotropin , testosterone, 17-hydroxyprogesterone increase

5. The causes of hyperaldosteronism may be all of the following except:


A. Hormone-active tumor of the adrenal gland (aldosteroma)
B. Bilateral hyperplasia of the glomerular zone of the adrenal glands
C. Bilateral hyperplasia of the retinal mesh
D. Long-term use of medicines (diuretics, contraceptives)
E. May develop in some kidney diseases

6. Which statement will be correct for changes in blood in chronic adrenal insufficiency?
A. Hypernatremia, hypercholesterolemia, hyperkalemia
B. Hyponatremia, hypercholesterolemia, hypokalemia
C. Hyponatremia, hypocholesterolemia, hypokalemia
D. Hyponatremia, hypercholesterolemia, hyperkalemia
E. Hypernatremia, hypocholesterolemia, hypokalemia

7. Which statement will be true for secondary chronic adrenal insufficiency?


A. Cortisol levels, ACTH elevated
B. Cortisol levels, ACTH reduced
C. Cortisol levels are lowered, ACTH increased
D. Cortisol levels are elevated, ACTH is lowered
E. Cortisol levels unchanged, ACTH elevated

8. Patient D., 45, complains of general weakness, headache, muscle twitching, palpitations,
thirst, increased blood pressure. Biochemical examination of the blood revealed: hypocalcemia,
hypomagnesemia, hypernatremia. What is your diagnosis?
A. Primary aldosteronism
B. Chronic adrenal insufficiency
C. Thyrotoxic goiter
D. Pheochromocytoma
E. Hypoparathyroidism

9. Which of the hormones is produced in the glomerular zone of the adrenal cortex?
A. Cortisone
B. Adrenaline
C. Testosterone
D. Aldosterone
E. Estrogens

10. What clinical symptom is uncharacteristic of hyperaldosteronism?


A. Hypoisostenuria, polyuria, nocturia
B. Paralysis and paresis, muscle aches
C. Systolic and diastolic hypertension
D. Signs of hyperkalemia on ECG
E. Myasthenic syndrome

1. Girl 1.5 years, on examination, excess fat on the face, abdomen, stretch marks on the
extremities. The baby was born healthy. Total: AT - 145/95 mmHg, Ps - 140 beats / min.
Laboratory data: ACTH - 38.6 mg / ml (N 8.3 - 57.8), testosterone 22.1 nmol / l, estradiol 109.8
pg / ml, cortisol 454.8 ng / ml. CT of the internal organs: in the projection of the left adrenal
gland revealed a circular formation with clear contours. What is the diagnosis?
A. Pheochromocytoma.
B. Obesity.
C. Corticosteromas (glucosteromas).
D. Androsteroma.
E. Aldosteroma.

2. Which statement will be true for changes in blood in chronic adrenal insufficiency?
A. Hypernatremia, hypercholesterolemia, hyperkalemia
B. Hyponatremia, hypercholesterolemia, hypokalemia
C. Hyponatremia, hypocholesterolemia, hypokalemia
D. Hyponatremia, hypercholesterolemia, hyperkalemia
E. Hypernatremia, hypocholesterolemia, hypokalemia

3. Which statement would be incorrect for acute adrenal insufficiency?


A. Develops in pathology of the adrenal glands
B. May accompany severe violations of other organs and systems
C. It is accompanied by hypertension
D. It is accompanied by collapse and hypotension
E. It is accompanied by a decrease in cortisol, corticosterone, aldosterone}

4. Which of the antihypertensive drugs is the drug of choice for treating hypertension with
pheochomocytoma?
A. Alpha blockers
B. Beta blockers
C. Clonidine
D. ACE inhibitors
E. Receptor antagonists for angiotensin II

5. Patient A., 57, was admitted to the endocrinological department with headaches, general
weakness, increased blood pressure, which is bad for correction, dry mouth, thirst, weight loss,
dry cough. Hypertensive disease is more than 10 years. On examination: thin, pale skin, dry, in
the lungs - vesicular breathing, heart tones rhythmic, heart rate - 88 beats per minute, blood
pressure 180/110 mm Hg Laboratory data: blood sugar - 7.8 mmol / l, ZAK and ZAS without
features. Adrenal ultrasound - enlarged left adrenal gland, CT of the adrenal gland - volumetric
formation of the left adrenal gland. What is your healing tactic?
A. Adrenalectomy unilateral
B. Phentolamine in / in to stabilization of blood pressure
C. Adrenalectomy bilateral
D. ß-blockers
E. Seduxen

6. Patient P., 32, suffering from Addison's disease for 3 years. Hospitalized with signs of adrenal
crisis. On examination: glucose level - 2.5 mmol / l. K + - 7.8 mmol / L. On the ECG -
reduction of the voltage of the teeth and slowing of conduction. What medications should be
used urgently?
A. Insulin - 60 U
B. Hydrocortisone succinate 100-150 mg / i jet
C. Camphor 2 ml
D. Glucose 5% 100 ml
E. Panangin 10 ml}
7. Patient M., 42, received complaints of severe weakness, anorexia, severe nausea, vomiting,
which does not bring relief, dizziness. Symptoms began to develop within 1-2 days after
cortisone was discontinued. Objectively: blood pressure 65/20 mmHg, pulse - 148 beats / min,
filamentous, skin turgor lowered, color pale, disturbed breathlessness, surface breathing. In the
blood: HB- 166 g / l, marked hyponatremia, hypochloremia, hyperkalemia. In urine -
proteinuria, leukocyturia, microhematuria Do you have a diagnosis?
A. Uremic coma
B. Brain Coma
C. Hypoglycemic coma
D. Acute cardiovascular failure
E. Acute adrenal insufficiency

8. How is glucocorticoid administered twice daily how is the daily dose distributed between
doses?
A. Half in the morning, half in the evening
B. Two-thirds in the morning, one-third at lunch
C. One third in the morning, two thirds in the evening
D. Half in the morning, half in the afternoon
E. A third at lunch, two thirds in the evening

Which statement will be true for secondary chronic adrenal insufficiency?


A. Cortisol levels, ACTH elevated
B. Cortisol levels, ACTH reduced
C. Cortisol levels are lowered, ACTH increased
D. Cortisol levels are elevated, ACTH is lowered
E. Cortisol levels unchanged, ACTH elevated

10. Which statement will be incorrect about congenital adrenal cortical hyperplasia
A. Boys have delayed physical and sexual development
B. Manifestations of boys are premature puberty
C. The manifestations of girls are false hermaphroditism
D. The development of hypertension is possible
E. Blood levels of corticotropin, testosterone, 17-hydroxyprogesterone increase}

1. Girl, 1.4 years old, under examination, excess fat on the face, abdomen, stretch marks on the
hips. The baby was born healthy. Objectively: AT- 140/95 mmHg, Ps - 130 beats / min,
abdomen somewhat painful. Laboratory data: ACTH - 38.6 mg / ml, cortisol 454.8 ng / ml. CT
scan of the head within the age norm. CT of the abdomen: in the projection of the left adrenal
gland revealed a circular formation with clear contours. What are the treatment tactics?
A. Unilateral adrenalectomy.
B. Bilateral adrenalectomy.
C. Hypophysectomy.
D. Nephrectomy.
E. Laparoscopic adrenalectomy.

2. Which of the antihypertensive drugs is the drug of choice for treating hypertension with
pheohomocytoma?
A. Alpha blockers
B. Beta blockers
C. Clonidine
D. ACE inhibitors
E. Receptor antagonists for angiotensin II
3. Patient L., 41, delivered by ambulance with complaints of a sharp general weakness, weight
loss, lack of appetite, nausea, vomiting, abdominal pain. On examination: hyperpigmentation of
the skin, blood pressure 70/45 mm Hg, bradycardia. In additional studies: reduced content of
aldosterone, cortisone in the blood, decreased excretion of 17-COP and 17-ACS with urine,
hyponatremia, hypochloremia, hypokalemia. What treatment measures should be taken?
A. Vitamin C Purpose
B. Purpose of glucocorticoids, mineralocorticoids, diets with high salt content
C. Purpose of prednisolone
D. Purpose of aldosterone
E. Purpose of insulin}

4. Patient V., 30, complains of a periodic sudden increase in blood pressure up to 280/180 mm
Hg, accompanied by sweating, trembling of the whole body, and fear of death. After
hypertensive crisis there is polyuria, sharp weakness. Between the crises of AO 120/80 mmHg
What additional research should be done to diagnose?
A. Investigation of catecholamines content of blood and urine
B. Investigation of the content of triiodothyronine and thyroxine
C. Investigation of acetone content in urine
D. Research on ACTH and cortisone content
E. Blood osmolarity studies

5. Which of the diagnostic criteria is uncharacteristic of pheochromocytoma?


A. Increased urinary excretion of vanillylmagic acid
B. Decreases in blood catecholamines
C. Increase in ESR
D. Leukocytosis, eosinophilia, erythrocytosis
E. Conduction disorders and left ventricular hypertrophy on the ECG}

The patient 46 suspected Conn”s syndrome. What blood tests should be done to confirm the
diagnosis?
A. The blood test for cortisone
B. Blood tests for calcium
C. General analysis of blood
D. Research on cholesterol
E. The blood test for aldosterone

7. 49 G. sick, sick Adison disease for 5 years. Gets prednisolone daily. After the transferred flu
the condition of the patient has sharply worsened, pains in the heart area, weakness, dizziness,
nausea, diarrhea. Pulse - 110 / min., BP - 60/30 mm Hg.St. What diagnosis is most probable?
A. Angina
B. Adrenal crises
C. Acute myocardial infarction
D. Acute gastroenteritis
E. Acute myocarditis

8. Patient G., 32 g., was admitted with complaints of increased fatigue, decreased appetite,
increased pigmentation in the area of open parts of the body, palms, increased cyanosis, weight
loss, nausea, vomiting. The symptoms began to grow within 1-2 weeks after acute poisoning.
Objectively, blood pressure 60/30 mmHg, pulse was 140 beats/min, skin turgor reduced, color-
enhanced dark pigmentation of elbows, scars, skin folds on the palms. Blood severe
hyponatremia, hipochloremia, hyperkalemia. Blood glucose 4.3 mmol/l. Your diagnosis?
A. The Introduction of 500 ml of 5% district glucose
B. Introduction of 4% sodium bicarbonate and 2.5 ml/kg
C. The introduction of 25 mg of hydrocortisone intramuscularly gemactin
D. Introduction of a 5% solution of ascorbic acid
E. The Introduction of 500 ml of 0.9% sodium chloride

9. When two-a-day use of glucocorticoids how to divide the daily dose between meals?
A. Half morning, half evening
B. Two-thirds in the morning, a third at lunch
C. The third morning, two thirds in the evening
D. Half in the morning half in the afternoon
E. A third in the afternoon, two-thirds of the evening

10. Patient Ya., 27 g., complains of the excess weight of the body, shortness of breath, violation
of ovarian-menstrual cycle, has been sick since 3 years. Family history is not burdened.
Objective: BMI of 36.2 kg/m2, obesity dysplastic, with a predominance in the abdomen, thin
limbs, hypertrichosis. In the area of the abdomen, inguinal areas multiple Burgundy striae. AD –
160/100, pulse – 96 beats/min. What kind of diagnostic test it is advisable to spend for the
diagnosis of disease and syndrome Itsenko-Kushinga?
A. Small dexametasona sample
B. Most dexametasona sample
C. Trial with clonidine
D. Test cerucalum
E. Test with insulin}

1. A 35-year-old woman who has recently had hypertension has serum potassium level of 2.7
mEq / l, plasma aldosterone (AP) level is 55 ng% (norm 1-6). The following studies revealed:
AP after saline infusion - 54 ng% (norm 1-8), after a 4-hour walk - 32 ng% (norm 4-31); serum
18-hydroxycorticosterone level is 108 ng% (norm <30). What is the most likely diagnosis?
A. Primary hyperaldosteronism (Conn syndrome)
B. Corticosteroma
C. Androsteroma
D. Pheochromocytoma
E. Corticosteroma.

2. Which statement will be incorrect about congenital adrenal cortex hyperplasia


A. The disease develops as a result of birth defects in the enzyme 21 hydroxylase
B. The disease develops as a result of birth defects in the enzyme 3β dehydrogenase
C. The disease develops as a result of birth defects in the enzyme 11 β hydroxylase
D. The disease develops as a result of birth defects of the enzyme 17α hydroxylase
E. The disease develops as a result of birth defects in the enzyme lactate dehydrogenase

3. Which hormone is produced in the retina of the adrenal cortex?


A. Cortisone
B. Adrenaline
C. Testosterone
D. Corticosterone
E. Norepinephrine}

4. Which of the diagnostic criteria is uncharacteristic of pheochromocytoma?


A. Increased urinary excretion of vanillylmagic acid
B. Decreases in blood catecholamines
C. Increase in ESR
D. Leukocytosis, eosinophilia, erythrocytosis
E. Conduction disorders and left ventricular hypertrophy on the ECG
5. Which statement will be true for Itsen-Cushing's syndrome?
A. Cortisol and ACTH levels are elevated
B. Cortisol and ACTH levels are reduced
C. Cortisol levels are elevated, ACTH is lowered
D. Cortisol levels are elevated, ACTH is lowered
E. Cortisol levels unchanged, ACTH elevated}

6. Which of the antihypertensive drugs is the drug of choice for treating hypertension with
pheochomocytoma?
A. Alpha blockers
B. Beta blockers
C. Clonidine
D. ACE inhibitors
E. Receptor antagonists for angiotensin II

7. Which statement would be incorrect for acute adrenal insufficiency?


A. Develops in pathology of the adrenal glands
B. May accompany severe violations of other organs and systems
C. It is accompanied by hypertension
D. It is accompanied by collapse and hypotension
E. It is accompanied by a decrease in cortisol, corticosterone, aldosterone

8. Which statement will be true for adipozogenital dystrophy?


A. Cortisol, ACTH, testosterone, progesterone levels are elevated
B. ACTH cortisol, testosterone, and progesterone levels decreased
C. Cortisol levels increased, ACTH, testosterone, progesterone reduced
D. ACTH cortisol levels increased, testosterone lowered, progesterone lowered
E. Cortisol levels unchanged, ACTH of testosterone, progesterone elevated}

Patient K., aged 54, complained of severe headache, heartache, severe general muscle weakness,
seizures, thirst, rapid urination in large portions. Laboratory studies: ZAK - without features,
ZAS - alkaline reaction, proteinuria, isohypostenuria, biochemical analysis of urine -
hyperkaliuria, hyponatriuria, increased daily excretion of aldosterone. What is your diagnosis?
F. Hyperaldosteronism
A. Hypoparathyroidism
B. Diabetes mellitus
C. Addison's Disease
D. Chronic renal failure}

10. Konn's disease was suspected in a patient of 46 years. What blood test should you do to
confirm your diagnosis?
F. Blood tests for aldosterone
G. Blood tests for cortisone
A. Blood tests for calcium content
B. Research on cholesterol
C. General blood test}

A patient, 36, complained to the endocrinologist for complaints of weight loss, dizziness,
dizziness, dyspeptic symptoms, muscle and joint pain. Objectively, the skin is dry. Height 162
cm, body weight 45.3 kg, BMI 17.3 kg / m2. AO 100/70 mm Hg, Ps 92, rhythmic. No edema.
ZAC: Hb 92 g / l, erythrocytes 3.1 * 1012 / l, leukocytes 3.7 * 109 / l, platelets 162 * 109 / l.
Blood sugar 4.0 mmol / l, K + 5.2 mmol / l, Na + 130 mmol / l. What examinations do you need
to make to diagnose it?
A. Urine cortisol, ACTH, aldosterone
B. Glucose tolerance test, blood cortisol, ACTH
C. FSH, LH, ACTH
D. Excretion of 17-CS, 17-ACS with daily urine
E. Glucose tolerance test, NOMA index, DHEA-C}

2. To confirm the diagnosis of diabetes mellitus, the following studies should be performed:
A. Urine planting on the flora
B. Urine analysis by Nechiporenko
C. General urinalysis
D. Urine analysis according to Adis-Kakovsky
E. Urine analysis of Zymnitsky}

The lesions of which areas of the brain most often lead to obesity?
A. The cerebellum
B. Cortex
C. Ventromedial nuclei of the hypothalamus
D. Pituitary gland
E. Subcortical structures

4. The causes of hyperprolactinemia include all but:


A. Primary hypothyroidism
B. Medicinal galactorrhea
C. Drug related
D. Macroprolactinoma
E. Occurrence of autoantibodies to the neurosecretory cells of the supraoptic nucleus of the
anterior part of the hypothalamus}

For carrying out small dexamethasone test it is necessary to appoint dexamethasone in a dose:
A. 0.5 mg 4 times a day for 3 days.
B. 1.0 mg 4 times a day for 3 days.
C. 2.0 mg 4 times a day for 3 days.
D. 0.5 mg 3 times a day for 4 days.
E. 1.0 mg 3 times a day for 4 days}

6. Patient for 30 years complains of thirst, increased urination, weakness. The disease is
associated with a traumatic brain injury. Urine specific gravity 1002. What disease can be
thought of?
A. Diabetes mellitus.
B. Diabetes mellitus.
C. Chronic nephritis.
D. Neurogenic polydipsia.
E. Enuresis.

7. Which of the following are characteristic of testicular femoral syndrome?


A. Positive sex chromatin.
B. Low testosterone.
C. Female karyotype.
D. The reproductive gland is the egg.
E. External genitalia – female}

Identify an endocrine disease that is accompanied by obesity:


A. Itsenko-Cushing's disease.
B. Addison's disease.
C. Hyperparathyroidism.
D. Toxic goiter.
E. Type I diabetes}

9. What areas of the brain are most likely to cause obesity?


A. The cerebellum.
B. Cortex.
C. Pituitary gland.
D. Subcortical structures.
E. Ventromedial nuclei of the hypothalamus.

10. A 13-year-old boy has bilateral gynecomastia. This can be the case when:
A. Klinefelter syndromes.
B. Raiffeisen Syndromes.
C. Corticosteromas.
D. Physiological puberty.
E. All of the above states.

A patient, 36, complained to the endocrinologist for complaints of weight loss, dizziness,
dizziness, dyspeptic symptoms, muscle and joint pain. Objectively, the skin is dry. Height 162
cm, body weight 45.3 kg, BMI 17.3 kg / m2. AO 100/70 mm Hg, Ps 92, rhythmic. No edema.
ZAC: Hb 92 g / l, erythrocytes 3.1 * 1012 / l, leukocytes 3.7 * 109 / l, platelets 162 * 109 / l.
Blood sugar 4.0 mmol / l, K + 5.1 mmol / l, Na + 130 mmol / l. What screening test is
appropriate for a patient to make a diagnosis?
A. Glucose tolerance test
B. ACTH-stimulating sample
C. Test with vespiron
D. Small dexamethasone sample
E. Large dexamethasone sample

2. Diabetes mellitus caused by a violation of secretion:


A. Corticotropin
B. Thyrotropin
C. Vasopressin
D. Somatotropin
E. Chorionic gonadotropin}

What are the clinical features of hypothalamic forms of obesity?


A. Slow weight gain
B. Presence of vegetative disorders
C. Even distribution of adipose tissue
D. Dysplastic obesity
E. Weight loss}

A patient of 23 years complains of disappearance of menstruation, headache. Genital


infantilism was revealed after examination by a gynecologist. Against the background of ethinyl
estradiol (0.05 mg twice daily), menstruation resumed, but disappeared after discontinuation of
treatment. The level of prolactin is 400 mcg / l (norm 2–15 mcg / l), MRI - macadenoma d 15
mm, suprasellar growth. Choose a diagnosis:
A. Hyperprolactinemic hypogonadism
B. Autonomous production of prolactin by the pituitary gland
C. Syndrome of "empty" Turkish saddle
D. Lawrence-Moon-Bardet-Biddle syndrome
E. Hypothalamic nanism}

5. Women of midget age include persons whose height is below:


A. 100 cm
B. 110 cm
C. 120 cm
D. 130 cm
E. 140 cm}

A small dexamethasone test is carried out for the purpose of diagnosis:


A. Itsen-Cushing's Diseases
B. Itsenko-Cushing's syndrome.
C. Connor syndrome (primary hyperaldosteronism)
D. Prolactinomas
E. Androsteromas

What are the characteristics of congenital adrenal cortex dysfunction:


A. Increased excretion of 17-COP with urine.
B. Negative sex chromatin.
C. Short neck with winglike folds.
D. Bilateral enlargement of the testicles.
E. Hypercalcemia}

8. What are the symptoms characteristic of hypothalamic obesity:


A. Increased appetite, periodic thirst
B. Diffuse obesity with stretch marks.
C. Dysplastic obesity with stretch marks.
D. Sympathoadrenal crises.
E. Violation of thermoregulation}

Which of the following is not typical for obese people?


A. Furunculosis
B. Mycosis of the skin
C. Phlegmon
D. Striae
E. Muscle enhancement

10. Which of the symptoms is characteristic of Klinefelter syndrome?


A. Chromosomal set 47 of XXU
B. Negative sex chromatin
C. Infertility
D. Eunuchid type of body
E. All of the above

Patient, 39, complains of weight loss, dizziness, dizziness, dyspeptic symptoms, muscle and joint
pain. Objectively, the skin is dry. Height 166 cm, body weight 47.3 kg, BMI 17.1 kg / m2. AO
90/60 mm Hg, Ps 98, rhythmic. No edema. Blood sugar 4.0 mmol / l, K + 5.1 mmol / l, Na +
128 mmol / l. In order to clarify the diagnosis, a stimulating trial was performed: for 30-60
minutes. after administration of 250 mg of corticotropin, the following results were obtained:
cortisol 160 and ACTH decreased, aldosterone within normal limits. Assigned replacement
therapy with a gradual reduction in dosage to supportive: cortef (hydrocortisone) 20 mg in the
morning and 20 mg at 19.00. Is the proposed supportive therapy regimen rational?
A. No, it is prudent to administer cortef 10 mg after breakfast and 5 mg after lunch (no later
than 5 pm) for maintenance therapy
B. Yes, rational supportive therapy is prescribed
C. No, it is advisable for the patient to administer cortineff (fludrocortisone) 0.2 mg three
times a week
D. No, it is reasonable to administer cortef 5 mg after breakfast and 10 mg after lunch (no
later than 5 pm) for maintenance therapy.
E. No, it is prudent to administer cortef 10 mg after breakfast and 5 mg at 19.00 for
maintenance therapy

In the treatment of diabetes insipidus, it is necessary to appoint:


A. Desmopressin
B. Cortisol
C. Sulfonamides
D. Furosemide
E. Penicillin

An examination of a patient with acromegaly reveals all of the following except for:
A. Backlog in physical and sexual development
B. Pituitary adenoma
C. Osteoporosis
D. Cardiomegaly
E. Splhomegaly

The cause of Itsenko-Cushing's disease is most often the primary lesion:


A. Pituitary gland
B. The hypothalamus
C. Adrenal glands
D. Sexual glands
E. Thyroid gland

Patient for 30 years complains of thirst, increased urination, weakness. The disease is associated
with a traumatic brain injury. Urine specific gravity 1002. What disease can be thought of?
A. Diabetes insipidus.
B. Diabetes mellitus
C. Chronic nephritis.
D. Neurogenic polydipsia.
E. Enuresis

A large dexamethasone test is performed to:


A. Differential diagnosis of the disease and Itsen-Cushing's syndrome
B. Diagnosis of Itsenko-Cushing's syndrome
C. Diagnosis of Itsenka-Cushing's disease
D. Diagnosis of Connor syndrome
E. Differential diagnosis of acromegaly and hypothalamic obesity

In terms of biochemical structure, androgens belong to the following group:


A. Steroids.
B. Polypeptides.
C. Proteins.
D. Glucopeptides.
E. Derivatives of amino acids

Name a sign not characteristic of Itsen-Cushing's disease:


A. Hyperostrogeny.
B. Hypertensive syndrome.
C. Hypocalcaemia.
D. Hyperglycemia.
E. Hyperphosphatemia

What are the changes in the gastrointestinal tract that are not typical for obese people?
A. Diarrhea.
B. Expansion of hemorrhoidal veins.
C. Musculoskeletal hypotension
D. Constipation
E. Enhanced cholesterol formation.

Which statement is incorrect for hypergonadotropic hypogonadism?


A. Decrease of gonadotropin levels.
B. Reduced testosterone production.
C. Increased levels of gonadotropins.
D. Infertility.
E. Increased levels of estrogen, progesterone

The patient, 36, went to the doctor complaining of an irregular cycle, frequent delays, the last
menstruation more than 50 days ago. After exclusion of pregnancy, a cyclic hormonal test was
performed: the result is negative. Interpret the result.
A. Progesterone deficiency
B. Excess of androgens
C. Estrogen deficiency
D. Uterine form of amenorrhea
E. Combined estrogen-gestagenic deficiency

The physiological effect of vasopressin is manifested by:


A. Intensification of Na excretion
B. Increased diuresis
C. Enhanced water reabsorption in the distal tubules
D. Uterine spasm
E. Increased glucose reabsorption

What bone changes are not typical for patients with acromegaly?
A. Arthropathy
B. Bone deformity
C. Osteoporosis
D. Fibrous bone dysplasia
E. Osteochondrosis

The patient was diagnosed with: acromegaly, active form; intracellular pituitary adenoma
(somatotropinoma) without impaired visual function. Euthyroid, diffuse grade II goiter. Choose
the primary treatment for this clinical case.
A. Bromocriptine - 2.5 mg twice daily
B. Cabergoline - 0.5 mg once a week
C. Cabergoline - 0.5 mg once daily
D. Transphenoidal removal of pituitary adenoma
E. Octreotide - 0.05 mg n / w twice daily

A woman of 26 years has obesity: the location of fat mainly on the shoulders, torso. The
menstrual cycle is broken, there is hirsutism. On the shoulders, lateral parts of the abdomen,
thighs cyanotic strips of skin stretch. Which diagnosis is most likely?
A. Alimentary obesity.
B. Hypothalamic syndrome of the pubertal period.
C. Itsenko-Cushing's disease.
D. Hypoovarian obesity.
E. Hypothyroid obesity.

At Itsenka-Cushing's normal level of aldosterone is observed:


A. In 10% of cases.
B. In 25% of cases.
C. In 50% of cases.
D. In 75% of cases.
E. In all patients.

Determine which cells in the bodybrain synthesize estrogens:


A. Chromaffin cells.
B. Yellow body lutein cells.
C. Cells of the glomerular zone of the adrenal glands.
D. Cells of the inner lining of the follicle.
E. Cylindrical cells of the functional layer of the endometrium.

A small dexamethasone test is performed to:


A. Differential diagnosis of the disease and Itsen-Cushing's syndrome.
B. Diagnosis of Itsenka-Cushing's disease.
C. Diagnosis of the Itsenko-Cushing syndrome.
D. Exclusion of the Itsen-Cushing syndrome.
E. Diagnosis of corticosteroma.

What changes in the cardiovascular system are not typical of obese people?
A. Hypertension
B. Myocardiodystrophy
C. Symptom of sinus node weakness
D. Exertion on exertion
E. Angina pectoris

Which statement is correct for hypogonadotropic hypogonadism


A. Increased levels of gonadotropins
B. Reduced testosterone production
C. Reduction of gonadotropin levels
D. Infertility
E. Increased levels of estrogen, progesterone

A 33-year-old female patient was admitted to the endocrinologist for complaints of dizziness,
dizziness, decreased blood pressure, weight loss, dyspeptic symptoms, muscle and joint pain. In
order to clarify the diagnosis, a stimulating trial was conducted: for 30-60 min. after
administration of 250 mg of corticotropin the following results were obtained: cortisol level
decreased, ACTH level at the lower limit of normal, aldosterone within normal limits. Make the
right diagnosis.
A. Primary adrenal insufficiency
B. Adrenogenital syndrome
C. Secondary adrenal insufficiency
D. Alimentary dystrophy
E. Blitzard Syndrome

What characteristic can be found in the general analysis of urine of a patient with diabetes
mellitus?
{= Low specific gravity
~ High proteinuria
~ Macrogematuria
~ Microhematuria
~ Leukocyturia}

With regard to acromegaly, the following statement is incorrect:


A. Hypersecretion of somatotropin
B. Frequent hypotension
C. Increased skull pneumatization
D. Impaired glucose tolerance
E. Skull thickening}

4. One of the statements is not admissible in relation to the clinical picture of hypopituitarism:
A. Weight loss
B. Hypogonadism
C. Hypothyroidism
D. Hypocorticism
E. Hirsutism}

5. For the diagnosis of pituitary nanism, somatotropic function should be studied. To this end, a
stimulation test with clonidine. Select the correct time of blood collection (in minutes).
A. 0–30–60
B. 0-30-60-90
C. 0–30–60–90–120
D. 0–30–60–90–120–150
E. 0-30-90}

Patient M., complained of a sharp weight gain (30 kg in 2 years), periodic headache, decreased
potency. Objectively: height 170 cm, weight 114 kg. Fat deposition mainly on the chest,
abdomen. On the shoulders, abdomen and thighs of the stretch of blue. Heart tones weakened,
art. pressure 170/100 mm Hg. Art. Secondary sexual characteristics and genital organs are well
developed. Blood and urine tests without pathological changes. Fasting glycemia 7.6 mmol / l.
A. Obesity alimentary-constitutional
B. Hypothyroid obesity
C. Hypoovarian obesity
D. Itsenko-Cushing's disease
E. Lipomatosis

Patient C. Virile form of adrenogenital syndrome (partial insufficiency21-hydroxylase).


Identify a drug that is appropriate for the patient?
{= Prednisone.
~ Progesterone.
~ Dostinex.
~ Chorionic gonadotropin.
~ Bromocretin}

Determine the pathological condition caused by somatotropin deficiency:


A. Insomnia.
B. Delay in physical development.
C. Acceleration of mental development.
D. Acceleration of sexual development.
E. Hyperglycemia}

9. What are the changes of the musculoskeletal system that are not typical for obese people?
A. Osteporosis
B. Deforming osteoarthritis
C. Formation of Dupuytren's contracture
D. Valgus deformities of the feet
E. Degenerative changes in the spine}

10. Which of the medicines can cause gynecomastia


A. Spironolactone
B. Cimetedin
C. Sibazon
D. Estrogens
E. All of the above}

You might also like